Está en la página 1de 146

tica UES - FCCNNMM - Escuela de Matema GEOMETR IA I

Equipo de Dise no: Nahomy Jhopselyn Hern andez Cruz Gabriel Alexander Chicas Reyes Eduardo Arnoldo Aguilar Ca nas H ector Enmanuel Alberti Arroyo Humberto Alfonso Serme no Villalta Ernesto Am erico Hidalgo Castellanos Juan Agust n Cuadra Claudia Patricia Corcio L opez de Beltr an Carlos Mauricio Canjura Linares Oscar Armando Hern andez Morales Aar on Ernesto Ram rez Flores Manuel Alejandro Mundo Due nas Clara Victoria Regalado Francisco Asdrubal Hern andez Ramirez 7 de agosto de 2013

Indice
1. Geometr a 1.1. Segmentos . . . . . . . . . . . . . . . . . . . 1.1.1. Introducci on . . . . . . . . . . . . . . 1.1.2. L neas . . . . . . . . . . . . . . . . . 1.1.3. Rayos . . . . . . . . . . . . . . . . . 1.1.4. Segmento de recta . . . . . . . . . . 1.1.5. Punto medio de un segmento . . . . 1.1.6. Operaciones con segmentos colineales 1.1.7. Ejercicios . . . . . . . . . . . . . . . 1.1.8. Ejercicios . . . . . . . . . . . . . . . 1.2. Angulos entre paralelas. . . . . . . . . . . . 1.2.1. Denici on de Angulo . . . . . . . . . 1.2.2. Angulos entre rectas paralelas . . . . 1.2.3. Problemas . . . . . . . . . . . . . . . 1 . . . . . . . . . . . . . . . . . . . . . . . . . . . . . . . . . . . . . . . . . . . . . . . . . . . . . . . . . . . . . . . . . . . . . . . . . . . . . . . . . . . . . . . . . . . . . . . . . . . . . . . . . . . . . . . . . . . . . . . . . . . . . . . . . . . . . . . . . . . . . . . . . . . . . . . . . . . . . . . . . . . . . . . . . . . . . . . . . . . . . . . . . . . . . . . . . . . . . . . . . . . . . . . . . . . . . . . . . . . . . . . . . . . . . . . . . . . . . . . . . . . . . . . . . . . . . . . . . . . . . . . . . . . . . . . . . . . . . . . . . . . . . . 4 4 4 4 5 5 5 6 8 9 10 10 11 11

INDICE 1.3. Tri angulos. . . . . . . . . . . . . . . . . . . . . . 1.3.1. Clasicaci on de Tri angulos . . . . . . . . 1.3.2. Teoremas Fundamentales del Tri angulo . 1.3.3. Teorema de Pit agoras . . . . . . . . . . . 1.3.4. Rectas Notables de un tri angulo . . . . . 1.3.5. Distancia de un punto a una recta . . . . 1.3.6. Problemas . . . . . . . . . . . . . . . . . 1.4. Congruencia de Tri angulos. . . . . . . . . . . . 1.4.1. Criterios de Congruencia de tri angulos . 1.4.2. Teorema de la Base Media . . . . . . . . 1.4.3. Problemas . . . . . . . . . . . . . . . . . 1.5. Cuadril ateros . . . . . . . . . . . . . . . . . . . 1.5.1. Clasicaci on de Cuadril ateros . . . . . . 1.5.2. Paralelogramos . . . . . . . . . . . . . . 1.5.3. Rect angulos . . . . . . . . . . . . . . . . 1.5.4. Rombos . . . . . . . . . . . . . . . . . . 1.5.5. Trapezoides . . . . . . . . . . . . . . . . 1.5.6. Trapecios . . . . . . . . . . . . . . . . . 1.5.7. Problemas . . . . . . . . . . . . . . . . . 1.6. La Circunferencia . . . . . . . . . . . . . . . . . 1.6.1. Elementos de la circunferencia . . . . . . 1.6.2. Angulos en la circunferencia . . . . . . . 1.6.3. Cuadril ateros Conc clicos . . . . . . . . . 1.6.4. Rectas y Circunferencias tangentes a una 1.6.5. Problemas . . . . . . . . . . . . . . . . . 1.7. Semejanza de Tri angulos. . . . . . . . . . . . . . 1.7.1. Proporcionalidad . . . . . . . . . . . . . 1.7.2. Teorema de Thales . . . . . . . . . . . . 1.7.3. Criterios de Semejanza de Tri angulos . . 1.7.4. Potencia de Punto . . . . . . . . . . . . 1.7.5. Problemas . . . . . . . . . . . . . . . . . 1.8. Puntos y Rectas Notables del Tri angulo. . . . . 1.8.1. Medianas . . . . . . . . . . . . . . . . . 1.8.2. Mediatrices . . . . . . . . . . . . . . . . 1.8.3. Alturas . . . . . . . . . . . . . . . . . . 1.8.4. Bisectrices . . . . . . . . . . . . . . . . . 1.8.5. Problemas . . . . . . . . . . . . . . . . . 1.9. Problemas resueltos. . . . . . . . . . . . . . . . 1.9.1. Tri angulos . . . . . . . . . . . . . . . . . 1.9.2. Congruencia de Tri angulos . . . . . . . . 1.9.3. Cuadril ateros . . . . . . . . . . . . . . . 1.9.4. La Circunferencia . . . . . . . . . . . . . 1.9.5. Semejanza de Tri angulos . . . . . . . . . 1.9.6. Puntos y rectas notables del Tri angulo . . . . . . . . . . . . . . . . . . . . . . . . . . . . . . . . . . . . . . . . . . . . . . . . . . . . . . . . . . . . . . . . . . . . . . . . . . . . . . . . . . . . . . . . . . . . . . . . . . . . . . . . . . . . . . . . . . . . . . . . . . . . . . . . . . . . . . . . . . . . . . . . . . . . . . . . . . . . . . . . . . . . . . . . . . . . . . . . . . . . . . . . . circunferencia . . . . . . . . . . . . . . . . . . . . . . . . . . . . . . . . . . . . . . . . . . . . . . . . . . . . . . . . . . . . . . . . . . . . . . . . . . . . . . . . . . . . . . . . . . . . . . . . . . . . . . . . . . . . . . . . . . . . . . . . . . . . . . . . . . . . . . . . . . . . . . . . . . . . . . . . . . . . . . . . . . . . . . . . . . . . . . . . . . . . . . . . . . . . . . . . . . . . . . . . . . . . . . . . . . . . . . . . . . . . . . . . . . . . . . . . . . . . . . . . . . . . . . . . . . . . . . . . . . . . . . . . . . . . . . . . . . . . . . . . . . . . . . . . . . . . . . . . . . . . . . . . . . . . . . . . . . . . . . . . . . . . . . . . . . . . . . . . . . . . . . . . . . . . . . . . . . . . . . . . . . . . . . . . . . . . . . . . . . . . . . . . . . . . . . . . . . . . . . . . . . . . . . . . . . . . . . . . . . . . . . . . . . . . . . . . . . . . . . . . . . . . . . . . . . . . . . . . . . . . . . . . . . . . . . . . . . . . . . . . . . . . . . . . . . . . . . . . . . . . . . . . . . . . . . . . . . . . . . . . . . . . . . . . . . . . . . . . . . . . . . . . . . . . . . . . . . . .

INDICE . . . . . . . . . . . . . . . . . . . . . . . . . . . . . . . . . . . . . . . . . . . . . . . . . . . . . . . . . . . . . . . . . . . . . . . . . . . . . . . . . . . . . . . . . . . . . . . . . . . . . . . . . . . . . . . . . . . . . . . . . . . . . . . . . . . . 15 15 16 18 18 18 19 24 24 26 27 32 32 33 34 35 35 36 37 40 40 41 43 44 46 55 55 56 59 60 62 71 71 72 73 74 75 84 84 88 91 96 106 108

INDICE

INDICE

2. Trigonometr a 2.1. Angulos. . . . . . . . . . . . . . . . . . . . . . . . . . . . 2.1.1. Medici on de un a ngulo . . . . . . . . . . . . . . . 2.1.2. Terminolog a . . . . . . . . . . . . . . . . . . . . 2.2. Seno y Coseno . . . . . . . . . . . . . . . . . . . . . . . . 2.3. Identidades Trigonom etricas . . . . . . . . . . . . . . . . 2.3.1. Las identidades b asicas . . . . . . . . . . . . . . . 2.3.2. Identidades de suma y resta de angulos . . . . . . 2.3.3. Identidades de duplicaci on y linearizaci on . . . . 2.4. Tabla de Identidades Trigonom etricas. . . . . . . . . . . 2.5. Relaciones trigonom etricas en un tri angulo. . . . . . . . . 2.6. Una tangente especial. . . . . . . . . . . . . . . . . . . . 2.7. Ecuaciones trigonom etricas. . . . . . . . . . . . . . . . . 2.7.1. Ecuaciones de la forma A cos (x) + B sen (x) = C 2.8. Problemas. . . . . . . . . . . . . . . . . . . . . . . . . . . 2.9. Problemas aplicados. . . . . . . . . . . . . . . . . . . . .

. . . . . . . . . . . . . . .

. . . . . . . . . . . . . . .

. . . . . . . . . . . . . . .

. . . . . . . . . . . . . . .

. . . . . . . . . . . . . . .

. . . . . . . . . . . . . . .

. . . . . . . . . . . . . . .

. . . . . . . . . . . . . . .

. . . . . . . . . . . . . . .

. . . . . . . . . . . . . . .

. . . . . . . . . . . . . . .

. . . . . . . . . . . . . . .

. . . . . . . . . . . . . . .

. . . . . . . . . . . . . . .

. . . . . . . . . . . . . . .

119 119 120 121 121 123 123 124 125 126 127 127 128 131 132 139

1 Geometr a

1.
1.1.
1.1.1.

Geometr a
Segmentos
Introducci on

Una gura geom etrica es una combinaci on de puntos, l neas y planos. Los conceptos de punto, l nea y plano son conceptos primitivos, que no se denen, u nicamente pueden describirse. Por ejemplo, un punto es un objeto que solo tiene posici on. No tiene longitud, anchura ni espesor. Se trata pu es, de una idealizaci on matem atica. La Geometr a es la ciencia que estudia la posici on, forma y magnitud de las guras geom etricas. La geometr a plana estudia las guras cuyos puntos se encuentran en el mismo plano (tri angulos, cuadrados, c rculos, etc.). La geometr a s olida estudia las guras que pueden estar en distintos planos (pir amides, cubos, esferas, etc.). 1.1.2. L neas

Una l nea es una gura geom etrica que tiene longitud, pero no tiene anchura ni grosor.
B A

Figura 1: Una linea recta Se puede considerar que una l nea se genera con el movimiento de un punto. Con este enfoque, una l nea recta se genera con el movimiento de un punto que se mueve siempre en la misma direcci on (ver Figura 1), mientras que una l nea curva se genera con el movimiento de un punto que cambia continuamente de direcci on (ver Figura 2).

Figura 2: Una linea curva Otro punto de vista consiste en considerar una l nea como formada por innitos puntos. En particular, una l nea recta (llamada por brevedad recta) queda completamente determinada por dos cualesquiera de sus puntos y se considera ilimitada en extensi on. Notaci on: Si A y B son dos puntos que pertenecen a una recta, dicha recta se denota como AB .

1.1 Segmentos 1.1.3. Rayos

1 Geometr a

Dada una l nea recta, a una porci on de dicha recta que se origina en un punto O y que se extiende ilimitadamente por el extremo que contiene a un punto A, se le llama rayo OA (ver Figura 3) y se denota por OA.

Figura 3: rayo OA

1.1.4.

Segmento de recta

Dados dos puntos distintos A y B en una recta, se le llama segmento a la gura formada por A, B y todos los puntos que se encuentran entre ellos dos (ver Figura 4).

Figura 4: Segmento AB Se denota por AB (o simplemente AB ) y se lee segmento AB. Los puntos A y B se llaman extremos y los otros puntos forman el interior del segmento. umero positivo que La medida de un segmento AB se denota m(AB ) o simplemente AB y es un n se compara con la longitud de un segmento unitario. 1.1.5. Punto medio de un segmento

a entre A y B y se verica que Un punto C se llama punto medio de un segmento AB si C est AC = CB (ver Figura 5).
A C B

Figura 5: C punto medio del segmento AB Todo segmento posee un punto medio el cual lo biseca, es decir, lo divide en dos segmentos de igual longitud.

1.1 Segmentos

1 Geometr a

x A a C b Figura 6 1.1.6. Operaciones con segmentos colineales B

Medir es comparar una magnitud con otra de su misma clase que sirve como patr on y a la que se llama unidad de medida. Para medir los segmentos se utilizan diversos instrumentos, siendo el m as sencillo una regla graduada. En la Figura 6 se cumplen las siguientes relaciones: AB = AC + CB AC = AB CB CB = AB AC x=a+b a=xb b=xa (1)

Observaci on: La relaci on de adici on (1) se puede generalizar as : Tomemos n puntos consecutivos A1 , A2 , A3 , . . . , An en una misma recta, entonces se vericar a la siguiente relaci on: A1 An = A1 A2 + A2 A3 + A3 A4 + + An1 An Este resultado se conoce como el Teorema de Chasles. Debemos notar que los segmentos considerados son consecutivos. Ejemplo 1. Ejemplos de problemas con segmentos a) Sobre una recta est an ubicados los puntos A, B , C y D. Si AD = 24 cm, AC = 15 cm y BD = 17 cm, cu anto mide BC en cm? Soluci on Seg un los datos del problema (ver Figura 7), se tiene que: CD = AD AC = 24 15 = 9 cm y puesto que: BC = BD CD, se tiene que BC = 17 9 = 8 cm. b) Sean A, B , C y D puntos consecutivos de una recta. Si AC + BD = 16 m, y BC = 4 m, cu al es el valor de AD en m? Soluci on Construimos la gura seg un las condiciones del problema y se observa que: AD = AC + CD, pero CD = BD 4. Luego, AD = AC + BD 4 = 16 4 = 12 m 6

1.1 Segmentos

1 Geometr a

Figura 7 c) M , A, O y B son puntos consecutivos sobre una recta, siendo O el punto medio de AB . Si M A = 2 y AB = 6, calcular (M O)2 . Soluci on Ya que O es el punto medio de AB , se tiene: AO = OB = Luego, M O = M A + AO = 2 + 3 = 5 Por lo tanto, (M O)2 = 25 d) Sobre una recta se tienen los puntos consecutivos A, B y D. Entre los puntos B y D se toma un punto C tal que AC = CD/4. Determinar BC , sabiendo que BD 4AB = 20. Soluci on Sea x = BC . Del dato AC =
CD 4

AB 6 = =3 2 2

y llamando AC = a, se tiene CD = 4a.

Reemplazando los literales x y a en la igualdad BD 4AB = 20, se tiene (dibujar la gura): (x + 4a) 4(a x) = 20 x + 4a 4a + 4x = 20 5x = 20 x=4 Es decir, BC = 4. e) Al dividir un segmento en partes cuyas medidas son directamente proporcionales a 1/3, 1/4 y 1/2 se obtienen tres segmentos, el segundo de los cuales mide 12 cm. Cu al es la suma en cm de las medidas del segundo y tercer segmento? Soluci on Sean los segmentos AB , BC y AC . Seg un las condiciones del problema: AB = ya que BC = 12 entonces
k 4

k k , BC = 3 4

CD =

k 2
48 2

= 12, de donde k = 48. = 24.

Esto nos permite calcular el valor del tercer segmento CD, as CD = Luego, la suma pedida es BC + CD = 36. 7

1.1 Segmentos 1.1.7. Ejercicios

1 Geometr a

1) Sobre una recta se consideran los puntos consecutivos A, B y C . Si AB = 8 cm y BC = 12 cm, hallar AC . 2) Dados los puntos colineales y consecutivos A, B , C y D, tales que AB = 7, BC = 8 y AD = 24, calcular CD. 3) En una l nea recta se consideran los puntos consecutivos A, B y C tales que AC = 25 y BC = 15. Calcular AB . 4) A, B , C y D son puntos ubicados en una l nea recta de modo que AB = BC , CD = 20 y AB = 5. Hallar AD. 5) Dado el segmento AB y su punto medio O, si P es un punto interior al segmento OB , OP = 1 y P B = 5, calcular AB . 6) En una l nea recta se ubican los puntos consecutivos A, B , C y D de modo que AB = 2BC y CD = 3BC . Si BC = 1, calcular AD. Denici on 1.1. 1. Raz on: se llama raz on, al cociente de dos cantidades, expresadas en la a misma magnitud, por ejemplo . b a c 2. Proporci on: se llama proporci on a la igualdad de dos razones. Por ejemplo = , a los b d t erminos a y d se les llama extremos y los t erminos b y c se les llama medios, al t ermino d se le llama cuarta proporcional entre a, b y c en este orden. on r si 3. Un punto P AB divide al segmento AB en una raz es el punto medio de AB .
1

PA = r. Si r = 1 entonces P PB

Figura 8 4. Sean AB y CD y sean X AB y Y CD, decimos que X e Y dividen a AB y CD en segmentos proporcionales si XA YC = XB YD Ejemplo 2.
A

P divide AB en la raz on

PA 8 = =4 PB 2

Consideremos las divisiones siguientes:


Cuando el puntos P est a entre los puntos A y B decimos que P divide interiormente al segmento AB en la raz on r. Si P no est a entre los puntos A y B , decimos que P divide exteriormente a segmento AB .
1

1.1 Segmentos
A X B

1 Geometr a

Figura 9 x PA x 1 = = PB 2x 2 NA 3x 1 = = NB 6x 2

2x 3x

3x

Dado un segmento AB y una raz on k = 1, conseguimos encontrar dos puntos que dividen AB en a dividido por dos punto P y N , en la esta raz on: una interior y otra exterior. Cuando AB est a dividido arm onicamente. Y, los puntos P y N se misma raz on, decimos que el segmento AB est llaman conjugados arm onicos con respecto a A y B . Denici on 1.2. ( Divisi on arm onica) Decimos que los puntos P y N dividen arm onicamente al segmento AB cuando NA PA = . PB NB
N A P B

Esta denici on de divisi on arm onica es equivalente a los sigueinte: Se dice que dos puntos dividen un segmento de l nea arm onicamente si lo dividen interna y externamente en la misma raz on. 1.1.8. Ejercicios

1. Sobre una linea recta se consideran los puntos colineales y consecutivos A, B, C y D; tal que AC = 19 y BD = 23. Encuentre la longitud del segmento que une los puntos medios de AB y CD. 2. Sobre una linea recta se consideran los puntos colineales y consecutivos A, B, C y D; siendo C el puto medio de BD; displaystyle CB =2 y AD = 12. Encuentre CD. CA 3 3. Sobre una l nea recta se consideran los puntos consecutivos M, N, P, Q tal que: P Q = 3N P y 3M N + M Q = 4. Encuentre la longitud del segmento M P 4. Sobre una l nea recta se consideran los puntos consecutivos A, B y C y luego se ubican los puntos medios M y F de AB y M C respectivamente. Encuentre la longitud de AF . Si AB + F C AM = 2 5. 5. Sobre una l nea recta se consideran los puntos consecutvos A, B, C, D y luego se toman M y F puntos medios de AB y CD respectivamente. Encuentre M F , si: AC = 18 y BD = 34. 6. Dado el segmento AB y un punto M interior a el. Demuestre que si el producto AM M B , es m aximo entonces M es el punto medio de AB .

1.2 Angulos entre paralelas.

1 Geometr a

7. A, B, C y D son puntos colineales y consecutivos tal que: AB CD = AD BC ; AB .BC = x; AD CD = y . Calcule BD. 8. Sobre una nea recta se consideran los puntos consecutivos A, B, C, D y D tal que: DC = 2 AB ; AB = a y BD = b. Encuentre AC . 9. A, B, C y D son puntos colineales y consecutivos. Si AC es la media proporcional2 entre AD AD AB 1 . y BD. Calcular k , si: k = 2 AC CD

1.2.
1.2.1.

Angulos entre paralelas.


Denici on de Angulo

Denimos como a ngulo a la gura geom etrica formada por dos rayos (o semirrectas) distintas que tienen el mismo origen. Ese origen se llama v ertice del angulo. Al a ngulo de v ertice O y rayos OA y OB se le denota AOB . Dos a ngulos AOB y BOC son adyacentes si y s olo si tienen un lado com un OB y los lados no comunes OA y OC est an en semiplanos distintos, determinados por el lado com un. Bisectriz de un a ngulo es la semirrecta que lo divide en dos a ngulos adyacentes iguales. Dos angulos son: Congruentes o Iguales : si tienen igual medida. Suplementarios : si su suma es 180. Complementarios : si su suma es 90. Por otra parte, dos rectas en el plano pueden ser secantes o paralelas,3 dependiendo si se cortan o no; adem as, si las rectas son secantes, el punto de corte es u nico, y denen cuatro a ngulos, que se agrupan por parejas en angulos opuestos por el v ertice (las parejas de a ngulos tales que uno est a formado por la prolongaci on de los lados del otro). Los a ngulos opuestos por el v ertice son iguales (Justique), por lo que dos rectas secantes forman cuatro a ngulos que denen dos parejas de angulos iguales, y si tomamos un miembro de cada pareja, se tienen dos a ngulos suplementarios. En particular, si las rectas son secantes y forman cuatro a ngulos iguales, ser an llamadas rectas perpendiculares,4 y los angulos as generados son llamados angulos rectos. Y como es muy conocido, un angulo agudo es aquel cuya medida es menor a la de un angulo recto, y un angulo obtuso es aquel cuya medida es mayor que un a ngulo recto; en particular, un angulo obtuso ser a llamado angulo llano si su medida es el doble que la de un a ngulo recto.
Dados dos segmentos de longitudes a y b se llama media proporcional a un segmento de longitud x, talque a x verique = . x b 3 Si la recta AB es paralela a la recta CD, se denota AB CD. 4 Si la recta AB es perpendicular a la recta CD, se denota AB CD.
2

10

1.2 Angulos entre paralelas. 1.2.2. Angulos entre rectas paralelas

1 Geometr a

Al intersecar un par de rectas paralelas por una recta llamada transversal o secante, se forman los siguientes tipos de a ngulo: Angulos Correspondientes : Son dos a ngulos no adyacentes situados en el mismo lado de la secante, uno en el interior y otro en el exterior de las paralelas. Angulos Alternos Internos : Son dos a ngulos no adyacentes situados en el interior de las paralelas, y en distintos lado de la secante. Angulos Alternos Externos : Son dos a ngulos no adyacentes situados en el exterior de las paralelas, y en distintos lado de la secante. Angulos Conjugados : Son dos a ngulos internos o externos, no adyacentes y situados del mismo lado de la secante. Las propiedades fundamentales de los angulos entre paralelas son: 1. Los a ngulos correspondientes son iguales entre s . 2. Los a ngulos alternos internos son iguales entre s . 3. Los a ngulos alternos externos son iguales entre s . 4. Los a ngulos conjugados son suplementarios.

Figura 10: Angulos entre las rectas paralelas L1 y L2 .

1.2.3.

Problemas

1. Tres angulos adyacentes forman un semiplano y tienen sus medidas proporcionales a los n umeros 5, 7 y 8. Hallar la medida del menor a ngulo. 2. Demostrar que las bisectrices de dos angulos suplementarios son perpendiculares.

11

1.2 Angulos entre paralelas.

1 Geometr a

3. En la gura adjunta, L1 L3 L4 . Calcular x.

L2 y

12

1.2 Angulos entre paralelas. 4. Con ayuda de la gura 11, demuestre que: Si L1 L2 entonces = + .

1 Geometr a

Figura 11 5. En la gura 12, AB F G. Hallar el a ngulo x si el AM F = 90 y el M AB = 110.

Figura 12 6. Calcular el OP Q, si OP es bisectriz del a ngulo O, L1 L2 y P Q L1 . Ver gura 13.

Figura 13

13

1.2 Angulos entre paralelas. 7. En la gura 14, L1 L2 y L3 L4 , calcular .

1 Geometr a

Figura 14 8. En la gura 15, calcular x, si L1 L2 .

Figura 15

9. Calcular la medida del gr aco anexo, si las rectas L1 y L2 son paralelas.

14

ngulos. 1.3 Tria 10. En la gura 16, L1 L2 y L3 L4 . Hallar el valor del angulo .

1 Geometr a

Figura 16 11. Sea AOB = 24, en la regi on exterior a dicho angulo se traza el rayo OC. Hallar la medida del angulo formado por las bisectrices de los a ngulos AOC y BOC . 12. Del gr aco 17, calcular y , cuando x tome su m aximo valor entero.

Figura 17

1.3.
1.3.1.

Tri angulos.
Clasicaci on de Tri angulos

1. Con relaci on a sus lados: a ) Escaleno : si sus tres lados no son congruentes. b ) Is osceles : si por lo menos dos de sus lados son congruentes. c ) Equil atero : si sus tres lados son congruentes (note un tri angulo equil atero es tambi en is osceles, y que los tres angulos internos son iguales entre s e iguales a 60) 2. Con relaci on a sus angulos internos: a ) Acut angulo : si su angulo mayor es agudo (note que entonces los tres angulos son agudos)

15

ngulos. 1.3 Tria

1 Geometr a

b ) Rect angulo : si su angulo mayor es a ngulo recto (note que el a ngulo en cuesti on es u nico y que los otros dos angulos son agudos; as , en un tri angulo rect angulo, la hipotenusa es mayor a los catetos) c ) Obtus angulo, si el a ngulo mayor es a ngulo obtuso (note que el a ngulo en cuesti on es u nico y que los otros son agudos; as , en un tri angulo obtus angulo, el lado que se opone al angulo obtuso es el lado mayor) 1.3.2. Teoremas Fundamentales del Tri angulo

Diremos que tres puntos que pertenecen a una misma recta son puntos colineales ; de manera an aloga, si tres rectas pasan por un mismo punto, ser an llamadas rectas concurrentes. Si tomamos al azar tres puntos en el plano, en muy raras ocasiones estos puntos estar an alineados,5 y diremos entonces que son los v ertices de un tri angulo; an alogamente sucede con las rectas, tres rectas por lo general no concurren, y la gura geom etrica que estas denen es tambi en un tri angulo.6 Una denici on completa para nuestros intereses es la siguiente: Denici on de Tri angulo. Si A, B y C son tres puntos cualesquiera no colineales (Ver gura 18), entonces la reuni on se los segmentos AB , BC y AC se llama tri angulo ABC y se denota por ABC . Los puntos A, B y C se llaman v ertices y los segmentos AB , BC y AC se llaman lados. Simb olicamente: ABC = AB BC AC . Todo tri angulo ABC determina tres angulos internos o interiores : ABC , ACB y BAC , y se llamar a angulo externo o exterior, al a ngulo determinado por un lado y la prolongaci on del lado adyacente, en la gura 18, , y son angulos exteriores.

Figura 18: Elementos del Tri angulo Dado el ABC , se tiene que AB + BC + CA = p = 2s, donde p es llamado el per metro y s el semiper metro del tri angulo. Para abreviar, suele asociarse a cada v ertice un lado opuesto, y viceversa, por ejemplo, el lado opuesto de A es BC , y es frecuente que se denote por a; an alogamente b = CA, c = AB .
En teor a de probabilidades, la probabilidad que esto ocurra es cero! El t ermino m as riguroso para esta gura es tril atero. En este caso, habr a que hacer una consideraci on: si hay un par de rectas paralelas, el tril atero denido ya no es normal seg un nuestro sentido com un, sin embargo, sigue siendo un tril atero!
6 5

16

ngulos. 1.3 Tria

1 Geometr a

Teorema 1: En todo tri angulo, la medida de un a ngulo exterior es igual a la suma de las medidas de dos a ngulos interiores del tri angulo no adyacentes a el. La demostraci on de este teorema se basa en las relaciones de angulos entre paralelas; se deja al lector que haga la demostraci on (Sugerencia: por un v ertice, trace una recta paralela al lado opuesto) Corolario: En todo tri angulo, la suma de las medidas de sus tres a ngulos internos es igual a 180. Teorema 2: Desigualdad Triangular. En todo tri angulo, la longitud de uno de sus lados est a comprendido entre la suma y la diferencia de los otros dos. Sin ser muy rigurosos, suponga que dado el segmento AB se traza con centro en A una circunferencia de radio r1 , y con centro en B una circunferencia de radio r2 ; si AB < r1 + r2 , las circunferencias se cortar an en dos puntos, y cualquiera de ellos puede ser el v ertice C , as AB < BC + CA; en cambio, si AB = r1 + r2 o peor a un, si AB > r1 + r2 , la construcci on del ABC no es posible. La Desigualdad Triangular es un resultado fundamental, a partir de esta y de su modelo de demostraci on se generan los Criterios de Congruencia de Tri angulos ; a groso modo, si dadas ciertas condiciones, la construcci on de una gura geom etrica (un tri angulo en particular) queda determinada de manera u nica, entonces dos guras que reunen las mismas condiciones ser an llamadas guras congruentes. As , si se tienen tres segmentos (cuyas longitudes cumplen la desigualdad triangular), dejando uno jo y construyendo las circunferencias con centros en los extremos de este segmento y radios las longitudes de los otros segmentos, por construcci on, s olo ser a posible obtener dos tri angulos (uno con cada punto de intersecci on de las circunferencias), que son b asicamente el mismo pero la orientaci on de los a ngulos es contraria; as , si se sabe que dos tri angulos cumplen tener lados respectivamente iguales, por construcci on, deben de ser iguales. Este es el conocido criterio LLL de congruencia de tri angulos; m as adelante se detallar an el resto de criterios, pero a partir de este probaremos el siguiente resultado: Teorema 3: En todo tri angulo, se cumple que a lados iguales se oponen angulos iguales, y viceversa. Suponga que ABC es tal que AB = AC , entonces, por criterio LLL, ABC es congruente al ACB (en ese orden, porque AB = AC , BC = CB y CA = BA), entonces, los a ngulos que se oponen a los a ngulos iguales son iguales. Para el rec proco necesitamos otro criterio de congruencia, por lo que la demostraci on se dejar a incompleta; retome esto en la secci on de congruencia de tri angulos. Teorema 4: En todo tri angulo se cumple que a mayor lado se opone mayor a ngulo y viceversa. Este teorema se deja como ejercicio para el lector (Sugerencia: utilice el teorema anterior, tome el lado mayor y dena un punto adecuado que genere un tri angulo con dos lados iguales.)

17

ngulos. 1.3 Tria 1.3.3. Teorema de Pit agoras

1 Geometr a

Abordamos el estudio de las Relaciones M etricas, del cual solo realizaremos el an alisis del famoso Teorema de Pit agoras, cuyo enunciado es el siguiente: Teorema: Pit agoras. En un tri angulo rect angulo, el cuadrado de la hipotenusa es igual a la suma de los cuadrados de los catetos. Una demostraci on de este teorema es debida a Thabit ibn Qurra (836-901), la cual consiste en diseccionar la gura que se forma al construir dos cuadrados de lados respectivamente iguales a los catetos de un tri angulo rect angulo, como se muestra en el gr aco 19.

Figura 19 Rec proco del teorema de Pit agoras: Si en un tri angulo el cuadrado de un lado es igual a la suma de los cuadrados de los otros dos lados, el tri angulo es rect angulo.7

1.3.4.

Rectas Notables de un tri angulo

1. Altura: Se llama altura de un tri angulo al segmento que parte de uno de sus v ertices y llega en forma perpendicular al lado opuesto o a su prolongaci on. 2. Mediana: Se llama Mediana al segmento que une un v ertice con el punto medio del lado opuesto. 3. Mediatriz: Se denomina mediatriz de un lado de un tri angulo es la recta perpendicular a dicho lado en su punto medio. 4. Una Bisectriz: La bisectriz es la recta que divide en dos angulos iguales a un a ngulo dado; en particular, es bisectriz interna si es la bisectriz de un a ngulo interno de un tri angulo, y bisectriz externa si es la bisectriz de un a ngulo externo de un tri angulo. 1.3.5. Distancia de un punto a una recta

En la gura 20, sea P un punto exterior a una recta L, la longitud de la perpendicular P M a la recta L es la distancia del punto P a dicha recta. Esta perpendicular tiene la propiedad de ser u nica y su longitud es la distancia m nima del punto a la recta (Pru ebelo utilizano el hecho que la hipotenusa es mayor que los catetos). Los segmentos P A y P B no son perpendiculares a L y se llaman oblicuas.
7

Ver demostraci on en la secci on de congruencia de tri angulos.

18

ngulos. 1.3 Tria

1 Geometr a

Figura 20 1.3.6. Problemas

1. En la gura adjunta ambos tri angulos son equil ateros. Encuentre el valor de .

2. En la gura 21, calcular el x si el AOB = 100 y L1

L2 .

Figura 21 3. (*) En la gura 22, ABDE es un cuadrado y BCD es un tri angulo is osceles con BD = DC . Si ABC = 160, determinar la medida de AEC .

19

ngulos. 1.3 Tria

1 Geometr a

Figura 22

4. (*) (XV Competencia de Clubes Cabri Primera Ronda) En la gura adjunta, ABCD es un rect angulo tal que AB = 2BC . M es el punto medio de AB y los tri angulos AM E y M BF son equil ateros. Si P es la intersecci on de las rectas DE y CF , encuentre los a ngulos del CDP .

5. Probar que una bisectriz exterior de un tri angulo es paralela al lado opuesto si y s olo si el tri angulo es is osceles. 6. Si AB y F G son rectas paralelas, el ABC = CDE = , el DEF = Calcule . Figura 23
2

y el GF H = 150.

Figura 23

20

ngulos. 1.3 Tria 7. (*) Hallar la suma de los a ngulos + + + en la gura 24.

1 Geometr a

Figura 24 8. Determine el valor de la suma A + B + I + H + F + G. Figura 25.

Figura 25 9. En el ABC el BAC = 36 y AC = AB . Probar que la bisectriz interior BD (D en AC ) es congruente con el lado BC . 10. Sea ABC un tri angulo rect angulo en B con AB = BC , se construye exteriormente el tri angulo equil atero BCD. Encuentre el a ngulo DAB . 11. En el ABC , AB = AC y D un punto sobre la recta AC , tal que BC = BD = DA. Determine la medida del angulo ABD, si: a ) D est a entre A y C . b ) A est a entre D y C . 12. En un ABC , D es un punto sobre el lado AC tal que AB = AD. Si ABC ACB = 90, hallar el CBD. 13. Se tiene un tri angulo is osceles ABC , AB = BC en el cual se traza al altura AF tal que BF = 6 y F C = 2. Hallar AC .

21

ngulos. 1.3 Tria

1 Geometr a BCA?

14. En la gura 26, el ABC = ACE , DC = EC , Qu e l nea notable es AD del

Figura 26 15. Cu al es el valor de b a en la gura 27?

Figura 27 16. (*) Sea ABC un tri angulo tal que las medianas respectivas a B y C son perpendiculares. Demuestre que se cumple la relaci on. 5BC 2 = CA2 + AB 2 . 17. La hipotenusa BC de un tri angulo rect angulo ABC se divide en 4 segmentos congruentes por los puntos G, E y H . Si BC = 20, encuentra la suma de los cuadrados de las longitudes de los segmentos AG, AE y AH . Figura 28.

Figura 28

22

ngulos. 1.3 Tria

1 Geometr a

18. (*) Dado un cuadrado ABCD, se construyen los tri angulos equil ateros ABP (exteriormente) y ADQ (interiormente). Probar que C , P y Q est an alineados. 19. (*) Sea ABC un tri angulo rect angulo con CAB = 90. D es un punto sobre la prolongaci on de BC tal que BD = BA. E es un punto en el mismo semiplano que A respecto de BC , tal que CE BC y adem as CE = CA. Mostrar que A, D y E est an alineados. 20. El cuadril atero ABCD mostrado en la gura 29 cumple que AB CD y BC DA.8 Sobre las prolongaciones de AB y AD se construyen puntos E y F tales que BC = BE y DC = DF . Demuestre que C , E y F est an alinedos.

Figura 29

21. (*) En la gura adjunta, AB = BC = CD = DE = EF = F G = GA. Calcule la medida del DAE .

22. (*) (XXVIII Olimpiada Brasile na de Matem atica) En la gura 30, AB = AC , AM = AN y CAM = 30, encuentre el valor del BM N .

Figura 30 23. Los lados de un tri angulo is osceles son 12 y 5 metros, cu al es su per metro? 24. Muestre que los lados de un tri angulo cumplen que |a b| < c y que c <
8

a+b+c . 2

El cuadril atero ABCD es un paralelogramo.

23

ngulos. 1.4 Congruencia de Tria

1 Geometr a

25. Muestre que es posible construir un tri angulo con segmentos de longitudes a, b, c si y s olo existen n umeros positivos x, y , z tales que: a = x + y , b = y + z , c = z + x. 26. (*) (Etapa seminal Estatal de XXII Olimpiada Mexicana de Matem aticas) En la gura 31 se muestra un hex agono regular ABCDEF de lado 1. Los arcos del c rculo que est an dibujados tienen centro en cada v ertice del hex agono y radio igual a la distancia al v ertice opuesto. P , Q, R, S , T y U son los puntos de corte de estos arcos. Cu anto mide cada lado del hex agono P QRST U ?

Figura 31

1.4.
1.4.1.

Congruencia de Tri angulos.


Criterios de Congruencia de tri angulos

Denici on de Congruencia de tri angulos. El ABC es congruente al A B C si: AB = A B , AC = A C , BC = B C , ABC = A B C , ACB = A C B y BAC = B A C . Simb olicamente: ABC = A B C . V ease gura 32.

Figura 32: Denici on de Igualdad de Tri angulos. La denici on anterior establece que dos tri angulos son congruentes si tanto los lados como los a ngulos se presentan en pares respectivos congruentes. Esto, seg un la visi on de Euclides, signica que un tri angulo es posible superponerlo sobre el otro (se puede desplazar, girar o reejar) y coincidir a de manera perfecta. Sin embargo, es importante mencionar que en muy raras ocasiones se tendr a a disposici on tanta informaci on, de all la importancia de los criterios de congruencia, que 24

ngulos. 1.4 Congruencia de Tria

1 Geometr a

establecen los requisitos m nimos para garantizar que dos tri angulos son congruentes. El siguiente es el primero de los tres criterios de congruencia de tri angulos, y se denomina criterio de LADO-ANGULO-LADO, en s mbolos: L-A-L. Criterio L-A-L. Si los tri angulos ABC y A B C presentan las congruencias: AB = A B , AC = A C y BAC = B A C , entonces ABC = A B C .

Figura 33: Criterio LAL Seg un el criterio L-A-L, dos tri angulos son congruentes si en uno de ellos existen dos lados y el a ngulo (comprendido entre dichos lados), respectivamente congruentes a dos lados y el a ngulo (comprendido entre dichos lados), en el otro tri angulo. Criterio A-L-A. Sean ABC y A B C dos tri angulos tales que: AC = A C , BCA = B C A y BAC = B A C , entonces ABC = A B C .

Figura 34: Criterio ALA. Criterio L-L-L. Si un tri angulo tiene sus tres lados respectivamente congruentes a los tres lados de otro tri angulo, entonces estos dos tri angulos son congruentes.

Figura 35: Criterio LLL.

25

ngulos. 1.4 Congruencia de Tria Ahora demostraremos el Rec proco del Teorema de Pit agoras.

1 Geometr a

Demostraci on: Sea ABC un tri angulo talque BC 2 = AB 2 + AC 2 , por construcci on sea el A B C rect angulo en A tal que A B = AB y A C = AC , entonces por el teorema de Pit agoras 2 2 2 2 2 B C = A B + A C , as que B C = BC , de donde B C = BC y por el criterio LLL, se deduce que el A B C = ABC , por lo tanto el BAC = B A C = 90.

1.4.2.

Teorema de la Base Media

En todo tri angulo, el segmento que une los puntos medios de dos lados es paralelo al tercer lado e igual a su mitad.

Figura 36: Teorema de La Base Media. En la gura 36, M N es el segmento que une los puntos medios de los lados AB y BC del ABC , AC a este segmento se le llama BASE MEDIA DEL TRIANGULO . Se verica que M N = y que 2 M N AC . Demostraci on: 1. Prolongar el segmento M N hasta el punto P tal que M N = N P . 2. Los tri angulos M N B y P N C son congruentes, ya que BN = N C , M N = N P y el CN P = M N B , por consiguiente, el N CP = M BN , por lo tanto, CP M B (Por angulos alternos internos iguales). Adem as, P C = M B = M A; con lo cual se tiene que: M A = P C . 3. Uniendo el punto A con el punto P se forman los tri angulos congruentes AM P y ACP (por L A L) ya que M A = P C , AP = AP , M AP = AP C (por a ngulos alternos internos MP = 1 AC . Adem as, entre las paralelas M A y P C ). Luego, M P = AC , entonces N P = 1 2 2 P AC = M P A, de donde M P AC o que M N AC . Corolario: Menor mediana de un tri angulo rect angulo. En todo tri angulo rect angulo, la mediana relativa a la hipotenusa es la mitad de la longitud de la hipotenusa y es la menor de las tres medianas del tri angulo. Demostraci on: En la gura 37, BM es la mediana relativa a la hipotenusa AC del ABC , ; (con lo cual se tendr a que BM = AM = M C ). Si por M se traza probaremos que BM = AC 2 una paralela al lado AB , que corte al lado BC en N , entonces N es el punto medio de BC y el M N C = 90, los tri angulos BN M y CN M son congruentes por el criterio L-A-L, luego 26

ngulos. 1.4 Congruencia de Tria M B = M C = AM . Probar que BM es la menor mediana (Ejercicio).

1 Geometr a

Figura 37: Menor Media en un Tri angulo Rect angulo.

1.4.3.

Problemas

1. (*) En la gura adjunta, ABC es un tri angulo equil atero y CDEF es un cuadrado. Se construye un punto G tal que CF = CG y adem as CF G = 15. Probar que AGC = BDC .

2. Dado un tri angulo equil atero ABC , se construye un tri angulo equil atero DEF cuyos v ertices est an sobre los lados del ABC , tal como muestra la gura 38. Demuestre que los tri angulos ADF , BED, CF E son todos congruentes entre si.

Figura 38 3. ABCD es un cuadrado, E , F , G y H son puntos sobre los lados AB , BC , CD, DA, respectivamente, tal que EF GH tambi en es cuadrado. Demuestre que los tri angulos AEH , BF E , CGF , DHG son todos congruentes entre si. Figura 39. 4. ABCDE y F GHIJ son pent agonos regulares (Vease gura 40). Demuestre que los tri angulos AF J , BGF , CHG, DIH , EJI son todos congruentes entre si.

27

ngulos. 1.4 Congruencia de Tria

1 Geometr a

Figura 39

Figura 40 5. Si AB CD y AB = CD entonces, AD = BC y AD BC 9 .

6. Demuestre que dos tri angulos desplazados son congruentes. Sugencia: Utilice el problema anterior.

El cuadril atero ABCD se denomina paralelogramo.

28

ngulos. 1.4 Congruencia de Tria

1 Geometr a

7. Demuestre que dos tri angulos rotados son congruentes.

8. Demuestre que dos tri angulos reejados con respecto a un punto

10

son congruentes.

9. Demuestre que dos tri angulos reejados con respecto a una recta son congruentes.

Importante: Las traslaciones, rotaciones y reexiones no cambian el tama no ni la forma de un tri angulo.

10. (*) En la gura adjunta, ABCD un cuadrado y EF GH . Demuestre que que EF = GH .

10

La reexi on con respecto a un punto es equivalente a una rotaci on de 180

29

ngulos. 1.4 Congruencia de Tria

1 Geometr a

11. Dos cuadrados ABCD y EHGF , ambos de lado l, est an colocados en manera tal que un v ertice de uno est a en el centro del otro (como en la gura anexa). l2 Demuestre que el area del cuadril atero EJBK es 4 y por ende no depende de la posici on de J (o K ).

12. En un ABC el B = 2C , la mediatriz del lado AC corta en F al lado BC . Hallar AB , si F C = 9. 13. En la gura 41, AC = 12 AF = 4 y BAF = 30. Hallar BF si AG = GC .

Figura 41 14. En la gura 42, AG = GC , el AF G = 20. Hallar el F AC , si AC = 2BF .

Figura 42

30

ngulos. 1.4 Congruencia de Tria

1 Geometr a

15. (*) (Examen nal de XVI Olimpiada Mexicana de Matem atica) Los a ngulos de un tri angulo ABC est an en progresi on aritm etica (B A = C B = ), D, E , y F son los puntos medios de los lados BC , CA y AB , respectivamente. Llamamos H al pie de la altura trazada desde C (que cae entre B y F ) y G a la intersecci on entre DH y EF . Hallar los a ngulos del F GH . 16. (*) Sea ABCD un cuadrado. Se construyen tri angulos equil ateros ADP y ABQ como se muestra en la gura 43. Sea M la intersecci on de CQ con AD y N la intersecci on de CP con AB . Demuestre que CM N es un tri angulo equil atero.

Figura 43 17. En la gura 44, ABC , CDE y EF A son tri angulos is osceles, con el ABC = CDE = EF A = 120. Probar que el BDF es equil atero.

Figura 44 18. (*) ABC es un tri angulo is osceles con ABC = ACB = 80. D es un punto en AC tal que ABD = 10. Demuestre que AD = BC .

31

teros 1.5 Cuadrila

1 Geometr a

1.5.
1.5.1.

Cuadril ateros
Clasicaci on de Cuadril ateros

Los cuadril ateros pueden clasicarse de acuerdo a sus diagonales de la siguiente forma: Cuadril atero Convexo: Es un cuadril atero con las dos diagonales en su interior.
A D

Cuadril atero Entrante: Es un cuadril atero con una diagonal en el interior y otra en el exterior.
A

Cuadril atero Cruzado Es un cuadril atero con las diagonales en su exterior.


B A

C D
11

Es muy frecuente que se considere que un cuadril atero es convexo, a menos que se especique lo contrario. Esto es as porque muchos resultados son m as claros en un cuadril atero convexo, sin embargo, es importante darse cuenta que existen teoremas que no se cumplen para cualquier tipo de cuadril ateros, por ejemplo: Teorema: La suma de los angulos internos de un cuadril atero no cruzado es 360 . La demostraci on de este resultado se basa en la disecci on del cuadril atero en dos tri angulos cuyos a ngulos internos conforman los a ngulos internos del cuadril atero, sin embargo, estas condiciones no pueden lograrse en un cuadril atero cruzado; de hecho, la suma de los a ngulos internos puede hacerse arbitrariamente peque na cuando el cuadril atero es cruzado.
Tanto los cuadril ateros convexos como los entrantes son cuadril ateros simples, que son los cuadril ateros cuyos lados no se cortan salvo en los extrenos; en contraposici on, los cuadril ateros cruzados no son simples.
11

32

teros 1.5 Cuadrila

1 Geometr a

Tambi en hay otras clasicaciones de cuadril ateros de acuerdo a sus lados y angulos. Cuadril atero Equi angulo: un cuadril atero (convexo) es equi angulo si todos sus angulos internos son iguales; dado el teorema anterior, los angulos son iguales a 90 , por ello este cuadril atero es llamado rect angulo. Cuadril atero Equil atero: un cuadril atero (convexo) es equil atero si todos sus lados son iguales. A este cuadir atero tambi en se le conoce como rombo. Cuadrado: es un cuadril atero que es equi angulo y equil atero.
C B

Paralelogramo: es un cuadril atero con los lados opuestos paralelos. Trapecio: es un cuadril atero con un par de lados opuestos paralelos.12

1.5.2.

Paralelogramos

Es el cuadril atero que tiene sus lados opuestos paralelos y congruentes. En todo paralelogramo se cumple que sus a ngulos opuestos son congruentes y sus diagonales se bisecan. El paralelogramo tambi en se conoce como romboide.
D E C

Dado el paralogramo ABCD, por propiedades de a ngulos entre paralelas es posible probar el siguiente resultado: Teorema: Los a ngulos opuestos son iguales y los angulos consecutivos son suplementarios: ABC = CDA = y BCD = DAB = 180 . Por otra parte, por criterio ALA,
12

ABC

CDA; esto implica que AB = CD y BC = DA, i.e.

Note que un paralelogramo es tambi en un trapecio.

33

teros 1.5 Cuadrila Teorema: Los lados opuestos de un paralogramos son iguales. A partir de esto, si M es la intersecci on de AC con BD, por criterio ALA, por lo que AM = CM y BM = DM , i.e. Teorema: Las diagonales de un paralelogramo se bisecan. Adem as, se cumple un resultado sosticado y muy importante:

1 Geometr a

ABM

CDM ,

Teorema: Ley del Paralelogramo. Si ABCD es un paralelogramo entonces el doble de la suma de los cuadrados de los lados es igual a la suma de los cuadrados de las diagonales, es decir 2 AB 2 + BC 2 = AC 2 + BD2 . Demostraci on: Aplicando la Ley del Coseno a

ABC y

ABD se tiene

AC 2 = AB 2 + BC 2 AB BC cos DB 2 = AB 2 + AD2 AB AD cos(180 ) AC 2 + DB 2 = 2 AB 2 + BC 2 AB BC (cos + cos(180 )) y dado que cos = cos(180 ) el resultado se sigue inmediatamente.

1.5.3.

Rect angulos

Rect angulo: Sus cuatro a ngulos son igual a 90 , sus lados opuestos son iguales y paralelos.
D C

En primer lugar, es importante notar que todo rect angulo es paralelogramo (por a ngulos entre paralelas), por lo que todos los resultados probados anteriormente son heredados a todo rect angulo; pero los rect angulos tienen propiedades adicionales: Observe que por criterio LAL, ABC ABD, por lo que AC = BD y entonces Teorema: Las diagonales de un cuadrado son iguales; adem as, el punto de intersecci on de estas equidista de los cuatro v ertices y por tanto es el centro de una circunferencia que pasa por todos los v ertices. Por otra parte, observe que si se aplica la ley del paralelogramo a un rect angulo se obtiene el Teorema de Pit agoras. 34

teros 1.5 Cuadrila 1.5.4. Rombos

1 Geometr a

Rombo: Sus cuatro lados son iguales.


B

Dado un rombo ABCD, por criterio LLL, ABC CDA, y por lo tanto BAC = DAC y BCA = DAC , lo cual implica BC AD y AB CD, i.e., todo rombo ABCD es un paralelogramo. Adem as, por las mismas congruencias se tiene Teorema: Las diagonales de un rombo cumplen ser una mediatriz de la otra. Teorema: Las diagonales de un rombo bisecan a los a ngulos interiores del rombo; esto implica que el punto de corte de las diagonales equidista de los cuatro lados del rombo y es el centro de una circunferencia tangente a estos. 1.5.5. Trapezoides

Es un cuadril atero que no tiene pares de lados paralelos. Los trapezoides se clasican en: Trapezoide asim etrico: No tiene ning un par de lados paralelos o congruentes.
A

Trapezoide sim etrico: Dos pares de lados consecutivos son congruentes; adem as una de las diagonales es mediatriz de la otra.

35

teros 1.5 Cuadrila

1 Geometr a

1.5.6.

Trapecios

Es el cuadril atero que tiene dos lados paralelos denominados base y los otros dos no son paralelos. La distancia entres sus bases se llama altura, y el segmento que une los puntos medios de los lados no paralelos se denomina mediana. Sea M y N los puntos medios de AD y BC . AB DC .
D

Base MenorC
F

Base Mayor
G

Los trapecios se clasican en: Trapecio escaleno: Es aquel en que sus lados no paralelos son diferentes.
D C

B A

Trapecio is osceles:Es aquel en sus lados no paralelos son congruentes. AB = CD


D C

Trapecio rect angulo: Cuando uno de sus lados no paralelos es perpendicular a las bases. 36

teros 1.5 Cuadrila

1 Geometr a

Dado el trapecio ABCD (con AB CD), se construyen los puntos medios de BC y DA, M y N , respectivamente. Si el cuadril atero M N AB se rota con centro en M y angulo 180 se genera un cuadril atero M N A C ; observe que N D = N A y N D N A , por lo que DN N A es un paralelogramo y N N = DA 2M N = DC + CA 2M N = DC + AB AB + CD MN = 2 El segmento M N es llamado base media del trapecio, y por lo reci en demostrado se tiene Teorema: La base media de un trapecio es igual a la semisuma de las bases. Por otra parte, hay ciertos trapecios que reciben nombres particulares; el trapecio rect angulo es aquel que las bases son perpendiculares a alguno de los otros lados; y por otra parte, el trapecio is osceles es aquel que los lados (distintos de las bases) tienen igual longitud. 13 1.5.7. Problemas CD, demuestre que la bisectriz interior del A es paralela

1. Dado el trapecio ABCD con AB a la bisectriz exterior del D.

2. A un rombo ABCD se le construyen exteriormente los cuadrados ABEF y BCGH . Demuestre que ABD = EBH . 3. (*) Sea ABCD un paralelogramo. Se construyen tri angulos equil ateros exteriores ADQ. Demuestre que el BP Q es equil atero. CDP y

4. Demuestre que las bisectrices interiores de un paralelogramo forman un rect angulo (qu e sucede si el paralelogramo es adem as rombo?). 5. Demuestre que las bisectrices exteriores de un paralelogramo forman un rect angulo. 6. Sea ABCD un paralelogramo. La bisectriz interna del CDA corta a BA en M , y la bisectriz interna del BAD corta a CD en N . Demuestre que ADN M es un rombo.
Los trapecios is osceles son muy importantes cuando se estudian los angulos en la circunferencia; resulta que un trapecio es is osceles si y s olo si los cuatro v ertices se ubican sobre una misma circunferencia.
13

37

teros 1.5 Cuadrila

1 Geometr a

7. Demuestre que si por el punto de intersecci on de las diagonales de un rombo se trazan perpendiculares a los lados del rombo, entonces los puntos de intersecci on de dichas perpendiculares con los lados del rombo forman un rect angulo. 8. Demuestre que las bisectrices de los a ngulos denidos por las diagonales de un rombo, cortan a los lados del rombo en cuatro puntos que forman un cuadrado. 9. En un ABC sea G la intersecci on de las medianas BB y CC . Sean B , C las reexiones de G respectivas a los puntos B y C . a) Demuestre que AGCB y AGBC son paralelogramos. b) A partir de lo anterior, demuestre que BCB C tambi en es paralelogramo. c) Demuestre que A pertenece a la recta AG, y concluya que las tres medianas de un tri angulo concurren en el punto G, llamado el centroide del ABC . d) Demuestre que CG = 2GC ; relaciones similares se cumplen para las otras dos medianas. 10. Teorema de Varignon: Dado un cuadril atero ABCD (no necesariamente convexo), se construyen los puntos medios L, M , N , O, P , Q, de los segmentos de recta AB , BC , CD, DA, BD, AC , respectivamente. Figura 45. a) Demuestre que LM N O, LP N Q, OP M Q, son paralelogramos. b) Demuestre que LN , OM , P Q concurren en un punto, llamado el centroide del cuadril atero ABCD. c) Demuestre que el per metro de LM N O es igual a AC + BD; resultados similares se cumplen para los otros paralelogramos.

Figura 45: Teorema de Varignon 11. Sea ABCD un paralelogramo tal que existe un punto E sobre el lado AB que cumple CED = 90. Sean M y N los pies de las perpendiculares trazadas desde A y B hacia DE y CE , respectivamente. Demuestre que AC , BD y M N concurren. 12. (*) (H ector Alberti) Sea ABCD un cuadrado. Se construyen los tri angulos equil ateros BDA , ACB , BDC y ACD . Demuestre que el A B C D es tambi en un cuadrado. 38

teros 1.5 Cuadrila

1 Geometr a

Figura 46 13. (*) (II Olimpiada Matem atica del Cono Sur) En la gura 46 ABCD y AECF son paralelogramos. Demuestre que BEDF es paralelogramo. 14. (*) ABCD es un cuadril atero convexo y O es un punto en su interior. Sean P , Q, R, S , los puntos medios de los lados AB , BC , CD, DA, respectivamente. Por P se traza una paralela a OR, por Q se traza una paralela a OS , por R se traza una paralela a OP , y por S se traza una paralela a OQ. Demuestre que estas cuatro rectas concurren. 15. (*) Un trapecio is osceles tiene diagonales perpendiculares y su area es 2010, determine su altura. 16. (*) (IX Competencia de Clubes Cabri, Segunda Ronda) Sea ABCDEF un hex agono regular cuyo centro es O. Se construyen los cuadrados F SOP y ORCQ. Demuestre que AP QB y SEDR son rect angulos. Figura 47.

Figura 47

39

1.6 La Circunferencia

1 Geometr a

17. (*) Sobre los lados del ABC se trazan exteriormente los cuadrados ABP Q, CARS y BCT U . Luego se trazan los paralelogramos AQA R, CSC T y BU B P . a ) Sean A , B , C los centros de los cuadrados BCT U , CARS , ABP Q, respectivamente. Demuestre que estos centros est an sobre los lados del A B C . b ) Demuestre que AA , BB , CC concurren. 18. (*) Se dibujan cuadrados exteriores a los lados de un paralelogramo, demuestre que: a ) El cuadril atero determinado por los centros de esos cuadrados es un cuadrado. b ) Las diagonales de ese cuadrado son concurrentes con las del paralelogramo. 19. (*) Dado un ABC , se construyen exteriormente los tri angulos rect angulo is osceles ACP y BCQ, con AC y BC como hipotenusas. Si M es el punto medio de AB , demuestre que el M P Q tambi en es un tri angulo rect angulo is osceles.

1.6.
1.6.1.

La Circunferencia
Elementos de la circunferencia

Una circunferencia es el lugar geom etrico de puntos que equidistan de un punto dado, llamado el centro de la circunferencia; la distancia de cada punto de la circunferencia al centro es el radio. Por otra parte, todos los puntos que est an a una distancia del centro menor o igual al radio forman el c rculo ; estos puntos quedan al interior o sobre la circunferencia. Si A y B son dos puntos de una circunferencia, el segmento de recta AB dene una cuerda ; en particular, si el centro de la circunferencia pertenece a la cuerda, esta es llamada di ametro. Es importante mencionar que para cada punto de la circunferencia existe exactamente un punto diametralmente opuesto. En la gura 48, se tiene una circunferencia de centro O y radio r = OA = OB = OA ; AB y AA son cuerdas, pero AA es tambi en di ametro, i.e, A es diametralmente opuesto a A y viceversa. Observe que por la desigualdad triangular aplicada al tri angulo is osceles AOB

Figura 48

40

1.6 La Circunferencia

1 Geometr a

AB < AO + BO = r+r = AA Si A es un punto jo, esta desigualdad es v alida para cualquier punto B sobre la circunferencia (excepto cuando B = A lo cual implica AB = AA ). Esto quiere decir que el di ametro es la mayor de todas las cuerdas. 1.6.2. Angulos en la circunferencia

A las porciones de circunferencia que quedan entre dos puntos ubicados en la circunferencia, se les llama arcos de circunferencia ; note que dos puntos sobre una circunferencia denen dos arcos de circunferencia. Tambi en, si un a ngulo tiene v ertice sobre el centro de la circunferencia y est a formado por dos radios, ser a llamado angulo central ; de nuevo, AOB hace referencia a dos a ngulos, cuya suma es 360, y subtienden respectivamente a uno de los arcos AB . Finalmente, si un a ngulo tiene el v ertice sobre la circunferencia y est a formado por dos cuerdas, ser a llamado angulo inscrito ; en la gura anterior, AA B es un a ngulo inscrito que subtiende al arco AB . Teorema: El angulo central es el doble del angulo inscrito que subtiende el mismo arco. Demostraci on: Considere la gura 49, se demostrar a que AOB = 2AP B en los tres casos mostrados. En la circunferencia de la izquierda, sea P el punto diametralmente opuesto a P ; observe que AP O y BP O son tri angulos is osceles, y por el teorema del a ngulo externo se tiene AOB = = = = = AOP + BOP (AP O + OAP ) + (BP O + OBP ) 2AP O + 2BP O 2 (AP O + BP O) 2AP B

Figura 49 El caso de la circunferencia del medio es m as sencillo y se deja como ejercicio para el lector. Para la

41

1.6 La Circunferencia

1 Geometr a

circunferencia de la derecha, el trabajo es an alogo y s olo cambia en un peque no arreglo algebraico AOB = = = = = BOP AOP (BP O + OBP ) (AP O + OAP ) 2BP O 2AP O 2 (BP O AP O) 2AP B

Corolario: Todos los angulos inscritos que subtienden el mismo arco son iguales (Ver gura 50). En particular, los a ngulos internos son iguales a 90 si subtienden a una semicircunferencia.

Figura 50 Demostraci on: Todos los a ngulos mostrados en la gura 50 son iguales a la mitad del AOB , y por tanto, son iguales entre s . En particular, si AB fuera un di ametro, AOB = 180 y por tanto 14 AP B = 90. Hay un par de a ngulos m as que son importantes: Si un punto P es interno a la circunferencia, el a ngulo de v ertice P formado por dos cuerdas que pasan por P se llama a ngulo interior. De forma similar, si P es exterior y dos cuerdas de la circunferencia (al prolongarse) pasan por P , el a ngulo con v ertice P es llamado a ngulo exterior. Dejamos como ejercicio demostrar el siguiente teorema: Teorema: Los angulos interior y exterior mostrados en la gura 51 cumplen las f ormulas siguientes: AQC = BOD + AOC 2 BOD AOC AP C = 2

14

Observe que en cualquier tri angulo rect angulo, el punto medio de la hipotenusa equidista de los tres v ertices.

42

1.6 La Circunferencia

1 Geometr a

Figura 51 1.6.3. Cuadril ateros Conc clicos

Ahora suponga que sobre una circunferencia se ubican cuatro puntos A, B , C , D, como se muestra en la gura 52. Al cuadril atero ABCD se le llama cuadril atero c clico o conc clico. Observe que ABC + CDA = + = 180 . 2 2

Figura 52 Y an alogamente DAB + BCD = 180. Esto signica que si ABCD es un cuadril atero c clico y convexo, entonces los a ngulos opuestos son suplementarios. Tambi en, es posible demostrar por contradicci on el rec proco de este resultado: si suponemos que ABCD es tal que B + D = 180 pero no es c clico, se dene el punto D como la otra intersecci on de AD con el circunc rculo del ABC , y como ABCD es c clico (por construcci on) entonces B + D = 180, luego, D = D , lo cual implica la contradicci on CD CD (rectas paralelas que se cortan en C ). As , se ha demostrado el siguiente teorema: Teorema: El cuadril atero convexo ABCD es un cuadril atero c clico si y s olo si A + C = 180 = B + D Tambi en, otro criterio muy u til y cuya demostraci on tambi en se basa en el corolario anterior es

43

1.6 La Circunferencia

1 Geometr a

Teorema: El cuadril atero convexo ABCD es un cuadril atero c clico si y s olo si se cumple alguna de las siguientes igualdades ABD BCA BAC CAD = = = = ACD BDA BDC CBD

Es importante recalcar que NO todo cuadril atero puede ser inscrito en una circunferencia; por ejemplo, un paralelogramo no ser a c clico a menos que sea rect angulo.

1.6.4.

Rectas y Circunferencias tangentes a una circunferencia

Dada una circunferencia, una recta puede ser tangente o secante a la circunferencia, dependiendo si la corta en uno o dos puntos, respectivamente; en cualquier otro caso, se dice que la recta no corta a la circunferencia.15 Sea l una recta secante a la circunferencia que corta a la circunferencia en A y B (A = B ); como el AOB es is osceles, OAB < 90. Rec procamente, si por A se traza una recta l tal que uno de los a ngulos que forma con OA es menor que 90, se puede construir un punto B sobre l tal que OAB = ABO < 90 y A = B (basta proyectar O sobre l y luego reejar A con respecto a este punto, el resultante es el punto B ); entonces el AOB es is osceles, por lo que OA = r = OB , i.e. B pertenece a la circunferencia y por tanto l corta a la circunferencia en dos puntos distintos. As Teorema: Una recta l corta a una circunferencia de centro O en dos puntos distintos A y B si y s olo si un angulo entre l y OA es agudo. Corolario: Si l es una recta tangente en A a una circunferencia de centro O, ninguno de los a ngulos entre l y OA puede ser agudo, y por tanto l OA. A partir de este resultado se prueban otros resultados muy conocidos y u tiles, que dejamos de ejercicios para el lector. Teorema: Dado un punto P externo a una circunferencia de centro O, si P A y P B son segmentos tangentes a la circunferencia en A y B , respectivamente, entonces el cuadril atero P AOB es c clico y bis osceles. Corolario: Dado un punto P externo a una circunferencia de centro O, la circunferencia de di ametro P O corta a la circunferencia dada en dos puntos A y B tales que P A y P B son rectas tangentes.
Cuando la recta es tangente a la circunferencia puede considerarse como un caso muy peculiar en el cual los dos puntos de corte coinciden.
15

44

1.6 La Circunferencia

1 Geometr a

Denici on: El a ngulo semi-inscrito en una circunferencia es aquel que se forma con una cuerda y la recta tangente en alguno de los extremos de la cuerda. Teorema: La media del angulo semi-inscrito denido por la cuerda AB es igual a la medida de un angulo inscrito que subtiende al arco AB . Demostraci on: Considere la gura 53. Como AP BO es c clico, entonces P AB = P OB ; adem as, como P O es la mediatiz de AB , P OB = P OA, por lo que P AB = AOB = AQB 2

Figura 53 Por otra parte, dada una circunferencia, otra circunferencia puede ser secante o tangente a la primera, dependiendo si la corta en uno o dos puntos, respectivamente; en cualquier otro caso se dice que las circunferencias no se cortan.16 Adem as, dos circunferencias pueden posicionarse una dentro de la otra, y claramente, la circunferencia de radio mayor es la externa mientras que otra es la interna ; particularmente, si las circunferencias tienen el mismo centro se llaman conc entricas. Finalmente, combinando estas denciones se tienen las circunferencias tangentes exteriormente y las tangentes interiormente. Teorema: Dadas dos circunferencias de centros O1 y O2 que se cortan en dos puntos distintos A y B , se cumple que O1 O2 AB . Teorema: Si dos circunferencias de centros O1 y O2 son tangentes en A, se cumple que O1 , A y O2 est an alineados. Teorema: a) Dos circunferencias, una dentro de la otra, no tienen rectas tangentes en com un. b) Dos circunferencias tangentes interiormente tienen una recta tangente com un. c) Dos circunferencias secantes (en dos puntos distintos) tienen dos rectas tangentes en com un.
Tambi en ac a puede considerarse a las circunferencias tangentes como un caso especial de circunferencias secantes en el cual los puntos de corte coinciden.
16

45

1.6 La Circunferencia

1 Geometr a

d) Dos circunferencias tangentes exteriormente tienen tres rectas tangentes en com un. e) Dos circunferencias no secantes y tal que ninguna contiene a la otra, tienen cuatro rectas tangentes en com un. 1.6.5. Problemas

1. Si el M P Q = 20, determine el valor del QON en la gura adjunta.

2. Dado un a ngulo inscrito BAC , y su angulo central BOC , se sabe que BAC + BOC = 180. Calcular el OBC . 3. En la gura 54, BCDO es un rombo. Determine el valor del angulo y la medida de las diagonales de BCDO si el radio de la circunferencia mide 6.

Figura 54 4. Un cuadril atero c clico ABCD satisface ABC = 2CDA = . Calcule .

46

1.6 La Circunferencia 5. En la gura 55. Calcule el valor del P QR.

1 Geometr a

Figura 55

6. En la gura adjunta, el AF E = 100 y el BCD = 150. Calcule el AGB .

7. Dado un angulo AOB , se trazan dos rectas l y m perpendiculares a los lados del angulo en A y B respectivamente. Si P es el punto de corte de l y m, demuestre que A, B , O, P se ubican sobre una misma circunferencia. 8. En la gura 56 se ha tomado un punto C sobre la circunferencia de centro O; AC y BC cortan a la segunda circunferencia en D y E respectivamente. Probar que OC DE .

Figura 56

47

1.6 La Circunferencia 9. (*) Dada la gura 57, demuestre que AB AB.

1 Geometr a

Figura 57 10. En la gura 58 CR es una recta tangente en C , demuestre que AB CR.

Figura 58 11. Dos circunferencias 1 y 2 son tangentes (interior o exteriormente) en P (Ver gura 59). Dos rectas que pasan por P cortan a 1 y 2 en A y C , y en B y D, respectivamente. Demuestre que AB CD.

Figura 59

48

1.6 La Circunferencia

1 Geometr a

12. (*) Dos circunferencias de centros O1 y O2 son tangentes (interna o externamente) en un punto P ; por este punto se traza una recta que corta nuevamente a la circunferencias en A y B , respectivamente. Demuestre que AO1 BO2 . 13. Dos circunferencias son tangentes externamente en el punto A. Una tangente exterior com un toca a una circunferencia en B y a la otra en C . Demostrar que BAC = 90. 14. En la gura 60, DE es tangente en D, y C es el punto medio del arco AD. Encuentre el valor del angulo seminscrito ADE .

Figura 60 15. Determine el valor del DCF , sabiendo BE es tangente en el punto D a la circunferencia de centro O. Ver Figura 61.

Figura 61

49

1.6 La Circunferencia

1 Geometr a

Figura 62 16. Si el AEB = 30, ADE = 20 y ACE = 35, calcule el AF B . V ease gura 62. 17. Dada una circunferencia de di ametro BC , se toma un punto P en la prolongaci on de BC , y se traza la tangente AP . Si AP = AB y O es el centro de la circunferencia, demuestre que el AOC es equil atero. 18. (*) Dadas dos circunferencias una fuera de la otra, demuestre que las tangentes comunes externas forman segmentos iguales; an alogamente, las tangentes comunes internas forman segmentos iguales. 19. (*) Teorema de Pithot. Demuestre que en todo cuadril atero inscribible, la suma de lados opuestos es igual. 20. (*) Teorema de Steiner. En todo cuadril atero exinscrito a una circunferencia, la diferencia de las longitudes de lados opuestos es igual. 21. En la gura 63, AB es una cuerda y por D se traza una recta tangente a la circunferencia paralela a AB . Demuestre que CD es bisectriz del ACB .

Figura 63 22. (X OMCC - P2, Aar on) Sea ABCD un cuadril atero conc clico con di ametro AC , y sea O el centro de su circunferencia. Se construyen los paralelogramos DAOE y BCOF . Demuestre que si E y F est an sobre la circunferencia entonces ABCD es rect angulo.

50

1.6 La Circunferencia

1 Geometr a

23. Cuatro cilindros de di ametro 1 est an pegados apretadamente por una cuerda muy na, como en la gura adjunta. Demostrar que la cuerda tine longitud 4 + . Demostrar tambi en que el area som breada entre los cilindros es 1 4 .

24. En la gura 64, ABCD es un trapecio is osceles con AB CD y DA = BC = 2; tomando DA y BC como di ametros, se construyen dos circunferencias tangentes. Si DC = 3AB , calcule el area del trapecio.

Figura 64 25. La gura 65 est a formada por un paralelogramo y dos circunferencia tangentes entre s y tangentes a tres lados del paralelogramo. Sabiendo que el radio de las mismas mide la cuarta parte del lado menor del paralelogramo, calcule la raz on entre el lado mayor del paralelogramo y el radio de las circunferencias.

Figura 65 26. (*) Teorema de Miquel: Dado un ABC , sean X , Y , Z puntos sobre AB , BC , CA, respectivamente. Demuestre que los circunc rculos de AXZ , BY X , CZY tienen un punto en com un M . 27. (*) Sea ABC un tri angulo, y sean L y N las intersecciones de la bisectriz del angulo A con el lado BC y el circunc rculo de ABC respectivamente. Construimos la intersecci on M del circunc rculo de ABL con el segmento AC . Prueba que los tri angulos BM N y BM C tienen la misma a rea. 28. (*) Sea AB el di ametro de una semicircunferencia. Se colocan los puntos M y K sobre la semicircunferencia y sobre AB , respectivamente.17 Sea P el centro de la circunferencia que
17

M y K son distintos de A y B .

51

1.6 La Circunferencia

1 Geometr a

pasa por A, K y M ; sea Q el centro de la circunferencia que pasa por B , K y M . Demuestre que M P KQ es conc clico. 29. En la gura 66, ABCDEF es un hex agono regular y las circunferencias de centro en los v ertices son tangentes dos a dos. Si las circunferencias sobre los v ertices B , D, F son iguales, demuestre que las circunferencias restantes son iguales.

Figura 66 30. (*) Las circunferencias 1 y 2 se cortan en los puntos A y B . Por el punto A se traza una recta que corta nuevamente a las circunferencias 1 y 2 en los puntos C y D, respectivamente. Por los puntos C y D se trazan tangentes a las circunferencias, las cuales se cortan en el punto M . Demuestra que M CBD es c clico. 31. (*) El ABC cumple que A = 90 y AB = AC . Se toma un punto E del segmento AB , se construye interiormente un tri angulo equil atero AEF . EF corta BC en I , y se construye exteriormente un tri angulo equil atero BIJ . Encuentre EJB . 32. (*) En la gura 67, se sabe que AO1 B AO2 B = 70 y adem as la tangente EB forma el tri angulo is osceles ABE , con AB = AE . Encuentre EBC .

Figura 67

52

1.6 La Circunferencia

1 Geometr a

33. (*) Dos circunferencias 1 y 2 se cortan en A y B . Una recta por A corta a 1 y 2 en C y D, respectivamente, y la paralela a CD por B corta 1 y 2 en E y F , respectivamente. Demuestre que CDB EAF . 34. (*) La Recta de Simson-Wallace. Sean X , Y y Z los pies de las alturas trazadas desde un punto P en el circunc rculo del ABC hacia AB , BC y CA, respectivamente. Demuestre que X , Y y Z est an alineados. 35. (*) Sea P un punto exterior al cuadrado ABCD tal que AP C = 90 , Q es la intersecci on de AB y P C , y R el pie de la perpendicular por Q a CA. Demuestre que P , R y D est an alineados. 36. En la gura 68, ABCD es un trapecio rect angulo tal que la circunferencia de di ametro AB (y centro O) es tangente a CD. Demostrar que O pertenece a la circunferencia de di ametro CD y que esta circunferencia es tangente a BA.

Figura 68 37. El ABC es rect angulo en C , la circunferencia de centro O es tangente a cada uno de los lados del ABC en los puntos P , Q y R (como se muestra en la gura 69), y se cumple que AP = 20 y BP = 6. Calcule OP .

Figura 69 38. Los v ertices A y B de un tri angulo equil atero ABC est an sobre una circunferencia de radio 1 y el v ertice C est a en el interior de la circunferencia. Un punto D (distinto de B ) que esta en la circunferencia es tal que AD = AB . La recta DC corta por segunda vez a la circunferencia en E . Encuentre la longitud del segmento CE . Ver gura 70.

53

1.6 La Circunferencia

1 Geometr a

Figura 70 39. En la gura 71 se muestran tres semicircunferencias, una de di ametro AB (de centro O y radio r), otra de di ametro AO y la u ltima de di ametro OB . Determine la raz on entre el radio de la circunferencia tangente a estas tres semicircunferencias y r.

Figura 71 40. El segmento AB es di ametro de un semic rculo con centro en O. Un c rculo con centro en P es tangente a AB en O y tambi en al semic rculo. Otro c rculo con centro en Q es tangente a AB , al semic rculo y al c rculo de centro en P . Si AB = 2, cu al es el radio del c rculo con centro en Q?

Figura 72 41. (*) (OIM 2002, P-4) En un tri angulo escaleno ABC se traza la bisectriz interior BD, con D sobre AC . Sean E y F puntos sobre la recta BD tales que (AE CF ) BD, y sea M el punto sobre el lado BC tal que DM BC . Demuestre que EM D = DM F . 42. (*) (OMCC 2003, P-2) Sea S una circunferencia y AB un di ametro de ella. Sea t la recta tangente a S en B y considere dos puntos C y D en t tales que B este entre C y D. Sean E 54

ngulos. 1.7 Semejanza de Tria

1 Geometr a

y F las intersecciones de S con AC y AD y sean G y H las intersecciones de S con CF y DE . Demuestre que AH = AG. 43. (*) (The 59th Romanian Mathematical Olympiad District Round) Considere un cuadrado ABCD y un punto E sobre el lado AB . La diagonal AC corta al segmento DE en el punto P . La perpendicular por P a DE corta al lado BC en F . Probar que EF = AE + CF . 44. (*) Teorema de Arqu medes: En la gura 73, la regi on delimitada por tres semicircunferencias mutuamente tangentes, es conocida como cuchilla de zapatero o arbelos. Demostrar que las circunferencias sombreadas son congruentes.

Figura 73: Teorema de Arqu medes.

1.7.
1.7.1.

Semejanza de Tri angulos.


Proporcionalidad

Denici on 1. Raz on: se llama raz on, al cociente de dos cantidades, expresadas en la misma magnitud, por a ejemplo b .
c 18 2. Proporci on: se llama proporci on a la igualdad de dos razones. Por ejemplo a = d , a los b t erminos a y d se les llama extremos y los t erminos b y c se les llama medios, al t ermino d se le llama cuarta proporcional entre a, b y c en este orden.

Propiedades de las proporciones: 1. 2. 3. 4. a c = si y s olo si a c = b d. b d a c b d a b = si y s olo si = o = . b d a c c d a c ab cd = si y s olo si = . b d b d a c a+b c+d = si y s olo si = . b d ab cd

18 En algunos textos de geometr a se utiliza la notaci on de proporci on as a : b :: c : d que se lee a es a b como c es a d.

55

ngulos. 1.7 Semejanza de Tria 1.7.2. Teorema de Thales

1 Geometr a

Denici on 1. Un punto P AB divide al segmento AB en una raz on dada r, si


PA PB

= r.

2. Sean AB y CD dos segmentos, y sean P AB y Q CD, decimos que P y Q dividen a CQ AP AB y CD en segementos proporcionales si P = QD . B

Figura 74 Teorema de Thales. Si tres paralelas cortan a dos secantes entonces los segmentos que determinan en ellas son proporcionales. 19 Antes de demostrar el Teorema de Thales, se enunciar an dos teoremas que a pesar de su aparente sencillez es de mucha utilidad en problemas que involucran Areas y Proporcionalidad. Lema 1. Sea AB CD. Demuestre que: (ABC ) = (ABD). ABC . Pruebe que:

Lema 2. Sea P un punto sobre el lado AB (o su prolongaci on) del (AP C ) AP = PB (P BC )

A continuaci on se enuncian los pasos a seguir en la demostraci on del teorema de Thales. Demostraci on. Sean AA , BB y CC rectas paralelas que cortan a dos secantes en los puntos A, A , B , B , C , C respectivamente (ver gura 75). Pruebe que: 1. 2. AB (ABB ) = BC (BCB ) AB (A B B ) . = BC (B C B )

3. (ABB ) = (A B B ) y (BCB ) = (B C B ).
El teorema de Thales puede enunciarse de manera general como sigue: Si tres o m as paralelas cortan a dos o m as secantes entonces los segmentos que determinan en ellas son proporcionales.
19

56

ngulos. 1.7 Semejanza de Tria

1 Geometr a

Figura 75: Teorema de Thales Con ayuda de las igualdades demostradas concluya que: AB AB = . BC BC Observaci on Importante: Utilice las propiedades de las proporciones para demostrar las equivalencias siguientes (interpr etelas geom etricamente): AB AC AC AC AC AB = = = BC BC AB AB BC BC Sea ABC un tri angulo, sabemos que su area puede calcularse al multiplicar la longitud de uno de sus lados por la longitud de la altura correspondiente a ese lado. Si denotamos por a la longitud de un lado del ABC y ha la longitud de la altura correspondiente, el area del ABC se denota por (ABC ) y es igual a: a ha 2 Ahora estamos en condiciones de probar el siguiente lema: Lema:Si dos triangulos tienen una misma altura entonces la razon entre sus areas es igual a la razon de las bases donde se levanta la altura comun. Demostracion:Sean ABC y A B C triangulos con alturas iguales h. Sean D y D los pies de las alturas trazadas de los vertices A y A a los lados BC y B C respectivamente. Entonces: (ABC ) = La razon entre las areas es: (ABC ) = (A B C ) BC h BC h (A B C ) = 2 2
BC h 2 B C h 2

BC BC

Como queriamos demostrar. El siguiente lema se puede probar de manera analoga al anterior: Lema:Si dos triangulos tienen una base igual entonces la razon de sus areas es igual a la razon entre las alturas que se levantan sobre la base igual. Corolario (Teorema de Thales en el tri angulo). Toda recta paralela a un lado de un tri angulo y que corte a los otros dos lados, divide a estos lados en segmentos proporcionales.

57

ngulos. 1.7 Semejanza de Tria

1 Geometr a

Rec proco del Teorema de Thales. Si tres rectas cortan a dos secantes en segmentos proporcionales y dos de estas rectas son paralelas entonces las tres rectas son paralelas. Demostraci on. Sean AA , BB y CC rectas que cortan a dos secantes en los puntos A, A , B , AB AB B , C , C respectivamente, tales que AA CC y = . Por el punto B tracemos una BC BC recta paralela a AA , la cual interseca a A C en el punto D (ver gura 76). Entonces, por el AD AB AD AB = . De donde, = , as por las propiedades Teorema de Thales se tiene que: BC DC BC DC AC AC = , por lo que B C = B D + DC = DC y por tanto B D = 0, o de las proporciones BC DC equivalentemente B = D y por lo tanto, BB AA .

Figura 76: Rec proco del Teorema de Thales Corolario (Rec proco del Teorema de Thales en el tri angulo.) Si una recta intercepta dos lados de un tri angulo en segmentos proporcionales entonces la recta es paralela al tercer lado del tri angulo. Teorema:Sea ABC un triangulo y sean D y E puntos en los lados AB y AC respectivamente. Si se cumple que: AC AB = AD AE entonces DE es paralela a BC . Supongamos que, por el contrario DE no es paralela a BC . Sea C un punto en AC , distinto de AB C , tal que BC es paralela a DE , entonces por el teorema de Thales, AD = AC . Pero ademas, se AE AB AC cumple por hipotesis del problema, que AD = AE . Podemos entonces decir que AC = AB AE = AC AD AC = AC Por tanto, los puntos C y C deben ser iguales y DE es paralela a BC . Teorema: Consideremos tres rectas paralelas y dos rectas transversales a estas como se muestra AB en la gura. Tenemos que si AD, BE y CF son paralelas entonces BC = DE . Reciprocamente, si EF AB DE = y dos de las rectas AD, BE o CF son paralelas entonces las rectas son paralelas. BC EF Demostracion. Diremos que G es el punto de interseccion de AF con BE . Si aplicamos el teorema de Thales y su reciproco en los triangulos ACF y F DA, vemos que las rectas AD, BE, CF son paralelas si y solo si AB AG G FE = GF y F = ED BC GA 58

ngulos. 1.7 Semejanza de Tria

1 Geometr a

AB AG Luego, son paralelas si y solo si BC =F = DE . G EF AB Ahora, supongamos que BE y CF son paralelas y que la otra recta AD cumple que BC = DE . EF Denimos G como el punto de interseccion de AF con BE . Como BE y CF son paralelas, se AG AB AG AB = GF . Ahora bien, como BC = DE tenemos que, GF = DE . Luego, el primer cumple que BC EF EF teorema de Thales, GE es paralela a AD y de ahi que BE es tambien paralela AD.

1.7.3.

Criterios de Semejanza de Tri angulos ABC es semejante al A B C (Ver gura 77), lo cual

Tri angulos semejantes. Decimos que el denotamos as ABC A B C , si:

AB AC BC = = AB AC BC y BAC = B A C , ABC = A B C , ACB = A C B .

Figura 77: Denici on de Semajanza de Tri angulos. En los tres teoremas que se muestran a continuaci on (los cuales son una consecuencia directa del Teorema de Thales) se establecen las condiciones m nimas para demostrar que dos tri angulos son semejantes, a los cuales denominaremos: Criterios de Semejanza de Tri angulos. Primer criterio de semejanza de tri angulos: Angulo-Angulo A-A. Si dos a ngulos de un tri angulo son congruentes con dos a ngulos de otro tri angulo, entonces los dos tri angulos son semejantes. Demostraci on. Supongamos que en el ABC y A B C se tiene que ABC = A B C y ACB = A C B , entonces BAC = B A C (Por la suma de angulos internos en un tri angulo). Sea D AB y E AC tales que AD = A B y AE = A C , dado que DAE = BAC = B A C , se sigue por L-A-L que ADE = A B C , por consiguiente ADE = A B C = ABC , de donde DE BC (por ser iguales los a ngulos correspondientes) y por el teorema de Thales AB AC = AD AE y por consiguiente AB AC = (2) AB AC 59

ngulos. 1.7 Semejanza de Tria

1 Geometr a

Sea F BC tal que DF AC , entonces F C = DE = B C (porque DECF es paralelogramo y por ser ADE = A B C ) y por el teorema de Thales BA BC = DA FC o lo que es lo mismo AB BC = AB BC Luego, de (1) y (2) se tiene que: AC BC AB = = . AB AC BC As , se ha demostrado que los tres pares de a ngulos son congruentes y los tres pares de lados son proporcionales, por lo tanto, ABC A B C . Segundo criterio de semejanza de tri angulos: L-A-L. Si un angulo de un tri angulo es congruente con otro a ngulo de otro tri angulo y los lados que comprenden al a ngulo en el primer tri angulo son respectivamente proporcionales a los lados que comprende al a ngulo en el segundo tri angulo, entonces los dos tri angulos son semejantes. AC AB = . Considere los puntos Demostraci on. Suponga que el BAC = B A C y que AB AC D y E , como en la demostraci on del teorema anterior. Entonces por el criterio L-A-L, ADE = AB AC A B C , de lo cual se deduce que ADE = A B C . Por otra parte tenemos que: = ,y AD AE al aplicar el rec proco del teorema de Thales, se puede armar que DE BC , de lo cual a su vez se deduce que ADE = ABC , por angulos correspondientes entre paralelas. Finalmente por transitividad se concluye que ABC = A B C . Por lo tanto, ABC A B C (Por el criterio A-A.) Tercer criterio de semejanza de tri angulos: L-L-L. Si los tres lados de un tri angulo son respectivamente proporcionales a los tres lados de otro tri angulo, entonces los dos tri angulos son semejantes. AB AC BC Demostraci on. Por hip otesis se tiene que: = = y como antes sean D y E AB AC BC puntos sobre AB y AC respectivamente tales que AD = A B y AE = A C . Entonces por el rec proco del teorema de Thales se tiene que DE BC y por consiguiente el ABC = ADE AB BC y el ACB = AED, de donde ABC ADE (por el criterio A-A). Por ende = , AD DE BC BC luego por transitividad = , de donde DE = B C . En consecuencia ADE = A B C DE BC (por el criterio L-L-L), de lo cual se sigue que A B C = ADE y A C B = AED, y por transitividad A B C = ABC y A C B = ACB =. Por lo tanto, A B C ABC (Por el criterio A-A.) 1.7.4. Potencia de Punto (3)

Proposici on. Si dos cuerdas AB y CD de una circunferencia se intersectan en un punto P , entonces P A P B = P C P D.

60

ngulos. 1.7 Semejanza de Tria

1 Geometr a

Demostraci on. Si P es un punto sobre la circunferencia est a claro que ambos productos son 0 y por tanto iguales. Si P es un punto dentro de la circunferencia entonces el cuadril atero ADBC es c clico con los a ngulos BAD = BCD y ADC = ABC y por tanto los tri angulos PD PA = y despejando obtenemos que AP D y BP C y por tanto se cumple las proporciones P C PB P A P B = P C P D. Si P es un punto fuera de la circunferencia el cuadril atero ABDC es ciclico y los a ngulos CAP y BDP son iguales y los tri angulos P AC y P BD son tri angulos semejantes con lo que se PA PD cumple de nuevo en este caso que P = y por tanto, P A P B = P C P D. C PB Proposici on 2. Si A, B y C son puntos sobre una circunferencia y si la tangente en C, intersecta en un punto P a la prolongaci on de la cuerda AB , entonces P C 2 = P A P B . Demostraci on. Sabemos que un angulo seminscrito es igual a un a ngulo inscrito que sostenga el mismo arco asi pues, el a ngulo P CA = CBP y dado que, CP A = CP B los tri angulos PC PA 2 P CA y BCP son semejantes y se cumple la raz on P C = P B y por tanto, P C = P A P B . Ahora veremos los inversos de las proposiciones anteriores. Proposici on 3. a. Si AB, CD son dos segmentos que se intersectan en P de manera que, como segmentos dirigidos, P A P B = P C P D entonces A,B , C y D se encuentran sobre una circunferencia. b. Si A, B , C, y P son puntos tales que P, A y B est an alineados y P C 2 = P A.P B , entonces P C es tangente en C al circunc rculo del tri angulos ABC . Demostraci on. Denotemos por la circunferencia que inscribe al tri angulo ABC . a. Supongamos que D no est a sobre la circunferencia y sea D la intersecci on de P C con . Por la proposici on 1 P A P B = P C P D y dado que la hip otesis dice que P A P B = P C P D, tenemos que P D = P D , por lo que D y D son iguales. Contradicci on. Por tanto, D debe estar sobre la circunferencia. b. Sea C la otra intersecci on de P C con . Por la primera proposici on, P A P B = P C P C y 2 como por hip otesis P A P B = P C , obtenemos que P C = P C y entonces C coincide con C . Podemos concliur que P C es tangente a en C . Para cualquier punto P , una circunferencia C y cualquier recta trazada por P que corte a C en puntos A y B (A y B pueden ser iguales) que: el producto P A P B es constante y se denomina la potencia de punto del punto P con respecto a C . Otra forma de calcular la potencia de un punto se comentara a continuaci on. Si P es exterior a C 2 y P C es tangente a C por P , la potencia es P C . Si O es el centro de la circunferencia C y radio r, tenemos que, por el teorema de Pit agoras: P C 2 = P O2 r2 . Ahora bien, si P es un punto dentro de la circunferencia y que est a sobre un di ametro AB , podemos calcular la potencia como P A P B . Pero vemos que P A lo podemos escribir como r P O 61

ngulos. 1.7 Semejanza de Tria

1 Geometr a

y P B como r + P O, o viceversa, dependiendo de si P esta mas cerca de A o de B . De ah que P A P B = (r P O)(r + P O) = r2 P O2 . Si un punto esta sobre la circunferencia sabemos que su potencia es cero, lo cual tambi en es descrito por r2 P O2 . Para teminar podemos decir que la potencia de punto de P con respecto a la circunferencia C = (O, r) es P O2 r2 . Adem as, la potencia es positiva, cero o negativa, dependiendo si P se encuentra fuera, sobre o dentro de la circunferencia. Eje Radical Problema. Cual es el lugar geom etrico de los puntos cuya potencia de punto con respecto a dos circunferencias no conc entricas?

1.7.5.

Problemas

1. Sean AB y CD las bases del trapecio ABCD, cuyas diagonales se intersecan en E perpendicularmente. Si AD = 13, AE = 12 y CE = 4 encuentre las longitudes de CD y AB . 2. En la gura 78, el ABC es equil atero, sus lados tienen longitud 3 y P A es paralela a BC . Si P Q = QR = RS , encontrar la longitud de CS .

Figura 78 3. Sea ABCD un trapecio de bases BC y AD, sus diagonales se cortan en E . Si BE = 3, ED = 4 y CE = 2, determine la medida de AE . 4. Las bases de un trapecio miden 3 y 5, y si su altura mide 4. Encontrar la distancia desde el punto de corte de las diagonales hasta la base mayor.

5. En la gura adjunta, el ABC es rect angulo en A y el ADB es rect angulo en D. El punto E es el punto de intersecci on de los segmentos AD y BC . Si AC = 15, AD = 16 y BD = 12, calcule el a rea del ABE .

62

ngulos. 1.7 Semejanza de Tria

1 Geometr a

6. El ABC es rect angulo en B . Se dibuja un rect angulo BEDF con D sobre la hipotenusa, BC E y F sobre BC y AB , respectivamente. Si AB = 1, demuestre que BE = 11 . DE 7. Consid erese los puntos A, B , C y D tales que A y B est an sobre el segmento OC y OD respectivamente, donde O es el centro de la circunferencia de radio r (Ver gura 79). Si OA OC = r2 r2 = OB OD, demuestre que el AOB DOC y que CD = AB .20 OA OB

Figura 79 8. Sobre la circunferencia de centro O, se trazan los di ametros AB y CD tales que AB CD. Sea P un punto sobre el arco CBD y Q el punto de intersecci on de las cuerdas AP y CD. Si DO = 1, demuestre que AP AQ = 2. 9. Un segmento de recta AB es divido por los puntos interiores K y L de manera que AL2 = AK AB . Sea P un punto exterior al segmento AB tal que AP = AL. Pruebe que KP L = LP B . Figura 80.

Figura 80 10. En la gura 81, AB y AC son tangentes a la circunferencia, y CE BD, siendo BD un di ametro. Probar que BE BO = AB CE . 11. Demostrar que 1 1 1 + = si se cumple que AX AX BY CZ BY CZ . (Ver gura 82.)

12. En la gura 83, el ABC es rect angulo. Se construyen exteriormente los cuadrados ABEF y BCP Q. Demostrar que BM = BN . 13. Sean O, P y R los centros de las tres circunferencias. Si OR = r y Q es la intersecci on de P O con la circunferencia de centro R, demuestre que OP OQ = r2 . Ver gura 84.
20

La medida del segmento CD se denomina Distancia Inversa.

63

ngulos. 1.7 Semejanza de Tria

1 Geometr a

Figura 81

Figura 82

Figura 83: . 14. Si en un tri angulo rect angulo se traza la altura correspondiente a la hipotenusa, entonces: a ) Los dos nuevos tri angulos que resultan, son semejantes entre si y semejantes al tri angulo original. b ) La altura es media proporcional hipotenusa.
21

entre los segmentos que ella determina sobre la

c ) Cada cateto es media proporcional entre la hipotenusa y la proyecci on del cateto sobre
b Si b es una magnitud tal que a b = c , entonces decimos que b es media proporcional entre a y c,o de manera equivalente: b es media proporcional entre a y c si y solo si b2 = a c. 21

64

ngulos. 1.7 Semejanza de Tria

1 Geometr a

Figura 84 la hipotenusa. d ) Demuestre el teorema de Pit agoras. 15. Si dos tri angulos tienen sus lados respectivamente paralelos o respectivamente perpendiculares, entonces los dos tri angulos son semejantes. 16. Las alturas, las bisectrices y las medianas hom ologas de dos tri angulos semejantes est an en la misma raz on que sus lados hom ologos.
AB BC CA 17. Sean ABC y A B C dos tri angulos semejantes con A =B =C = k . Demuestre que: B C A la raz on entre los per metros de los tri angulos es k y que la raz on entre sus areas es k 2 .

18. Teorema de Menelao. Dado el ABC , sea P un punto sobre la recta AB , Q un punto sobre la recta BC , R un punto sobre la recta CA. Si los puntos P , Q, R est an alineados AP BQ CR entonces = 1. P B QC RA

Figura 85: Teorema de Menelao. Para demostrar este teorema, sea W un punto sobre la recta P QR tal que BW a ) Demuestre que los tri angulos AP R y BP W son semejantes. b ) Demuestre que los tri angulos CQR y BQW son semejantes. AP BQ CR c ) De los literales a ) y b ) deduzca que = 1. P B QC RA 65 AC :

ngulos. 1.7 Semejanza de Tria

1 Geometr a

19. Teorema de Ceva. Dado el ABC , sea P un punto sobre el recta AB , Q un punto sobre la recta BC y R un punto sobre la recta CA. Si las rectas AQ, CP , BR concurren, AP BQ CR entonces = 1. P B QC RA

Para demostrar este teorema, sean W y V los puntos de intersecci on de la recta que pasa por B paralela a AC , con las rectas CP y AQ, respectivamente. a ) Demuestre que b ) Demuestre que AP C BW O BP W y que RCO y que AQC V QB .

BV O RAO. AP BQ CR = 1. c ) Utilice los literales a ) y b ) para probar que P B QC RA

20. Sea ABC un tri angulo, con D sobre AB , E sobre AC y F sobre BC , tal que DE es paralelo a BC , Demuestre que BF = F C 21. Sea ABC un tri angulo, con D sobre AB , con E sobre BC y F sobre AC , tal que AD = 2BD y F A = 2CF . Demuestre que E es punto medio de B . 22. Sea una circunferencia y dado un tri angulo ABC , sean L, L , M, M , N, N los puntos de corte de la circunferencia con el tri angulo , sobre los lados BC, AC, AB respectivamente. Demostrar que si AL, BM, CN concurren, entonces AL , BM , CN concurren. 23. En el tri angulo ABC , rect angulo en A , se consideran las circunferencias inscritas y circunscritas. La recta AM es tangente a la circunferencia circunscrita en el punto A (M es punto de BC ). S y R son los puntos de tangencia de la circunferencia inscrita con los catetos AC y AB , respectivamente. La recta RS corta a la recta BC en N . Las rectas AM y SR se cortan en U . Demostrar que el tri angulo U M N es is osceles. 24. Demuestre que si ABC es un tri angulo y AA es su bisectriz externa (con A sobre BC ) AB entonces BA = AC AC 25. Demuestre que si ABC es un tri angulo y suponga que las bisectrices internas de B y C cortan a CA y AB en b y C respectivamente y que la bisectriz externa de A corta a BC en 66

ngulos. 1.7 Semejanza de Tria A . Demuestre que A , B , C son colineales.

1 Geometr a

26. Demuestre que si ABC es un tri angulo. Demuestre que los puntos en los que las bisectrices externas cortan a su lado opuesto correspondientes son colineales. 27. Sea ABC un tri angulo, se denen los puntos A , B , C tales que los segmentos AA , BB , CC son las bisectrices de ABC . Sean A , B , C tales que si las rectas l, m, n son las mediatrices de AA , BB , CC y adem as A es la intersecci on de l con BC , B es la intersecci on de m con AC y C es la intersecci on de n con BA. Demuestre que A , B , C son colineales. 28. Si dos cuerdas se interceptan en el interior de una circunferencia entonces el producto de las medidas de los segmentos determinados por el punto de intersecci on en una de las cuerdas es igual al producto de las medidas de los segmentos determinados en la otra cuerda. 29. Si dos segmentos se interceptan en un punto que esta en el interior de los dos segmentos y el producto de las medidas de los segmentos determinados por el punto de intersecci on en el primer segmento es igual al producto de las medidas de los segmentos determinados por el punto en el segundo segmento,entonces los extremos de los segmentos est an sobre una circunferencia. 30. Si desde un punto P exterior a una circunferencia se trazan dos semirrectas secantes que cortan a la circunferencia en los puntos A, B y C , D respectivamente, entonces P A P B = P C P D. 31. Si desde un punto P se trazan dos semirrectas con los puntos A, B sobre una y los puntos C , D sobre la otra, tales que P A P B = P C P D, entonces los puntos A, B , C , D est an sobre una circunferencia. 32. Si desde un punto exterior a una circunferencia se trazan dos semirrectas, una tangente y la otra secante, entonces el segmento entre el punto y el punto de tangencia es media proporcional entre los segmentos determinados entre el punto exterior y los puntos de intersecci on 22 de la secante con la circunferencia. 33. Si P es un punto sobre el mismo plano que una circunferencia de centro O y radio r, y d es la distancia del punto P al centro O de la circunferencia, demuestre que: a ) Si P est a en el interior de la circunferencia, entonces la potencia de P es r2 d2 . b ) Si P est a en el exterior de la circunferencia, entonces la potencia de P es d2 r2 . c ) Si P est a sobre de la circunferencia, entonces la potencia de P es cero. 34. En un tri angulo ABC los puntos D y E est an en los lados AC y AB de forma tal que los a ngulos ADE = ABD. Si AE = 2 y BE = 3 encuentre AD.
Los problemas anteriores nos permite establecer la siguiente denici on de Potencia de un punto con respecto a una circunferencia: La potencia de un punto P con respecto a una circunferencia de centro O y radio r es el producto P A P B , donde A y B son los puntos de intersecci on de la circunferencia con una recta que pasa por P .
22

67

ngulos. 1.7 Semejanza de Tria

1 Geometr a

35. El cuadril atero ABCD es c clico y la intersecci on de los lados AC y BD es P . Halle BP si P C = 4, DP = 2 y AP = 8. 36. En un cuadril atero ABCD los angulos en A y en C son rectos. Si P es el punto de intersecci on de AC con BD y los lados AB = 3, AD = 4 y DP = 2, hallar AP P C . 37. Por un punto P sobre la cuerda com un AB de dos circunferencias que se intersectan se trazan las cuerdas KM sobre el primer c rculo y LN sobre el segundo c rculo. Pruebe que el cuadril atero KLM N es conc clico. 38. En el tri angulo ABC con circuncentro O, las alturas BE y CF se cortan en H. Las rectas OB y HC se cortan en P . Si HP = 3, OP = 2 y P C = 5, cu anto vale OB ? 39. La recta OA es tangente a una circunferencia en el punto A y la cuerda BC es paralela a OA. Las rectas OB y OC intersectan a la circunferencia por segunda vez en los puntos K y L, respectivamente. Pruebe que la recta KL divide al segmento OA por la mitad. 40. En el paralelogramo ABCD la diagonal AC es mas larga que la diagonal BD. Sea M un punto en la diagonal AC tal que el cuadril atero BCDM es c clico. Pruebe que la linea BD es una tangente com un a los tri angulos ABM y ADM . 41. Dada la circunferencia S y los puntos A y B fuera de esta. Por cada linea L que pasa por el punto A e intersecta a la circunfenrencia S en los puntos M y N considere la circunc rculo del BM N . Pruebe que todas las circunferencias tienen un punto en com un distinto de B . 42. Dada una circunferencia S , los puntos A y B sobre esta y C en la cuerda AB . Para cualquier circunfenrencia S tangente a la cuerda AB en el punto C y que intersecta a S en P y Q, considere el punto de intersecci on M de las rectas AB y P Q. Pruebe que la posici on de M no depende de la eleccci on de S 43. Desde A se trazan una recta tangente a la circunferencia S en C y una recta que corta a S en los puntos ByD. Sea E otro punto en la circunferencia tal que CE es tangente a AD. Si CA = 6, AB = 12 y CE = 4BD hallar CE y BD. Si P es el punto de intersecci on de CD con BE cual es la potencia de punto de P con respecto a S . 44. Dos circunferencias se intersectan en los puntos A y B y sea M N su tangente comun. Pruebe que la recta AB divide a M N por la mitad.

68

ngulos. 1.7 Semejanza de Tria

1 Geometr a

45. Pruebe que el eje radical de dos circunferencias es el lugar geom etrico de los puntos P tales que al trazar las tangentes a las circunferencias la distancia de P a ambos puntos de tangencia es la misma. 46. A partir de un punto P exterior a una circunferencia de centro O se trazan las tangentes P Q y la secante P BA, tal que P Q = AB = 2, si el radio de la circunferencia mide 1 + 5 , hallar BOP . 47. (*) (IV OMCC, P-4) Sea ABC un tri angulo, D el punto medio de BC , E un punto sobre el segmento AC tal que BE = 2AD y F el punto de intersecci on de AD con BE . Si CAD = 60, encuentre la medida de los a ngulos del F EA. 48. (*) Sea ABCD es un trapecio con AD BC . M y N son los puntos medios de CD y BC , M respectivamente, y P el punto com un de las rectas AM y DN . Si P = 1 , demuestre que AP 4 ABCD es paralelogramo. 49. Dado el ABC se construye un cuadrado P QRS con P en AB , Q en AC , R y S en BC . Sea H el pie de la altura desde A hacia BC . Demuestre que: a) 1 1 1 = + PQ AH BC b ) (ABC ) = 2(P QRS ) si y s olo si AH = BC .

50. Sea P un punto en el interior del ABC . Se trazan por P las paralelas a los lados del tri angulo, que queda dividido en tres tri angulos y tres paralelogramos. Si las a reas de los tres tri angulos de la subdivisi on son, en alg un orden, 9, 16 y 25, hallar el a rea del ABC .

51. (*) En la gura anexa, BC = CD = DE = EA = x y AEB = 90. Demuestre que ABC + ACD + ADE = 90.

52. Las tres circunferencias de la gura 86 tienen el mismo radio r, sus centros son colineales y la circunferencia de centro O2 es tangente a las otras dos. Por A se traza una tangente a la circunferencia de centro O3 . Obtenga el valor del segmento BC en funci on de r. 53. Sea ABCD un rombo, con AC = 6 y BD = 8. Se construyen exteriormente los cuadrados ADEF y CDHG, cuyos centros son O1 y O2 , respectivamente (Vea gura 87). Calcular la medida del segmento O1 O2 . 54. Sea ABCD un cuadrado con P y Q sobre AB y BC tales que BP = BQ. Sea H el pie de la perpendicular de B a P C . Demuestre que DHQ = 90. 55. Dado un paralelogramo ABCD, se trazan dos circunferencias tangentes externamente en P , y tales que la primera es tangente internamente al ABC y la otra es tangente internamente al CDA, como en la gura 88. Demuestre que B , P y D est an alineados. 69

ngulos. 1.7 Semejanza de Tria

1 Geometr a

Figura 86

Figura 87

Figura 88 56. Alrededor de una circunferencia se construyen diez circunferencias tangentes a la original y tangentes entre s (V ease gura 89). Demuestre que la suma de las areas de las diez circunferencias es el doble del area de la circunferencia mayor.

70

ngulo. 1.8 Puntos y Rectas Notables del Tria

1 Geometr a

Figura 89 57. En un ABC el CAB = 120. Encuentre la medida de la bisectriz interna del CAB en funci on de los lados adyacentes. 58. El ABC tiene lados de 13, 14 y 15 unidades. El A B C est a dentro del ABC con lados paralelos a los de este y a 2 unidades de distancia de los lados del mismo. Calcule (ABC ) (A B C ). 59. (*) (Asi atico Pac ca) Sea ABC un tri angulo y D el pie de la altura con respecto a A. Sean E y F puntos en una recta que pasa por D (distintos de D) tales que AE CE y AF BF . Sean M y N los puntos medios de BC y EF , respectivamente. Demuestre que AN N M .

1.8.
1.8.1.

Puntos y Rectas Notables del Tri angulo.


Medianas

Denici on: En un tri angulo, una mediana es el segmento de recta que une un v ertice con el punto medio del lado opuesto. Teorema: Las tres medianas de un tri angulo concurren en un punto llamado el Centroide 23 del tri angulo y usualmente es denotado por G. Adem as, las medianas se cortan mutuamente en raz on 2:1. Demostraci on: Dado el ABC sean A , B , C , los puntos medios de BC , CA, AB , respectivamente. Dena G como la intersecci on de BB con CC . Por el teorema de la base media, B C BC y 2B C = BC ; observe que BCG B C G, con raz on de semejanza 2, por lo que GC GB = =2 GB GC An alogamente, si G = AA BB se cumple G A G B = =2 G B G A
23

Tambi en conocido como Geocentro, Centro de Gravedad, Baricentro, o m as formalmente Equibaricentro.

71

ngulo. 1.8 Puntos y Rectas Notables del Tria

1 Geometr a

As , G y G dividen al segmento BB en dos segmentos cuya raz on es 2:1, por lo que G = G , lo cual implica que AA , BB , CC concurren y GA GB GC = = =2 GA GB GC 1.8.2. Mediatrices

Denici on: Dado un segmento AB , la mediatriz del segmento es el lugar geom etrico de puntos que equidistan de A y B , i.e., un punto P est a sobre la mediatriz de AB si y s olo si P A = P B . Teorema: La mediatriz de AB es una recta l perpendicular a AB y que pasa por su punto medio. Demostraci on: Sea M el punto medio de AB , y l pasa por M y l AB . En primer lugar se probar a que todos los puntos de l satisfacen la denici on de mediatriz: Por denici on de punto medio M A = M B . por lo que claramente M pertenece a la mediatriz de AB ; sea P un punto de l distinto de M , por criterio LAL, P M A P M B por lo que P A = P B . Ahora, cabe preguntarse si existe alg un punto fuera de l que tambi en cumpla la denici on: suponga P tal que P A = P B , esto implica que P AB es is osceles, y entonces P AB = P BA; si M es la proyecci on de P sobre AB , por criterio ALA P AM P BM , lo cual implica que M A = M B , es decir que M = M , y esto obliga a que P est e sobre l (ya que P M = l). Teorema: Las mediatrices de un ABC concurren en un punto que equidista de los v ertices del tri angulo, llamado el Circuncentro del ABC

Figura 90: Concurrencia de Mediatrices, Circuncentro y Circunc rculo. Usualmente, el circuncentro es denotado por O, y R representa la distancia del circuncentro a los v ertices R = OA = OB = OC A esta distancia se le llama Circunradio del ABC . As , O es el centro de una circunferencia que pasa por A, B , C , cuyo radio es R, llamada el Circunc rculo del ABC .24
24

En ocasiones, denotaremos a esta circunferencia por (ABC ).

72

ngulo. 1.8 Puntos y Rectas Notables del Tria

1 Geometr a

Demostraci on: Sea O la intersecci on de las mediatrices de AB y BC , por el teorema anterior, como O pertenece a la mediatriz de AB se cumple OA = OB , y como tambi en pertenece a la mediatriz de BC , OB = OC ; entonces OC = OA, y utilizando de nuevo el teorema anterior, O debe pertenecer a la mediatriz de CA. As , las tres mediatrices concurren en O, y este punto equidista de los v ertices del ABC . Corolario: Dado un tri angulo, existe una circunferencia que pasa por los tres v ertices (el circunc rculo); adem as, esta circunferencia es u nica. Una observaci on importante es que la mediatriz del lado de un tri angulo NO siempre pasa por el v ertice opuesto; de hecho, esto s olo se da si el tri angulo es is osceles. 1.8.3. Alturas

La altura es un concepto que est a intr nsecamente relacionado con la distancia de un punto a una recta ; la altura es la recta que debe trazarse para determinar esta distancia, i.e., es una recta que pasa por el punto y es perpendicular a la recta. A la intersecci on entre la altura y la recta generalmente se le llama pie de la altura, o tambi en (m as formal) proyecci on del punto sobre la recta. En particular, para tri angulos, deniremos la altura de la siguiente forma: Denici on: Dado un tri angulo, una altura es una recta que pasa por un v ertice y es perpendicular al lado opuesto. Es importante observar que el pie de la altura NO siempre pertenece a un lado; de hecho, una altura puede estar al interior de un tri angulo, coincidir con un lado, o estar completamente afuera de un tri angulo. Teorema: Las alturas de un tri angulo concurren en un punto, llamado el Ortocentro del tri angulo, 25 usualmente denotado por H. Demostraci on: Dado el ABC , se construyen los puntos A1 , B1 , C1 , tales que ABA1 C , BCB1 A, CAC1 B son paralelogramos. Observe que el ABC es el tri angulo medial del A1 B1 C1 , y que las alturas del ABC son las mediatrices del A1 B1 C1 ; como las mediatrices de un tri angulo concurren (en este caso, las del A1 B1 C1 ), las alturas del ABC concurren. La altura tambi en puede escribirse en t erminos de lugar geom etrico: Teorema: La recta l es perpendicular a AB si y s olo si AL2 LB 2 es constante. Es decir, que una recta perpendicular a AB es el lugar geom etrico de los puntos L que satisfacen la condici on anterior. Demostraci on: sea P la intersecci on de l con AB , y L un punto arbitrario sobre l; por Pit agoras 2 2 2 2 se tiene AL LB = AP P B , y el t ermino derecho de la igualdad es constante. La otra direcci on de la implicaci on se prueba por contradicci on.
25

El tri angulo formado por los pies de las alturas de un

ABC es llamado el tri angulo ortico del

ABC .

73

ngulo. 1.8 Puntos y Rectas Notables del Tria

1 Geometr a

De esa denici on tambi en puede fabricarsele una demostraci on del teorema anterior, sin embargo, no se aborda porque la prueba se basa en un resultado sosticado llamadado Teorema de Steiner.26 1.8.4. Bisectrices

Denici on: La bisectriz de un angulo es una recta que divide al a ngulo en dos a ngulos de igual magnitud. Teorema: El lugar geom etrico de puntos que equidistan de dos rectas dadas, generan un par de rectas perpendiculares llamadas bisectriz interna y bisectriz externa del a ngulo formado por las rectas. Demostraci on: Suponga que las rectas se cortan en un punto O; sean a, b las rectas dadas, y P un punto que equidista de ellas; si A y B son las proyecciones de P sobre a y b, respectivamente, entonces P A = P B . Observe que por criterio LLL (utilizando Pit agoras previamente), OAP OBP , por lo que P OA = P OB , i.e., P pertenece a la bisectriz del AOB . Claramente aqu se dan dos casos, recuerde que para denir el angulo entre a y b se utilizan u nicamente semi-rectas, por lo que las rectas a y b denen cuatro a ngulos, que por parejas pueden ser opuestos por el v ertice o suplementarios; de estos se escoge cualquiera de ellos como referencia, entonces, si AOB coincide con este o con el opuesto por el v ertice, la recta P O es llamada bisectriz interna, y en caso contrario, bisectriz externa. As , el lugar geom etrico son dos rectas, y su perpendicularidad se basa en los pares de a ngulos que son suplementarios. Finalmente, si a b, el lugar geom etrico es una recta paralela a a y b que se ubica entre ellas a igual distancia de ambas (este es un caso extra no de bisectriz interna, sin embargo, en ocasiones es u til tener esta convenci on en mente; peor a un, la bisectriz externa es una recta ideal llamada recta al innito ). Teorema: Las bisectrices internas de un ABC concurren en un punto, llamado el Incentro del ABC , usualmente denotado por I. La distancia de I a los tres lados del tri angulo es igual a un n umero r, llamado el Inradio del ABC , y de aqu que la circunferencia de centro I y radio r sea tangente a los lados del tri angulo; dicha circunferencia es llamada el Inc rculo del ABC .27 Demostraci on Sea I la intersecci on de las bisectrices internas de A y B (obviamente, I est a en el interior del ABC ); como I pertenece a la bisectriz interna del A, por el teorema anterior dist(I, AB ) = dist(I, AC ), y an alogamente, como I pertenece a la bisectriz interna del B , dist(I, AB ) = dist(I, CB ); entonces dist(I, AC ) = dist(I, CB ), y de nuevo por el teorema anterior y dado que I est a al interior del tri angulo, I pertenece a la bisectriz interna del C . As , las tres bisectrices internas concurren en un punto que equidista de los lados del tri angulo. Es importante notar que las intersecci on de una bisectrices interna con el lado opueto del tri angulo 28 NO siempre coincide con el puntos de tangencia del inc rculo; de hecho, esto ocurre s olamente si el tri angulo es is osceles.
Sean l, m, n, tres rectas perpendiculares a los lados del AB , BC , CA del ABC , respectivamente. Sean L, M , N , puntos arbitrarios sobre l, m, n, respectivamente. Entonces las rectas l, m, n concurren si y s olo si AL2 + BM 2 + CN 2 = N A2 + LB 2 + M C 2 . 27 En algunas ocasiones denotaremos al inc rculo por (ABC ). 28 En la gura, el ABC es llamado tri angulo tangencial del DEF .
26

74

ngulo. 1.8 Puntos y Rectas Notables del Tria

1 Geometr a

Figura 91: Concurrencia de Bisectrices Internas, Incentro e Inc rculo. Corolario: Dado un tri angulo, existe una circunferencia que es tangente interiormente a los tres lados (el inc rculo); adem as, esta circunferencia es u nica.29 1.8.5. Problemas

1. Sea I el incentro del tri angulo ABC . Si el AIB = 130 , determine la medida del ACB y del ACI . 2. Las a reas de los seis tri angulos AGB , AGC , BGA , BGC , CGA , CGB son iguales e 1 area del tri angulo ABC . iguales a un 6 del

Figura 92 3. Los cuatro tri angulos AB C , BC A , CA B , A B C ,30 son congruentes entre si y semejantes al ABC con raz on de semejanza 1 . 2 4. El centroide del ABC coincide con el centroide del tri angulo medial A B C . Adem as, estos dos tri angulos tienen lados correspondientes paralelos (tri angulos homot eticos). 5. En la gura 93, G es el centroide. Si GD = 2 y el a rea sombreada vale 5, calcule AD y el (ABC ).
Existen 3 circunferencias m as que son tangentes a los tres lados del tri angulo, llamados exc rculos ; estas circunferencias se ubican en el exterior del tri angulo. 30 El A B C es llamado el tri angulo medial del ABC .
29

75

ngulo. 1.8 Puntos y Rectas Notables del Tria

1 Geometr a

Figura 93 6. Demostrar que las paralelas a los lados de un cada lado en tres partes iguales. ABC , trazadas por el centroide G dividen

7. ABCD es un paralelogramo de centroide (baricentro) E , M es el punto medio de AD, y F es la intersecci on de AC con BM . Si el a rea de ABCD es 1, calcule el a rea del cuadril atero DEF M . 8. En el ABC , se traza la mediana AM . Demostrar que si BM = AM , entonces el tri angulo es rect angulo en A. 9. La suma de las distancias del centroide a los puntos medios de los lados de un tri angulo es 20. Calcule la suma de las medianas del tri angulo. 10. La mediana tiene longitud menor que la semisuma de los lados adyacentes, es decir AA < c+a a+b b+c , BB < , CC < . 2 2 2 11. Dado el ABC , sean D y E puntos variables sobre los lados AB y AC respectivamente tales que BC DE . Entonces, la mediana AA puede denirse como el lugar geom etrico de los 31 puntos P tales que P CD BE . 12. Siempre es posible construir un tri angulo XY Z con las medianas AA , BB , CC de un ABC dado. Adem as, los segmentos que unen el centroide del XY Z con sus v ertices son iguales a la mitad de los lados del ABC . 13. En el ABC , AB = BC y la mediatriz de BC interseca a la mediana BM en L. Si LCB = 25, determine la medida del LAC . 14. Ley del Seno. Dado un ABC , se cumple que senA senB senC 1 = = = a b c 2R 15. Las reexiones de H con respecto a los lados del ABC caen sobre el circunc rculo del mismo, es decir HHa = Ha X y an alogo para los otros lados. 16. Las reexiones de H con respecto a los puntos medios de los lados del tri angulo, caen sobre el circunc rculo del mismo.
31

Si D = A se dene P = A, y cuando D = B entonces P es punto medio de BC .

76

ngulo. 1.8 Puntos y Rectas Notables del Tria

1 Geometr a

Figura 94 17. Si O y H son el circuncentro y el ortocentro de un BAH = CAO. 18. La altura AHa es bisectriz del Hb Ha Hc . 19. Los circunc rculos de ABC , ABH , BCH , CAH tienen igual radio. ABC , respectivamente, entonces

20. La perpendicular trazada desde A al lado Hb Hc del tri angulo o rtico, pasa por el circuncentro del ABC . 21. A, B , C y H forman un cuadril atero ortoc entrico, es decir que cada punto es el ortocentro del tri angulo formado por los otros tres. 22. El ortocentro de un tri angulo est a al interior, sobre un v ertice, o afuera del tri angulo, si el tri angulo es acut angulo, rect angulo, u obtus angulo, respectivamente. 23. El circuncentro del ABC es el ortocentro del tri angulo medial ABC .

24. Sea O el circuncentro del los angulos del ABC .

ABC . Si AOC = 100 y OCB = 30, determine la medida de

25. Hallar los angulos de un tri angulo cuyo tri angulo o rtico tiene a ngulos de 20, 50 y 110. 26. Sea ABC un tri angulo obtus angulo de circuncentro O y altura AD. Si OAB = 25 y OCB = 15, calcule el DAB . 27. El ABC de circuncentro O y altura BD. Si DAB = 35 y OBD = 10 encontrar los a ngulos del tri angulo ABC .

77

ngulo. 1.8 Puntos y Rectas Notables del Tria

1 Geometr a

Figura 95 28. En la gura 95, AB es di ametro de la circunferencia. Si X es la intersecci on de CG con AB , calcular el CXB . 29. En el ABC , se trazan la altura AH y la mediana BM . Demuestre que el is osceles. 30. Un ABC es rect angulo en C , A = 75 y CH es altura. Demuestre que CH = M HC es AB . 4

31. Sea O el circuncentro del la medida del ODC .

ABC con C = 45 y sea D el pie de la altura desde A. Calcule

32. Dado el ABC is osceles con A = 90, sean P y Q son puntos dentro del tri angulo tales que BP = AQ y AP = CQ. Si BP y CQ se cortan en R, demostrar que AR P Q. 33. Se ubican los puntos M y K sobre los lados BC y CD del cuadrado ABCD, respectivamente, de modo que M C = KD. Sea P la intersecci on de M D y BK , demuestre que AP M K . 34. Sean D, E , F los puntos de tangencia del inc rculo sobre los lados BC , CA, AB del ABC . Demuestre que se cumplen las siguientes relaciones, donde s denota el semiper metro del tri angulo: AE = AF = s a BD = BF = s b CD = CE = s c 35. El ortocentro del ABC es el incentro de su tri angulo o rtico.

36. Dado un ABC , su tri angulo o rtico y su tri angulo tangencial tienen lados correspondientes paralelos (tri angulos homot eticos). 37. Las bisectrices exteriores de B y C , junto con la bisectriz interior de A, concurren en un punto, llamado el Excentro con respecto al v ertice A (V ease gura 96), usualmente denotado por Ia . Este punto es equidistante a los lados del ABC , dicha distancia es el Exradio respecto a A, usualmente denotado por ra . As , la circunferencia de centro Ia y radio ra es tangente exteriormente a los lados del ABC , y es llamada el Exc rculo respecto a A.32
32

An alogamente se denene los exc rculos con respecto a los otros v ertices.

78

ngulo. 1.8 Puntos y Rectas Notables del Tria

1 Geometr a

Figura 96 38. I es ortocentro del Ia Ib Ic (V ease gura 96). Adem as se cumple: AX = AZ = s BX = BY = s c CY = CZ = s b 39. En un ABC , la bisectriz exterior del ABC y la bisectriz exterior del BCA se cortan en D. La paralela a BC por D corta a AC en L y a AB en M . Si LC = 5 y M B = 7, hallar LM . 40. El ABC es rect angulo en A. Si I es el incentro, calcular BIC .

41. En un ABC , el ABC CAB = 90. Sean D y E los pies de las bisectrices interior y exterior del BCA respectivamente. Demuestre que CD = CE . 42. En el ABC , AB < AC , AD es bisectriz, y E es un punto en AB tal que el EDB = 90. El punto F sobre AC es tal que el BED = DEF . Demuestre que el BAD = F DC . 43. En el ABC se trazan las bisectrices interiores BD y CE tales que D es el punto sobre AC , E es el punto sobre AB , 2BDE = 3B y CED = 2B . Calcular los a ngulos del ABC . 44. Dado el ABC con A = 90, sea D el pie de la perpendicular desde A. Sean adem as I y J los incentros respectivos de ABD y ACD. Demostrar que la bisectriz del BAC es perpendicular a IJ . 45. Un tri angulo es is osceles si cumple alguna de las siguientes condiciones: a) Dos medianas son iguales. 79

ngulo. 1.8 Puntos y Rectas Notables del Tria b) Dos alturas son iguales. c) Dos bisectrices son iguales.33

1 Geometr a

46. Teorema de la Bisectriz: Dado el ABC , sean P y P sobre BC . Se cumple que AP y AP son la bisectriz interna y la bisectriz externa del A si y s olo si BP BA BP = = PC AC PC Sugerencia: Para demostrar la primera igualdad, trace CD de AB . AP con D sobre la prolongaci on

47. (*) De acuerdo con los datos de la gr aca 97, calcular el valor de AB .

Figura 97 48. Dos circunferencias son tangentes internamente en P , y una cuerda AB de la circunferencia de radio mayor es tangente en Q a la otra circunferencia. Ver gura 98. a) Demuestre que P Q es bisectriz del AP B . b) Llame A y B a las otras intersecciones de P A y P B con la circunferencia de radio menor y suponga que AB = 15, P A = 3 y P B = 2; calcule AQ y BQ. 49. (*) Teorema de Poncelet: Demuestre si entonces 2(r + R) = b + c. ABC es un tri angulo rect angulo con A = 90,

50. Demuestre que las mediatrices de un cuadril atero son concurrentes si y s olo si es c clico. 51. Las bisectrices BP y CQ del P QI es is osceles. ABC se cortan en I . Demuestre que si BAC = 60 entonces

52. Demuestre que el cuadril atero convexo ABCD es inscribible si y s olo si los inc rculos respectivos del ABC y CDA son tangentes. 53. Demuestre que las bisectrices internas de un cuadril atero son concurrentes si y s olo si es inscribible.
Este caso es aparentemente tan sencillo como los anteriores, pero realmente es un resultado muy complicado y recibe el nombre de Teorema de Steiner-Lehmus.
33

80

ngulo. 1.8 Puntos y Rectas Notables del Tria

1 Geometr a

Figura 98 54. Demuestre que todo rombo es inscribible. 55. (*) Sea ABCD un paralelogramo. Q es el punto medio de AD, F el pie de la perpendicular por B sobre QC . Probar que AF = AB . 56. Dado el rombo ABCD, se trazan las bisectrices internas de DAC , CAB , BCA, ACD, y cortan a DC , CB , BA, AD en P , Q, R, S , respectivamente. Demuestre que P QRS es un rect angulo. 57. (*) Sea ABCD un cuadril atero tal que AB = CD. Las mediatrices de AC y BD se cortan en P . Probar que P AC = P CA = P BD = P DB . 58. (*) ABC es un tri angulo y P un punto en su interior. Sean A , B y C las reexiones de P sobre BC , CA y AB , respectivamente. D, E y F son los pies de las perpendiculares respectivos desde A, B y C hacia B C , C A y A B . Probar que AD, BE y CF son concurrentes. 59. (*) (Arnoldo Aguilar) En la gura 99, ABGH , BCF G y CDEF son cuadrados. Si I es el centro de ABGH y J = DH BG, demuestre que I , J y F est an alineados.

Figura 99 60. (*) (Arnoldo Aguilar) Sea ABC un tri angulo equil atero. M y N son los puntos medios de AB y BC , respectivamente. Exteriormente al ABC se construye un tri angulo rect angulo is osceles AP C , con AP C = 90 . Si I es la intersecci on de AN y M P , demuestre que CI es la bisectriz de ACM .

81

ngulo. 1.8 Puntos y Rectas Notables del Tria

1 Geometr a

61. (*) En la gura adjunta, el ABC es tal que A = 90 y B = 60. Cu al es el radio de la circunferencia, si CB = 2?

62. (*) Dado el paralelogramo ABCD, sea M el punto medio de AB , y N la intersecci on de CD con la bisectriz interna del ABC . Demuestre que M C BN si y s olo si AN es bisectriz del DAB . 63. (*) En el ABC , se sabe que los v ertices B , C , el circuncentro O y el ortocentro H del ABC est an todos sobre una misma circunferencia. a) Calcule el valor del A. b) Demuestre que el incentro tambi en pertenece al circunc rculo de BCOH . 64. (*) Sea ABC un tri angulo de ortocentro H . Sean P y Q los pies de las perpendiculares desde H a las bisectrices interior y exterior de A, respectivamente. Si M es el punto medio de BC , mostrar que P , Q y M est an alineados. 65. (*) En un tri angulo ABC , sea M el punto medio de BC . Si se cumple que AB = AC y adem as M AC + ABC = 90 , hallar BAC . 66. (*) Sea ABC un tri angulo y U un punto de su circunc rculo tal que AU es bisectriz. Las mediatrices en AB y AC cortan a AU en X y Y . Sea T la intersecci on de BX con CY . Demostrar que AU = T B + T C . 67. (*) (The 59th Romanian Mathematical Olympiad Final Round) Sea ABCD un rect angulo de centro O con AB = BC . La perpendicular en O a BD corta a las l neas AB y BC en los puntos E y F , respectivamente. Sean M y N los puntos medios de los segmentos CD y DA, respectivamente. Probar que las l neas rectas F M EN . 68. (*) Sea ABC un tri angulo rect angulo, con A = 90 . Sea D un punto en su interior tal que DAC = DCA = DBC = , y AC = BD. Determine el valor de . 69. (*) Sea ABC un tri angulo y M un punto tal que M AB = 10, M BA = 20, M AC = 40 y M CA = 30. Probar que el ABC es is osceles. 70. (*) En la gura 100, ABCD y P QRS son cuadrados, ABP BCQ CDR DAS y los los radios de las cinco circunferencias son iguales a r. Si a es el lado del cuadrado ABCD, determine r en funci on de a. 71. (*) Recta de Euler. El centroide G, el ortocentro H y el circuncentro O de un tri angulo est an alineados, y adem as GH = 2GO. 82

ngulo. 1.8 Puntos y Rectas Notables del Tria

1 Geometr a

Figura 100 72. Circunferencia de los 9 puntos:34 Dado un ABC de ortocentro H , se cumple que los puntos medios de los lados, los pies de las alturas, y los puntos medios de HA, HB , HC , se ubican sobre una misma circunferencia. Adem as, el centro de esta circunferencia es el punto R angulo. medio de HO y su radio es 2 , donde O y R son el circuncentro y el circunradio del tri Para demostrar este resultado se sugiere seguir los siguientes pasos: a) Si Ha es el pie de la altura trazada desde A, demuestre que la reexi on de H con respecto a Ha pertenece a circunc rculo del ABC . Resultados similares se cumplen para Hb y Hc . b) Si A es el punto medio de BC , demuestre que la reexi on de H con respecto a A pertenece al circunc rculo del ABC . c) De los resultados anteriores, observe que hay 9 puntos sobre el circunc rculo del ABC : los v ertices, las reexiones de H con respecto a los pies de las alturas, y las reexiones de H con respecto a los puntos medios de los lados; a partir de esto, concluya que los puntos medios de los segmentos que van de H a estos 9 puntos, tambi en deben pertenecer en una misma circunferencia. d) Concluya adem as que el centro de esta nueva circunferencia es el punto medio de HO. Otro camino de soluci on es el siguiente: a) Sea A B C el tri angulo medial del ABC . Pruebe que A B C = BHa C y concluya que Ha A B C es un cuadril atero c clico; los mismo debe cumplirse para Hb y Hc . b) Sea X el punto medio de HA. Demuestre que B A C + B XC = 180 y concluya que XC B A es un cuadril atero c clico; lo mismo debe cumplirse para los puntos medios de HB y HC . c) De lo anterior, concluya que los pies de las alturas y los puntos medios de los segmentos que van desde H hasta los v ertices del ABC , se ubican sobre el circunc rculo del A B C . d) Si N es el circuncentro del A B C , demuestre que N , O, G forman la recta de Euler del A B C y utilice sus propiedades para probar que N es el punto medio de HO.
34

Tambi en conocida como Circunferencia de Feuerbach.

83

1.9 Problemas resueltos. 73. El a rea del ABC , denotada por [ABC ], cumple: base altura 2 ab senC bc senA ca senB abc [ABC ] = = = = 2 2 2 4R [ABC ] = sr [ABC ] = [ABC ] = s(s a)(s b)(s c). (F ormula de Her on).

1 Geometr a

74. El circunradio, el inradio y los exradios de un tri angulo cumplen: 4R = ra + rb + rc r [ABC ] = ra (s a) = rb (s b) = rc (s c) r = ra = rb = rc = 75. Dado el (s a)(s b)(s c) s s(s b)(s c) sa s(s a)(s c) sb s(s a)(s b) ( s c)

ABC , sea I el incentro e Ia el excentro respecto a A.

a) Demuestre que BICIa es un cuadril atero c clico. b) Si M es la intersecci on de IIa con el circunc rculo del tri angulo (M = A), demuestre que dicho punto es el circuncentro de BICIa . c) Sea M el punto diametralmente opuesto a M en el circunc rculo, y sea P la proyecci on de I sobre AB . Demuestre que M CM AIP . d) Sea O el circuncentro del tri angulo Calcule la potencia de punto de I con respecto al circunc rculo, y utilizando los resultados anteriores deduzca la F ormula de Euler : OI 2 = R2 2Rr. e) A partir de la f ormula de Euler demuestre que R 2r.

1.9.
1.9.1.

Problemas resueltos.
Tri angulos

1. En la gura 101, ABDE es un cuadrado y BCD es un tri angulo is osceles con BD = DC . Si ABC = 160, determinar la medida de AEC . Soluci on: DBC = DCB = 160 ABD = 70, de donde se obtiene que BDC = 40 y EDC = EDB + BDC = 130. Como EDC es is osceles, entonces DEC = DCE = 25. Por lo tanto AEC = 90 DEC = 65. 84

1.9 Problemas resueltos.

1 Geometr a

Figura 101

Figura 102 2. Hallar la suma de los angulos + + + en la gura 102. Soluci on: CAB = , EDC = por ser opuestos por el v ertice. Como el AF D externo en el BDF , se tiene AF D = + . Sumando los angulos internos del AEF se tiene + + + = 180. 3. (XV Competencia de Clubes Cabri Primera Ronda) En la gura 103, ABCD es un rect angulo tal que AB = 2BC . M es el punto medio de AB y los tri angulos AM E y M BF son equil ateros. Si P es la intersecci on de las rectas DE y CF , encuentre los a ngulos del CDP . Soluci on: Note que AD = AE = F B = BC por lo que DAE y BCF son ambos is osceles. Luego DAE = CBF = 90 + 60 = 150 lo que implica que P DC = P CD = 90 15 = 75 y luego CP D = 30. 4. Sea ABC un tri angulo rect angulo con CAB = 90 (Ver gura 104). D es un punto sobre la prolongaci on de BC tal que BD = BA. E es un punto en el mismo semiplano que A respecto de BC , tal que CE BC y adem as CE = CA. Mostrar que A, D y E est an alineados. Soluci on: Sea CBA = 2; el ABD es is osceles y BAD + BDA = 2, por lo que BAD = BDA = . Como CAB = 90 entonces ACB + ABC = 90 y como CE es perpendicular a BC entonces ECA + ACB = 90; por lo tanto, ABC = ECA = 2. Con esto, como ECA es is osceles, CEA = CAE = 90 . Luego, EAC + CAB + BAD = 180 y as E , A y D est an alineados. 85

1.9 Problemas resueltos.

1 Geometr a

Figura 103

Figura 104 5. Dado un cuadrado ABCD, se construyen los tri angulos equil ateros ABP (exteriormente) y ADQ (interiormente). Probar que C , P y Q est an alineados. Figura 105. Soluci on: Observe que P AQ = BAD = 90 y P A = BA = DA = DQ, por lo que P AQ es tri angulo rect angulo is osceles, y por tanto, P QA = 45. Por otra parte, QDC = 90 ADQ = 30 y QD = AD = CD, es decir, el CDQ es is osceles con el a ngulo comprendido entre lados iguales de 30, por lo que DQC = 75. As , P QA + AQD + DQC = 180 y por lo tanto, C , P y Q est an alineados. 6. En la gura 106, AB = BC = CD = DE = EF = F G = GA. Calcule la medida del DAE . Referenciasg20 Soluci on: Sea DAE = . Como los tri angulos ABC y AGF son is osceles, ACB = AF G = . Calculando los a ngulos externos de ABC y AF G se tiene F BC = CGF = 2. Como GF E y BCD son is osceles, GEF = BDC = 2. Calculando angulos externos de ADC y AEF se obtiene ECD = DF E = 3. Como CDE y F ED son is osceles, CED = F DE = 3. Entonces, la suma de los a ngulos internos del AED da + 3 + 3 = 180, de donde = 180 . 7

86

1.9 Problemas resueltos.

1 Geometr a

Figura 105

Figura 106 7. (XXVIII Olimpiada Brasile na de Matem atica) En la gura 107, AB = AC , AM = AN y CAM = 30, encuentre el valor del BM N .

Figura 107 Soluci on: Como ABC y AM N son is osceles, sean ABC = ACB = y AM N = AN M . Por la f ormula del a ngulo externo se tiene ACM + M AC = AM B = AM N + BM N + 30 = AN M + BM N = (N BM + BM N ) + BM N = + 2BM N Esto implica que BM N = 15. 8. (Etapa seminal Estatal de XXII Olimpiada Mexicana de Matem aticas) En la gura 108 se muestra un hex agono regular ABCDEF de lado 1. Los arcos del c rculo que est an dibujados 87

1.9 Problemas resueltos.

1 Geometr a

tienen centro en cada v ertice del hex agono y radio igual a la distancia al v ertice opuesto. P , Q, R, S , T y U son los puntos de corte de estos arcos. Cu anto mide cada lado del hex agono P QRST U ?

Figura 108 Soluci on: El hex agono P QRST U es regular y con el mismo centro que ABCDEF . Sea O el centro de ambos (V ease Figura 72). El lado buscado es igual a OP . Tenemos que CF = F P = P C = 2 por ser radios de los arcos dibujados; entonces CF Pes equil atero de lado 2 y OP es una altura de este tri angulo que, por Pit agoras, es igual a 3. 1.9.2. Congruencia de Tri angulos

1. En la gura 109, ABC es un tri angulo equil atero y CDEF es un cuadrado. Se construye un punto G tal que CF = CG y adem as CF G = 15. Probar que AGC = BDC .

Figura 109 Soluci on: BCD = 180 ACB DCF = 30. Como GCF es is osceles, CGF = CF G = 15 y ACG = CGF + CF G = 30. Por criterio LAL, BCD ACG, por lo tanto BDC = AGC .

88

1.9 Problemas resueltos.

1 Geometr a

2. (Cuaderno de Olimpiadas Mexicanas - Geometr a) En la gura 110, ABCD un cuadrado y EF GH . Demuestre que que EF = GH .

Figura 110 Soluci on: Se construyen EK y GM con K sobre CD y M sobre AD tales que EK GM CD. Luego se demuestra que EF K GHM , con EF = GH . AD y

3. (Examen nal de XVI Olimpiada mexicana de Matem atica) Los angulos de un tri angulo ABC est an en progresi on aritm etica (B A = C B = ), D, E , y F son los puntos medios de los lados BC , CA y AB , respectivamente. Llamamos H al pie de la altura trazada desde C (que cae entre B y F ) y G a la intersecci on entre DH y EF . Cu anto vale F GH ? Soluci on: Note que A+B +C = 3A+3 = 180, lo cual implica que A+ = 60 = B . Entonces BCH es un tri angulo 30, 60, 90, y dado que D es punto medio de BC , el BDH es equil atero. Luego, como BC EG, F GH = BDH = 60. Ver gura 111.

Figura 111 4. Sea ABCD un cuadrado. Se construyen tri angulos equil ateros ADP y ABQ como se muestra en la gura 112. Sea M la intersecci on de CQ con AD y N la intersecci on de CP con AB . Demuestre que CM N es un tri angulo equil atero. Soluci on: Note que P D = AD, porque AP D es equil atero, y AD = CD porque ABCD es cuadrado, por lo que P D = CD, es decir, el CDP es is osceles, con CDP = CDA P DA = 30, entonces DP C = DCP = 75, y BCN = BCD DCP = 15. 89

1.9 Problemas resueltos.

1 Geometr a

Figura 112 An alogamente, BCQ es is osceles con angulos 30, 75, 75, por lo que M CN = BCQ BCN = 60. Finalmente, como la gura es sim etrica con respecto a AC , CM = CN , entonces, el tri angulo CM N es equil atero porque tiene dos lados iguales y un angulo interno igual a 60. 5. ABC es un tri angulo is osceles con ABC = ACB = 80. D es un punto en AC tal que ABD = 10. Demuestre que AD = BC . Figura 113. Soluci on: Se traza un punto D sobre AC tal que AD = BC . Se construye exteriormente el tri angulo equil atero AEB . Luego, AE = AB , D A = CB y EAD = ABC lo cual implica que EAD ABC , de donde se deduce que el D EB es is osceles y BED = BEA D EA = 40. Se sigue que EBD = 70 y como D BA = EBD ABE = 10, resulta que D = D y por lo tanto BC = AD = AD.

Figura 113

90

1.9 Problemas resueltos. 1.9.3. Cuadril ateros

1 Geometr a

1. Sea ABCD un paralelogramo. Se construyen tri angulos equil ateros exteriores CDP y ADQ, como se muestra en la gura 114. Demuestre que el BP Q es equil atero.

Figura 114 Soluci on: Observe que al hacer una rotaci on de centro P y a ngulo 60, el tri angulo P BC se transforma en el tri angulo P QD (observe los segmentos P C y CB tras esta transformaci on), mientras que al hacer una rotaci on de centro Q y a ngulo 60, el tri angulo P QD se transforma en tri angulo BQA. Como la rotaci on mantiene las distancias, P B = P Q = BQ, por lo que 35 el tr angulo BP Q es equil atero. 2. (II Olimpiada Matem atica del Cono Sur) En la gura 115 ABCD y AECF son paralelogramos. Demuestre que BEDF es paralelogramo.

Figura 115
Una demostraci on m as rigurosa se basa en el c alculo de los angulos P CB = P DQ = BAQ = 120 + ABC y en la utilizaci on del criterio LAL para justicar P CB P DQ BAQ.
35

91

1.9 Problemas resueltos.

1 Geometr a

Soluci on 1: Sea M el punto medio de AC . Las diagonales AC y BD se bisecan en M , mientras que las diagonales AC y EF tambi en se bisecan en M , entonces BD y EF se bisecan en M por lo que BEDF es un paralelogramo. Soluci on 2: Como AD CB y AE CF entonces DAE = BCF . Entonces, por propiedades de paralelogramos BAE = BAD EAD = BCD BCF = F CD; adem as, AB = CD y AE = CF . Por criterio LAL, BAE DCF , y entonces BE = DF . An alogamente se demuestre que ABF CDE , lo cual implica BF = DE . Como BEDF es un cuadril atero con lados opuestos iguales, es un paralelogramo. 3. ABCD es un cuadril atero convexo y O es un punto en su interior. Sean P , Q, R, S , los puntos medios de los lados AB , BC , CD, DA, respectivamente. Por P se traza una paralela a OR, por Q se traza una paralela a OS , por R se traza una paralela a OP , y por S se traza una paralela a OQ. Demuestre que estas cuatro rectas concurren. Soluci on: Al tomar las rectas OP y OR y sus paralelas se forma el paralelogramo P ORM , y al tomar las rectas OQ y OS y sus paralelas se forma el paralelogramo OQN S . Por el teorema de Varignon, sabemos que P QRS es un paralelogramo, y llamaremos T al punto de corte de sus diagonales. Observe que el punto de corte de las diagonales de P ORM es el punto medio de P R, i.e., T ; an alogamente, el punto de corte de las digonales de OQN S es el punto medio de SQ, i.e., T nuevamente. As , M es la reexi on de O con respecto a T , y de igual forma queda denido N , por lo que M = N y las cuatro rectas concurren. 4. (H ector Alberti) Sea ABCD un cuadrado. Se construyen los tri angulos equil ateros BDA , ACB , BDC y ACD (V ease gura 116). Demuestre que el A B C D es tambi en un cuadrado.

Figura 116 Soluci on: Como A B = A D, AB = AD, CB = CD, C B = C D, los puntos A , A, C , C pertenecen a la mediatriz de BD, y por tanto, est an alineados. An alogamente, B , B , D, D est an alineados; por lo tanto A C B D . Por otra parte, si O es el centro de ABCD, como los tri angulos equil ateros construidos son todos iguales (tienen lados iguales a la diagonal de ABCD) de altura h, OA = OB = OC = OD = h. Entonces, A B C D es un cuadril atero con diagones que se bisecan en O (es paralelogramo), son iguales A C = B D = 2h (es rect angulo) y son perpendiculares (es rombo), lo cual implica que es cuadrado.

92

1.9 Problemas resueltos.

1 Geometr a

5. Un trapecio is osceles tiene diagonales perpendiculares y su area es 2010, determine su altura. Soluci on: Considere la gura 117. Sea ABCD el trapecio del problema (AB = CD), como es trapecio is osceles, es sim etrico con respecto a la mediatriz de las bases, en particular, AC = BD. Sean P y Q los pies de las perpendiculares a AD trazadas desde B y C , respectivamente. Por LAL, ABD DCA lo cual implica CAD = BDA = 45 (debido a que AC BD) luego en el tri angulo rect angulo ACQ, ACQ = 45 por lo que AQ = CQ y es f acil ver que BC = P Q. Luego 2010 = (ABCD) = (CQ)(BC + AD) 2 (CQ)(P Q + AP + P Q + QD) = 2 = (CQ)(AP + P Q) = CQ2

Luego CQ =

2010.

Figura 117 6. (IX Competencia de Clubes Cabri, Segunda Ronda) Sea ABCDEF un hex agono regular cuyo centro es O. Se construyen los cuadrados F SOP y ORCQ. Demuestre que AP QB y SEDR son rect angulos. Figura 118. Soluci on: Por construcci on P F = P O = SF = SO, y por propiedades de hex agono regular36 AF = AO = EF = EO, entonces P , S A, E , pertenecen a la mediatriz de F O y por tanto, est an alineados sobre una recta perpendicular a F O. An alogamente, Q, R, B , D est an alineados sobre una recta perpendicular a CO; adem as, es f acil demostrar que AP = BQ = DR = ES . Observe adem as que AB CF DE , lo cual implica (AB CF ) (AP BQ), es decir, AP QB es rect angulo, y an alogamente para SEDR. 7. Sobre los lados del ABC se trazan exteriormente los cuadrados ABP Q, CARS y BCT U . Luego se trazan los paralelogramos AQA R, CSC T y BU B P , como en la gura 119.
36

Los tri angulos OAB , OBC , OCD, ODE , OEF , OF A son equil ateros.

93

1.9 Problemas resueltos.

1 Geometr a

Figura 118 a ) Sean A , B , C los centros de los cuadrados BCT U , CARS , ABP Q, respectivamente. Demuestre que estos centros est an sobre los lados del A B C . b ) Demuestre que AA , BB , CC concurren. Soluci on: a) Observe que al hacer una rotaci on de centro A y angulo igual a 90, el A U B se transforma en el A BA, y a la vez este u ltimo se transforma en el A CC (esto es porque A U A B A C y U B BA CC ); esto signica que A B A A y A A A C , por lo que B , A , C est an alineados, es decir, A pertenece a B C . An alogamente se prueban los otros casos. b) De lo anterior, observe que AA es mediatriz de B C , por lo que AA , BB , CC concurren en el circuncentro del A B C . 8. Se dibujan cuadrados exteriores a los lados de un paralelogramo (Vea gura 120), demuestre que: a ) El cuadril atero determinado por los centros de esos cuadrados es un cuadrado. b ) Las diagonales de ese cuadrado son concurrentes con las del paralelogramo. Soluci on: a ) Observe que al hacer una rotaci on de centro O2 y a ngulo igual a 90, el O2 BO1 se transforma en el O2 CO3 (observe que los segmentos O2 B y BO1 se transforman en O2 C y CO3 , respectivamente), por lo que O2 O1 = O2 O3 y O2 O1 O2 O3 . Repitiendo este razonamiento, O1 O2 = O2 O3 = O3 O4 = O4 O1 y estos segmentos son perpendiculares si son consecutivos, por lo que O1 O2 O3 O4 es un cuadrado. 94

1.9 Problemas resueltos.

1 Geometr a

Figura 119 b ) Basta demostrar que AC y O1 O3 se bisecan,37 y esto es equivalente a demostrar que AO1 CO3 es un paralelogramo. Esto es cierto porque AO1 = CO3 y AO1 CO3 (ambos segmentos son perpendiculares a O1 B )

Figura 120 9. Dado un ABC , se construyen exteriormente los tri angulos rect angulo is osceles ACP y BCQ, con AC y BC como hipotenusas. Si M es el punto medio de AB , demuestre que el M P Q tambi en es un tri angulo rect angulo is osceles. Soluci on: Construya los cuadrados exteriores ACDE y BCF G, como muestra la gura 121. Observe que P y Q son los puntos medios de AD y BF , respectivamente. Al rotar el BCD
37

Porque as los puntos de corte de las diagonales de ABCD y O1 O2 O3 O4 coincidir an.

95

1.9 Problemas resueltos.

1 Geometr a

con centro C y a ngulo de 90, se genera el F CA, entonces dichos tri angulos son congruentes y en por tanto BD = AF y BD AF . Por otra parte, observe que M P es base media del BAD, por lo que 2M P = BD y M P BD; an alogamente, M Q es base media del ABF , por lo que 2M Q = AF y M Q AF . Por lo tanto M P = M Q y M P M Q.

Figura 121 1.9.4. La Circunferencia AB.

1. Dada la gura 122, demuestre que AB

Figura 122 Soluci on: Observe que los cuadril ateros ABQP y A B QP son c clicos, por lo que P AB = P QB = 180P A B , por lo tanto AB A B . 2. Dos circunferencias de centros O1 y O2 son tangentes (interna o externamente) en un punto P ; por este punto se traza una recta que corta nuevamente a la circunferencias en A y B , respectivamente. Demuestre que AO1 BO2 . Soluci on: En la gura 123 se ha considerado que las circunferencias son tangentes exteriormente, sin embargo, el otro caso es an alogo. Se sabe que O1 , P , O2 est an alineados, y que 96

1.9 Problemas resueltos.

1 Geometr a

AP O1 y BP O2 son tri angulos is osceles (dos de sus lados son radios de una circunferencia), entonces O1 AP = AP O1 = BP O2 = O2 BP , por lo que AO1 BO2 .

Figura 123 3. Dadas dos circunferencias una fuera de la otra como en la gura 124, demuestre que las tangentes comunes externas forman segmentos iguales; an alogamente, las tangentes comunes internas forman segmentos iguales. Soluci on:38 Sea P la intersecci on de las tangentes comunes externas AA y BB . Entonces AA = P A P A = P B P B = BB . An alogamente se resuelve el caso de las tangentes comunes internas.

Figura 124 4. Teorema de Pithot. Demuestre que en todo cuadril atero inscribible, la suma de lados opuestos es igual.39 Soluci on: Considere la gura 125, ABCD es el cuadril atero inscribible, con P , Q, R, S , los puntos de tangencia sobre AB , BC , CD, DA, respectivamente. Entonces AB + CD = AP + P B + CR + RD = AS + BQ + CQ + DS = BC + DA
Suponemos que las circunferencias tienen radios distintos; cuando los radios son iguales, el problema se justica por la simetr a de la gura. 39 El rec proco de este teorema y del siguiente son tambi en es ciertos.
38

97

1.9 Problemas resueltos.

1 Geometr a

Figura 125: Teorema de Pithot. 5. Teorema de Steiner. En todo cuadril atero exinscrito a una circunferencia, la diferencia de las longitudes de lados opuestos es igual. Soluci on: El cuadril atero puede quedar en posiciones como las de la gura ??; en ambos casos, la demostraci on es muy similar, y an aloga a la de Pithot. Para la gura de la izquierda se tiene que AB CD = (AP BP ) (CR RD) = (AS BQ) (CQ DS ) = AD BC

Figura 126: Teorema de Steiner. 6. Teorema de Miquel: Dado un ABC , sean X , Y , Z puntos sobre AB , BC , CA, respectivamente . Demuestre que los circunc rculos de AXZ , BY X , CZY tienen un punto en com un M . Soluci on: Sea M el otro punto de corte de los circunc rculos de los cuadril ateros AXM Z y BY M X son c clicos, se tiene AXZ y BY X . Como

Y M Z = 360 XM Z Y M X = 360 (180 A) (180 B ) = 180 C Entonces, CY M Z es cuadril atero c clico, por lo que M est a sobre el circunc rculo del 98 CZY .

1.9 Problemas resueltos.

1 Geometr a

Figura 127: Teorema de Miquel. 7. Sea ABC un tri angulo, y sean L y N las intersecciones de la bisectriz del a ngulo A con el lado BC y el circunc rculo de ABC respectivamente (Ver gura 128). Construimos la intersecci on M del circunc rculo de ABL con el segmento AC . Prueba que los tri angulos BM N y BM C tienen la misma a rea. Soluci on: Observe que ABN C y ABLM son cuadril ateros c clicos, por lo que N CB = N AB = LAM = LBM , por lo que CN BM . Entonces, las distancias de N y C a la recta BM son iguales, y por tanto, el a rea del BM N es igual al a rea del BM C .

Figura 128 8. Sea AB el di ametro de una semicircunferencia. Se colocan los puntos M y K sobre la semicircunferencia y sobre AB , respectivamente.40 Sea P el centro de la circunferencia que pasa por A, K y M ; sea Q el centro de la circunferencia que pasa por B , K y M . Demuestre que M P KQ es conc clico.
40

M y K son distintos de A y B .

99

1.9 Problemas resueltos.

1 Geometr a

Soluci on: Como AB es di ametro, AM B = 90, entonces M AB + M BA = 90. As M P K + M QK = 2M AK + 2M BK 2 = 2 (M AB + M BA) = 180

Por lo tanto, M P KQ es conc clico.

9. Las circunferencias 1 y 2 se cortan en los puntos A y B . Por el punto A se traza una recta que corta nuevamente a las circunferencias 1 y 2 en los puntos C y D, respectivamente. Por los puntos C y D se trazan tangentes a las circunferencias, las cuales se cortan en el punto M . Demuestra que M CBD es c clico. Figura 129. Soluci on: Es suciente probar que M CBD es un cuadril atero con un par de angulos opuestos suplementarios. Por a ngulos seminscritos y suma de angulos internos de un tri angulo, se tiene CM D + CBD = CM D + CBA + DBA = CM D + M CA + M DA = 180

Figura 129 10. El ABC cumple que A = 90 y AB = AC . Se toma un punto E del segmento AB , se construye interiormente un tri angulo equil atero AEF . EF corta BC en I , y se construye exteriormente un tri angulo equil atero BIJ . Encuentre EJB . 100

1.9 Problemas resueltos.

1 Geometr a

Soluci on: Como el BJI = 60 = AEI , el cuadril atero BEIJ es c clico, por lo que el EJB = EIB = AEI EBI = 15.

11. En la gura 130, se sabe que AO1 B AO2 B = 70 y adem as la tangente EB forma el tri angulo is osceles ABE , con AB = AE . Encuentre EBC .

Figura 130 Soluci on: Sea AO2 B = 2, entonces ACB = y por hip oteis AO1 B = 2 + 70. Por a ngulo seminscrito, ABE = + 35, y como el ABE es is osceles, AEB = + 35. Finalmente, por la f ormula del a ngulo externo aplicada al BCE , EBC = AEB ECB = 35 12. Dos circunferencias 1 y 2 se cortan en A y B . Una recta por A corta a 1 y 2 en C y D, respectivamente, y la paralela a CD por B corta 1 y 2 en E y F , respectivamente. (Ver gura 131). Demuestre que CDB EAF . Soluci on: Sean G = AE BC y H = AF BD. Como AC BE y ACEB es c clico,41 CAG = GEB = ACG = GBE = ; an alogamente, DAH = HF B = ADH =
41

ACEB es un trapecio is osceles.

101

1.9 Problemas resueltos.

1 Geometr a

F BH = . Observe que GAC y GBE son tri angulos is osceles y por tanto AE = AG + GE = CG + GB = CB ; de forma similar se obtiene AF = DB . Finalmente, EAF = 180 = CBD, por lo que, por el criterio LAL, CBD EAF .

Figura 131 13. La Recta de Simson-Wallace. Sean X , Y y Z los pies de las alturas trazadas desde un punto P en el circunc rculo del ABC hacia AB , BC y CA, respectivamente. Demuestre que X , Y y Z est an alineados.42 Soluci on: Como BP Y X es c clico, Y XP = Y BP = . Como ABP C es c clico, CBP = CAP = . Como AXP Z es c clico, ZAP = ZXP = . Por lo tanto, dado que Y XP = ZXP , los puntos X , Y y Z est an alineados.

Figura 132: Recta de Simson-Wallace


El rec proco tambi en es cierto, si X , Y y Z est an alineados, entonces P debe estar sobre el circunc rculo del ABC ; en cualquier otro caso, el XY Z se llama el tri angulo pedal con respecto al punto P .
42

102

1.9 Problemas resueltos.

1 Geometr a

14. Sea P un punto exterior al cuadrado ABCD tal que AP C = 90 , Q es la intersecci on de AB y P C , y R el pie de la perpendicular por Q a CA. Demuestre que P , R y D est an alineados. Soluci on: Como AP C +ADC = 180, el cuadril atero P ADC es c clico, entonces AP D = ACD = 45. An alogamente, como AP Q + ARQ = 180, el cuadril atero P ARQ es c clico, entonces AP R = AQR = 90QAR = 45. Por lo tanto, como AP D = AP R, los puntos P , R, D est an alineados. Figura 133.

Figura 133 15. (OIM 2002, P-4) En un tri angulo escaleno ABC se traza la bisectriz interior BD, con D sobre AC . Sean E y F puntos sobre la recta BD tales que (AE CF ) BD, y sea M el punto sobre el lado BC tal que DM BC . Demuestre que EM D = DM F . Figura 134. Soluci on: Como DM M C y DF F C , DF CM es c clico, por lo tanto DM F = DCF = , y como AE F C , entonces EAD = DCF = . Sea G la intersecci on de AE con BC . Como AG BE , BE es altura y bisectriz del ABG, por lo que este tri angulo es is osceles y adem as BE es mediatriz de AG; entonces EGD = EAD = . Y nalmente, podemos ver que DEM G es c clico, pues DEG = DM G = 90 , as que EM D = EGD = . De aqu , el resultado es inmediato. 16. (OMCC 2003, P-2) Sea S una circunferencia y AB un di ametro de ella. Sea t la recta tangente a S en B y considere dos puntos C y D en t tales que B este entre C y D. Sean E y F las intersecciones de S con AC y AD y sean G y H las intersecciones de S con CF y DE . Demuestre que AH = AG. Soluci on: Como AEBF es c clico (Ver gura 135), AEF = ABF . Luego, como AB CD y BF AD, se cumple tambi en ABF = F DB , por lo que AEF = F DC , es decir, el cuadril atero CDF E es c clico. Utilizando este resultado y el hecho que EGHF tambi en es c clico, se tiene EDC = EF G = EHG, por lo que CD GH . Esto implica que AB GH , y como AB pasa por el centro de la circunferencia, debe ser mediatriz de GH , por lo tanto AG = AH. 17. (The 59th Romanian Mathematical Olympiad District Round) Considere un cuadrado ABCD y un punto E sobre el lado AB . La diagonal AC corta al segmento DE en el punto P . La 103

1.9 Problemas resueltos.

1 Geometr a

Figura 134

Figura 135 perpendicular por P a DE corta al lado BC en F . Probar que EF = AE + CF . Soluci on: Se construye E sobre BC de tal manera que CE = AE (como se muestra en la gura 136) y que C quede entre F y E , as por LAL se tiene que los tri angulos rect angulos DAE DCE por lo tanto ADE = CDE luego EDE = 90. Por otra parte, el cuadril atero DCF P es c clico, por lo que P DF = P CF = 45 entonces F DE = EDE EDF = 45. Ahora por LAL los tri angulos DEF DE F , por lo que EF = E F = E C + CF = AE + CF . 18. Teorema de Arqu medes: En la gura 137, la regi on delimitada por tres semicircunferen104

1.9 Problemas resueltos.

1 Geometr a

Figura 136 cias mutuamente tangentes, es conocida como cuchilla de zapatero o arbelos. Demostrar que las circunferencias sombreadas son congruentes.

Figura 137: Teorema de Arqu medes. Soluci on: Sean AB , AC , BC los di ametros de las semicircunferencias que forman el a rbelos, de radios r, r1 , r2 y centros O, O1 , O2 , respectivamente. De momento nos concentramos en el lado izquiero de la gura Referenciasgura58; sea C1 el centro de la circunferencia de la izquierda y R1 su radio; D, E y F son los puntos de tangencia de esta circunferencia con dos semicircunferencias del arbelos y con la recta perpendicular a los di ametros por C ; nalmente, G es la proyecci on de C1 sobre AB . En primer lugar, OO1 = OA O1 A = r r1 . Por otra parte, observe que O, C1 , D est an alineados y O1 , C1 , E tambi en est an alineados, entonces OC1 = OD C1 D = r R1 y O1 C1 = O1 E + EC1 = r1 + R1 . Adem as, como CF C1 G es un rect angulo, GC = F C1 = R1 , entonces O1 G = O1 C GC = r1 R1 y OG = O1 G O1 O = (r1 R1 ) (r r1 ) = 2r1 (r + R1 ). Ahora, aplicando el teorema de Pit agoras a GO1 C1 y GOC1 se tiene
2 2 O1 C1 O1 G2 C1 G2 = OC1 OG2 C1 G2 (r1 + R1 )2 (r1 R1 )2 = (r R1 )2 (2r1 (r + R1 ))2 2 4r1 R1 = 4rR1 4r1 + 4r1 r + 4r1 R1 r1 (r r1 ) R1 = r

An alogamente, si r2 y R2 son los radios del semic rculo y del c rculo de la derecha, respectivamente, entonces r2 (r r2 ) R2 = r 105

1.9 Problemas resueltos.

1 Geometr a

Figura 138 Pero 2r = AB = AC + BC = 2r1 + 2r2 , entonces r2 = r r1 , y sustituyendo en la ecuaci on anterior se tiene (r r1 )r1 (r r1 ) (r (r r1 )) = = R1 R2 = r r 1.9.5. Semejanza de Tri angulos

1. (IV OMCC, P-4) Sea ABC un tri angulo, D el punto medio de BC , E un punto sobre el segmento AC tal que BE = 2AD y F el punto de intersecci on de AD con BE . Si CAD = 60, encuentre la medida de los angulos del F EA. Figura 139. Soluci on: Se traza por D una paralela a BE y sea G el punto por el que esta paralela corta al lado AC . Como DG es base media del BCE se tiene que DG = BE = AD; entonces 2 ADG es is osceles y tiene un a gulo de 60, por lo que debe ser equil atero. Finalmente AEF tambi en es equil atero, por tanto sus angulos son iguales a 60.

Figura 139 2. Sea ABCD es un trapecio con AD BC . M y N son los puntos medios de CD y BC , M respectivamente, y P el punto com un de las rectas AM y DN . Si P = 1 , demuestre que AP 4 ABCD es paralelogramo. Soluci on: Sea Q el punto medio de DN , entonces QM BC DA. Como M Q es base media Q M del CDN , M Q = CN = CB . Por otra parte, como P M Q P AD, M = P = 1 , 2 4 AD AP 4 AD entonces M Q = 4 . Finalmente, como BC DA y BC = DA, ABCD es paralelogramo.

106

1.9 Problemas resueltos.

1 Geometr a

3. En la gura 140, BC = CD = DE = EA = x y AEB = 90. Demuestre que ABC + ACD + ADE = 90.

Figura 140 Soluci on 1: Por Pit agoras, AD = 2x. Observe que DA2 = 2x2 = DB DC , por lo que ABD CAD; entonces ABD = CAD y por tanto ABC + ACD + ADE = CAD + ACD + ADE = 2ADE = 90. Soluci on 2: Considere la siguiente cuadr cula (Figura 141). Observe que al hacer una rotaci on de centro A y a ngulo igual a 90, el segmento AC se transforma en AF , por lo que el ACF es tri angulo rect angulo is osceles, y ADE = ACF . Se cumple ABC = CF D, porque se forman con la diagonal de tres cuadrados; an alogamente, ACD = AF D, porque se forman con la diagonal de dos cuadrados. Sumando los angulos internos del ACF se obtiene el resultado buscado.

Figura 141 4. (Asi atico Pac ca) Sea ABC un tri angulo y D el pie de la altura con respecto a A. Sean E y F puntos en una recta que pasa por D (distintos de D) tales que AE CE y AF BF .

107

1.9 Problemas resueltos.

1 Geometr a

Sean M y N los puntos medios de BC y EF , respectivamente. Demuestre que AN N M . Soluci on: En la gura 142, AE CE y AD DC entonces, ADEC es c clico, as que DEA = DCA. Del mismo modo, como AF BF y AD DB , AF BD es c clico y entonces AF D = ABD. Esto implica que ABC AF E y a partir de esta semejanza, ABM AF N . Luego, AM B = AN F , por lo que el cuadril atero AN DM es c clico, y por lo tanto AN M = ADM = 90.

Figura 142

1.9.6.

Puntos y rectas notables del Tri angulo

1. De acuerdo con los datos de la gr aca 143, calcular el valor de AB .

Figura 143
AC CD 5 Soluci on 1: Por el teorema de la bisectr z AB = DB , de donde AC = 4 x, luego, aplicando 2 5 2 2 el teorema de pit agoras al ABC , se tiene que x + 18 = 4 x , que despu es de resolver se tiene que x = 24.

Soluci on 2: Dado que D es un punto de la bisectr z del BAC , entonces D equidista de los lados de dicho angulo, sea pues H AC talque DH AC y DH = DB = 8 entonces,

108

1.9 Problemas resueltos.

1 Geometr a

aplicando el teorema de pit agoras en el CDH se deduce que HC = 6, por lo que AC = x+6, y por el teorema de pit agoras en el ABC , x2 + 182 = (x + 6)2 , por lo tanto, x = 24. 2. Sea ABCD un paralelogramo. Q es el punto medio de AD, F el pie de la perpendicular por B sobre QC . Probar que AF = AB . Soluci on: Sea E el punto medio de BC y G la intersecci on de AE con BF . Como AE CQ, se tiene que AG BF . Pero tambi en, como AE CQ, entonces EG CF por lo que en el BCF , EG es base media. Entonces BG = GF de donde se sigue que ABF es is osceles porque BG es altura y mediana. 3. Sea ABCD un cuadril atero tal que AB = CD. Las mediatrices de AC y BD se cortan en P . Probar que P AC = P CA = P BD = P DB . Figura 144. Soluci on: Como P est a sobre las mediatrices de AC y BD, P A = P C y P B = P D, y por hip otesis, AB = CD, entonces por criterio LLL, ABP CDP . De aqu , AP B = CP D, entonces BP D = AP D + AP B = AP D + CDP = AP C ; por lo tanto, BP D CP A, dada la igualdad anterior y el hecho que son tri angulos is osceles. De esta semejanza se obtiene P AC = P CA = P BD = P DB .

Figura 144 4. ABC es un tri angulo y P un punto en su interior. Sean A , B y C las reexiones de P sobre BC , CA y AB , respectivamente. D, E y F son los pies de las perpendiculares respectivos desde A, B y C hacia B C , C A y A B . Probar que AD, BE y CF son concurrentes. Figura 145. Soluci on: Por propiedades de reexi on axial AC = AP = AB , por lo que el AB C es is osceles, y entonces AD es mediatriz de B C . An alogamente, BE es mediatriz de C A , mientras que CF es mediatriz de A B . Por lo tanto, las rectas AD, BE , CF concurren en el circuncentro del A B C . 5. (Arnoldo Aguilar) En la gura 146, ABGH , BCF G y CDEF son cuadrados. Si I es el centro de ABGH y J = DH BG, demuestre que I , J y F est an alineados. 109

1.9 Problemas resueltos.

1 Geometr a

Figura 145

Figura 146 Soluci on: Como G es punto medio de HF , BG es una mediana del BF H . Adem as, BDF H es un paralelogramo, luego sus diagonales BF y DH se cortan en su punto medio, digamos K . Se sigue que HK es tambi en una mediana del BF H , y en consecuencia el punto de corte de J = KH BG es el centroide del BF H . Pero I es el punto medio de BH , as que F I es la tercera mediana del BF H , por lo tanto J est a sobre el segmento F I . 6. (Arnoldo Aguilar) Sea ABC un tri angulo equil atero. M y N son los puntos medios de AB y BC , respectivamente. Exteriormente al ABC se construye un tri angulo rect angulo is osceles AP C , con AP C = 90 . Si I es la intersecci on de AN y M P , demuestre que CI es la bisectriz de ACM .

Soluci on: Observe que AN es bisectriz del BAC . Como AP C = BM C = 90, el cuadril atero AP CM es c clico, por lo que P M C = P AC = P CA = P M A = 45, entonces M P es bisectriz del AM C . De aqu se concluye que I = M P AN es el incentro del ACM , por lo que CI es bisectriz del ACM .

110

1.9 Problemas resueltos. 7. En la gura 147, el ferencia?

1 Geometr a

ABC es tal que A = 90 y B = 60. Cu al es el radio de la circun-

Figura 147 Soluci on: Por relaciones de tri angulos notables, BC = 2 y CA = 3. Sean P y Q las proyecciones de O sobre AB y AC respectivamente; por construcci on, AP OQ es un rect angulo, pero como OP = r = OQ, es tambi en cuadrado, por lo que AP = r. Observe que la circunferencia es el exc rculo del ABC , por lo que AP = s y entonces r = s 1+2+ 3 = 2 3+ 3 = 2 8. Dado el paralelogramo ABCD, sea M el punto medio de AB , y N la intersecci on de CD con la bisectriz interna del ABC . Demuestre que M C BN si y s olo si AN es bisectriz del DAB . Soluci on: () Si suponemos que M C BN entonces BN es mediatriz de M C , y como BM CN entonces CBN = M BN = CN B = M N B , esto implica que BC M N , y por tanto N es punto medio de CD; as , AM N D es un rombo y AN es bisectriz del DAM . () Si suponemos que AN es bisectriz del DAB , es propiedad conocida que AN BN , por lo que M es el circuncentro del ABN y por la relaci on entre a ngulo central y angulo inscrito se tiene AM N = 2ABN = ABC , por lo tanto M N BC y BCN M es un rombo, de donde se obtiene M C BN . 9. En el ABC , se sabe que los v ertices B , C , el circuncentro O y el ortocentro H del est an todos sobre una misma circunferencia. Figura 148. a) Calcule el valor de A. b) Demuestre que el incentro tambi en pertenece al circunc rculo de BCOH . 111 ABC

1.9 Problemas resueltos. Soluci on:

1 Geometr a

a) Sea BAC = . Como O es el circuncentro del ABC , tenemos que BOC = 2. Por otra parte, sabemos que al ser H ortocentro, se cumple que BHC = 180 . Ahora bien, la condici on de que B , C , H y O son conc clicos implica que BOC = BHC , de donde 2 = 180 , y por tanto = 60 .

Figura 148 b) Este problema se basa en el siguiente resultado: si I es el incentro del ABC entonces A . Como en este caso A = 60, entonces BIC = 120 = BOC = BIC = 90 + 2 BHC , por lo que B , C , O, H , I , se ubican sobre una misma circunferencia. 10. Sea ABC un tri angulo tal que las medianas respectivas a B y C son perpendiculares. Demuestre que se cumple la relaci on (Ver gura 149). 5BC 2 = CA2 + AB 2 .

Figura 149 Soluci on: Sean BB y CC las medianas que son perpendiculares, y sea G el centroide. Observe que el cuadril atero BCB C tiene diagonales perpendiculares; por el teorema de

112

1.9 Problemas resueltos. Pit agoras se cumple BC 2 + B C 2 = C B 2 + B C 2 2 2 BC AB AC BC 2 + = + 2 2 2 2 2 2 5BC = AB + AC


2

1 Geometr a

11. Sea ABC un tri angulo de ortocentro H . Sean P y Q los pies de las perpendiculares desde H a las bisectrices interior y exterior de A, respectivamente. Si M es el punto medio de BC , mostrar que P , Q y M est an alineados. Figura 150. Soluci on: Sean E y F los pies de las alturas trazadas desde a B y C , respectivamente. Se sabe que AP AQ, por lo que AP HQ es un rect angulo. Como AP H = AQH = AEH = AF H = 90, los puntos P , Q, E , F , pertenecen a una circunferencia de di ametro AH . Adem as, en esta circunferencia, como AP y AQ son bisectrices (interior y exterior, respectivamente) del EAF , P y Q son los puntos medios del arcos EF , por lo que P Q es la mediatriz de EF . Por otra parte, como BEC = BF C = 90, el cuadril atero BCEF es c clico, y el circuncentro es M , por lo que M E = M F ; entones M est a en la mediatriz de EF , la cual es P Q.

Figura 150 12. En un tri angulo ABC , sea M el punto medio de BC . Si se cumple que AB = AC y adem as M AC + ABC = 90 , hallar BAC . Figura 151. Soluci on: Sin p erdida de generalidad, suponga que AB > AC . Sea N la intersecci on de AB con la mediatriz de BC . Se forma el BCN que es is osceles, entonces CN M = 90 M CN = 90 M BN = CAN , lo cual implica que el cuadril atero ACM N es c clico. Por lo tanto, BAC = BM N = 90.

113

1.9 Problemas resueltos.

1 Geometr a

Figura 151 13. Sea ABC un tri angulo y U un punto de su circunc rculo tal que AU es bisectriz. Las mediatrices en AB y AC cortan a AU en X y Y . Sea T la intersecci on de BX con CY . Demostrar que AU = T B + T C . Figura 152. Soluci on:43 Como X y Y pertenecen a las mediatrices de AB y AC , respectivamente, y a la bisectriz AU , entonces ABX y ACY cumplen ser is osceles y semejantes entre si, porque XBA = XAB = Y AC = Y CA = . Esto implica T XY = XBA + XAB = 2 = Y AC + Y CA = T Y X , es decir, el T XY es is osceles con T X = T Y . Por otra parte, como ABU C es c clico, U BC = U AC = U AB = U CB = . De aqu se concluye que U BC es is osceles, con U B = U C . Adem as, XU B = ACB = Y CU y XBU = ABC = Y U C ; por criterio ALA, U XB CY U , por lo que BX = Y U . Finalmente, T B + T C = (BX T X ) + (CY + T Y ) = Y U + AY = AU .

Figura 152 14. (The 59th Romanian Mathematical Olympiad Final Round) Sea ABCD un rect angulo de centro O con AB = BC . La perpendicular en O a BD corta a las l neas AB y BC en los puntos E y F , respectivamente. Sean M y N los puntos medios de los segmentos CD y DA, respectivamente. Probar que las l neas rectas F M EN .
43

El caso cuando AB = AC es trivial, porque X , Y y T colapsan en el circuncentro del

ABC .

114

1.9 Problemas resueltos.

1 Geometr a

Soluci on: Considere la gura 153, sin p erdida de generalidad, se ha supuesto AB < BC .44 Sea L el punto medio de AB , y H es la intersecci on de EF con AD. Se tiene que LN BD, y como BD EF entonces LN EF ; adem as, como ABCD es un rect angulo, DA AB , por lo tanto, H es el ortocentro del ELN , y as , LH EN . Por otra parte, las reexiones de L y H con respecto a O son respectivamente M y F , por lo que LH M F , lo cual implica que F M EN .

Figura 153 15. Sea ABC un tri angulo rect angulo, con A = 90 . Sea D un punto en su interior tal que DAC = DCA = DBC = , y AC = BD. Determine el valor de . Figura 154. Soluci on: Sean P y Q los pies de las perpendiculares trazadas desde D hacia CA y AB , respectivamente, R es un punto sobre BC tal que DB DR, y E es la intersecci on de CD con AB . Como el ACD es is osceles, P es punto medio de AC , entonces AC = 2P A = 2DQ = BD, por lo que el BDQ es un tri angulo notable y DBQ = 30. Por otra parte, por criterio ALA, ACE DBR, por lo que CE = BR; como P D AE , D es punto medio de CE ; as , si M es el punto medio de BR (y circuncentro del BDR) se cumple que DC = RM = DM , por lo que el CDM es is osceles. Por la relaci on entre el angulo inscrito y el a ngulo central DM R = 2DBR, por lo tanto DCR = 2. Sumando los a ngulos internos del ABC se tiene A + B + C = 90 + 30 + + 3 = 180, lo cual implica = 15. 16. Sea ABC un tri angulo y M un punto tal que M AB = 10, M BA = 20, M AC = 40 y M CA = 30. Probar que el ABC es is osceles. Figura 155. Soluci on: Sea D la reexi on del punto A con respecto a la recta BM . Entonces el AM D es is osceles con AM D = 2 (M AB + ABM ) = 60 y por lo tanto es equil atero. Tambi en DBA = 2M BA = 40 y como BAC = 50, implica que DB AC . Sea E la intersecci on de BD con CM , se cumple que CED = 90 ACE = 60 = M AD, por lo que el cuadril atero AM ED es c clico. De aqu , DEA = DM A = 60. Como DEC = DEA y ED AC , se tiene que ED es bisectriz y altura en el AEC , por lo tanto ED es mediatriz de AC , lo cual implica que BA = BC .
44

El otro caso es completamente an alogo.

115

1.9 Problemas resueltos.

1 Geometr a

Figura 154

Figura 155 17. Teorema de Poncelet: Demuestre si entonces 2(r + R) = b + c. ABC es un tri angulo rect angulo con A = 90,

Figura 156: Teorema de Poncelet Soluci on: Sean O e I el circuncentro y el incentro del ABC . Como A = 90, O es el punto medio de BC , por lo que a = 2R. Por otra parte, si P y Q son las proyecciones de I sobre AB y AC , claramente AP IQ es rect angulo, pero como I es incentro IP = r = IQ, 116

1.9 Problemas resueltos.

1 Geometr a

por lo que AP IQ es cuadrado. Se sabe que para un tri angulo cualquiera AP = s a, por lo tanto r = sa b+ca r = 2 2r + a = b + c 2(r + R) = b + c 18. En la gura 157, ABCD y P QRS son cuadrados, ABP BCQ CDR DAS y los los radios de las cinco circunferencias son iguales a r. Si a es el lado del cuadrado ABCD, determine r en funci on de a.

Figura 157 Soluci on: Se tiene AB = a y se denen b = AP y c = BP ; observe que por las congruencias BQ = b, por lo que P Q = c b = 2r. Por otra parte (an alogamente a la demostraci on del teorema de Poncelet), al calcular el inradio del ABP se tiene que 2r = b + c a, entonces c b = b + c a, lo cual implica que a = 2b. Por lo tanto, el ABP es un tri angulo notable de 30, 60, 90, y as r = cb 2 3 a 1 a 2 = 2 2 31 = a 4

19. Recta de Euler. El centroide G, el ortocentro H y el circuncentro O de un tri angulo est an alineados, y adem as GH = 2GO. Soluci on: Considere la siguiente gura. Sean AHa y BHb alturas, OA y OB Observe que HA BC y OA BC , por lo que HA OA ; an alogamente tambi en, por el teorema de la base media AB A B y AB = 2A B . Esto ABH A B O y la raz on de semejanza es 2; en particular AH = 2A O. Si 117 mediatrices. HB OB ; implica que denimos G

1.9 Problemas resueltos.

1 Geometr a

Figura 158: Recta de Euler como la intersecci on de la mediana AA con HO, claramente AHG A OG y la raz on de semejanza es la misma que la anterior, por lo que GA = 2GA , i.e., G es el centroide del ABC . Esto implica que el ortocentro, el centroide y el circuncentro de un tri angulo est an alineados, y por la semejanza GH = 2GO.

118

2 Trigonometr a

2.

Trigonometr a

La trigonometr a es el estudio de las relaciones entre los lados y los angulos de los tri angulos. La palabra trigonometr a se deriva de las palabras griegas trigono ( o), que signica tri angulo, y metro ( ), que signica medici on. Aunque los griegos antiguos, como Hiparco y Ptolomeo, usaban trigonometr a en su estudio de astronom a entre el 150 A.C. y el 200 D.C., su historia es mucho m as antigua. Por ejemplo, el escriba egipcio Ahmes registr o algunos c alculos trigonom etricos rudimentarios (acerca de las proporciones de los lados de las pir amides) en el famoso Papiro de Rhind cerca del 1650 A.C. Incluso Ahmes dijo haber copiado el papiro de un trabajo a un m as antiguo que posiblemente data de alrededor del 3000 A.C. Iniciaremos nuestro estudio de la trigonometr a con el concepto de angulo como rotaci on de un segmento de recta alrededor de un punto, y c omo esto lleva naturalmente a estudiar las relaciones trigonom etricas dentro de un c rculo. Note que aunque se comienza la discusi on con el concepto elemental de a ngulo, se asume que el lector est a familiarizado con el trabajo con tri angulos (clasicaci on, terminolog a, rectas notables, angulos dentro de un tri angulo, area, per metro, teorema de Pit agoras) y que tiene nociones b asicas de trigonometr a en tri angulos rect angulos.

2.1.

Angulos.

Imag nese una l nea recta, comenzando de una posici on ja OA (Fig. 159a) y rotando alrededor de un punto O en la direcci on opuesta al movimiento de las agujas del reloj hasta llegar a la posici on indicada por OB . Al rotar de OA a OB , se describe el a ngulo AOB . De esta manera, tenemos el concepto de un a ngulo como formado por la rotaci on de un segmento de recta alrededor de un punto jo, que llamaremos el v ertice del angulo. Como se puede observar en la gura 159a, si se toma cualquier punto C del segmento de recta OA, este punto marcar a con su movimiento el arco CD de un c rculo. No existe l mite en la rotaci on de OA y, por consiguiente, se pueden formar angulos de cualquier magnitud al rotar un segmento de recta de esta manera. Suponga, por ejemplo, que la rotaci on de OA se contin ua hasta la posici on OA (Fig. 159b), en los que A , O y A son colineales. El punto C habr a delineado un semic rculo y el angulo A OA se le llama angulo llano. Si dejamos que OA continue su rotaci on hasta que llegue a su posici on original, habr a realizado una rotaci on completa y el punto C habr a marcado la circunferencia de un c rculo.
B

(a)

(b) Angulo Llano

Figura 159 119

2.1 Angulos.

2 Trigonometr a

II A A A

I A

D III

O IV

(a) Rotaci on completa

(b) Angulo recto

Figura 160 2.1.1. Medici on de un angulo

Por convenci on, los angulos se miden en sentido antihorario. Cualquier angulo formado a partir de la rotaci on de un segmento sobre un punto en el sentido de las agujas del reloj tendr a una medici on negativa. El concepto de la formaci on de un a ngulo por rotaci on nos lleva a un m etodo conveniente de medir a ngulos. Si imaginamos que la rotaci on completa se divide en 360 divisiones iguales, tendremos 360 peque nos angulos iguales, cada uno de los cuales ser a llamado un grado 45 . Note que esta medici on de a ngulos, aunque com unmente utilizada, no es la u nica existente. Los gradianes, por ejemplo, dividen al c rculo en 400 partes iguales. Esta medici on fue adoptada en Francia para que el sistema de medida de a ngulos fuese consistente con otros sistemas de medici on m etricos. Un tercer m etodo para medir a ngulos es absoluto, en el sentido que no depende de una divisi on arbitraria del c rculo formado B por una revoluci on completa. Para obtener la medici on, t omese un c rculo con centro O (ver gura 161). Rote el radio OA hasta la posici on OB , tal que la longitud del arco AB es igual a la del = 1 rad A radio. A la medida del a ngulo AOB le llamaremos un radi an. O Note que la descripci on anterior nos dice que la medici on en radianes de un angulo est a denida como la raz on de la longitud del arco denido por la rotaci on que describe al a ngulo y la longitud del radio del c rculo. Dado que la cirfunferencia se obtiene con 2r, donde r es el radio del c rculo, una revoluci on completa Figura 161 equivale a un a ngulo de 2 radianes.
Puede que se est e preguntando el porqu e el 360 como el n umero de divisiones de una rotaci on completa para obtener un grado. La selecci on de este n umero fue hecha de forma muy temprana en la historia de la civilizaci on humana y sabemos, por ejemplo, a partir de las inscripciones en templos que el n umero fue empleado por los antiguos Babilonios. El n umero probablemente surgi o de la divisi on de los cielos por astr onomos antiguos en 360 partes, correspondientes a lo que se tomaba como el n umero de dias en el a no.
45

120

2.2 Seno y Coseno 2.1.2. Terminolog a

2 Trigonometr a

La gura 160b representa una rotaci on completa, como la mostrada en la gura 160a. Sea E y F los puntos medios de los arcos entre C y D en cada semic rculo. Si trazamos el segmento de recta EF , este pasa a trav es de O y la circunferencia queda dividida en cuatro partes iguales. Cada uno de los a ngulos COE , EOD, DOF y F OC representa a un cuarto de la rotaci on completa y son llamados angulos rectos, conteniendo cada uno 90 grados. El c rculo, adem as es dividido en cuatro partes iguales llamadas cuadrantes que se numeran en el orden de formaci on. Cualquier angulo menor que 90 grados es llamado agudo y cualquiera que se encuentre entre 90 y 180 grados es llamado obtuso. Dos angulos cuyas medidas suman 180 y 90 grados son llamados a ngulos suplementarios y complementarios, respectivamente. Dos a ngulos que tienen la misma medida son llamados congruentes.

2.2.

Seno y Coseno

Como en nuestra discusi on de la secci on anterior, estamos acostumbrados a denir las relaciones del seno y coseno dentro de un tri angulo rect angulo. As , tenemos que:
P

cos = sen =

OA OP AP OP

O A

Aunque estas deniciones son importantes en s mismas y ser an utilizadas ampliamente en las siguientes secciones, est an limitadas por 0 < < . Esto se debe a que dentro de un tri angulo 2 cualquiera, la suma de los a ngulos internos debe ser 180 o radianes, lo que obliga a que la suma de los angulos que se utilizan para la denici on del seno y coseno en un tri angulo rect angulo debe ser 2 radianes. Para la discusi on subsecuente, se considerar a a los angulos dentro del c rculo unitario (es decir, aquel formado por una rotaci on completa de un punto a una distancia de 1 unidad del punto de rotaci on). Asumiremos que el centro del c rculo (es decir, el punto de rotaci on) est a en el or gen 2 2 O del plano cartesiano, por lo que la ecuaci on de la circunferencia es a + b = 1. Sea P un punto dentro de la circunferencia, el angulo IOP , con 0 < < , A y B las proyecciones 2 de P sobre los ejes x e y , respectivamente, como se muestra en la gura 162a. Utilizando las deniciones antes mencionadas para el seno y el coseno, dado que OP = 1 (es el radio del c rculo), tenemos que cos = OA y sen = OB . Es decir, la coordenada en x del punto P corresponde al coseno de , mientras que su coordenada en y corresponde al seno de . Si ahora suponemos que P es un punto cualquiera de la circunferencia, toma cualquier valor entre 0 y 2 ; a partir de lo discutido en el p arrafo anterior, se generalizar an las razones trigonom etricas de forma que, para cualquier a ngulo descrito por la rotaci on de alg un punto P (x, y ) en el c rculo unitario, dado que la posici on inicial de P es (0,1), tendremos que x = cos y y = sen (ver gura 162b).

121

2.2 Seno y Coseno

2 Trigonometr a

J
J B P(cos , sin )

P (cos , sin )

O A I

(a)

(b)

Figura 162 De esta observaci on ya podemos extraer algunos resultados b asicos importantes: Dado que la ecuaci on de la circunferencia es x2 + y 2 = 1, tenemos que cos2 + sen2 = 1 Dado que los valores de x e y para los puntos en esta circunferencia pueden tomar valores entre -1 y 1, inclusive, tenemos que para cualquier angulo , 1 cos 1 y 1 sen 1

Si se realiza una rotaci on completa a partir de P , se llega al mismo punto y notamos que los valores del seno y coseno del nuevo a ngulo descrito por la rotaci on son los mismos que el a ngulo original. Esto sigue siendo cierto sin importar cu antas rotaciones completas se realicen. De aqu tenemos que, para cualquier entero k : cos( + 2k ) = cos() y sen( + 2k ) = sen()

La siguiente tabla resume algunos valores del seno y coseno de a ngulos agudos calculados a partir de guras geom etricas conocidas: cos sen 0 1 0 6 3 2 1 2 4 2 2 2 2 3 1 2 3 2 2 0 1
0 2 3

4 6

122

tricas 2.3 Identidades Trigonome

2 Trigonometr a

P
4

P
6 6

H1 H

1
4

1 h
3

a
(a) Para
4

H
(b) Para
3

I
y
6

Figura 163: C alculo de seno y coseno para a ngulos b asicos , los valores son Es importante saber c omo se obtienen los valores de estos a ngulos. Para 0 y 2 obtenidos directamente de los valores de las coordenadas x e y de los puntos sobre la circunferencia que los describen. Para (g. 163a), notamos que el tri angulo rect angulo es is osceles (ambos angulos agudos deben 4 2 1 sumar 2 ) y por el teorema de pit agoras, a = 2 o , equivalentemente, a = 2 . Ya que a corresponde a las coordenadas x e y del punto P (asumiendo que O se encuentra en el origen), tambi en corresponde al seno y coseno de 4 . y notamos que los a ngulos son complementarios y que el Para encontrar el seno y coseno de 3 6 tri angulo rect angulo que los contiene es la mitad de un tri angulo equil atero (ver g. 163b). Por geometr a sabemos que la altura en un tri angulo equil atero corresponde tambi en a la bisectriz y 1 mediatriz. Por lo tanto, ya que OP = 1, sabemos que OH = (por ser P H mediatriz), de donde 2 3 3 1 el cos 3 = sen 6 = 2 . Luego, por pit agoras, la altura P H = 2 , de donde sen 3 = cos 6 = 2 .

2.3.

Identidades Trigonom etricas

Una Identidad es una igualdad que es v alida para todos los valores que pueda tomar la variable (o las variables, si hay m as de una).46 En Trigonometr a hay muchas identidades y en esta secci on 2 obtendremos algunas de las m as conocidas. 2.3.1. Las identidades b asicas
sin cos

Para obtener identidades trigonom etricas a partir de nuestra denici on de seno y coseno de un a ngulo dentro del c rculo unitario, primero observamos que los puntos de dicho c rculo asociados a , , + y son los v ertices de un rect angulo, suponiendo que los ejes de coordenadas son los ejes de simetr a, como se muestra en la gura 164. De esta observaci on, podemos hacer una
46

En cambio, una Ecuaci on es una igualdad que s olo es v alida para ciertos valores de la variable, llamados Figura 164 Soluciones de la ecuaci on, y la variable toma el nombre de inc ognita.

123

tricas 2.3 Identidades Trigonome lectura directa de la gura para obtener las siguientes identidades: cos() = cos cos( ) = cos cos( + ) = cos sen() = sen sen( ) = sen sen( + ) = sen

2 Trigonometr a

sin

Si ahora se realiza la construcci on para obtener angulos complementarios dentro de la circunferencia, como se muestra en la gura 165, por congruencia de tri angulos obtenemos las siguientes identidades: = sen 2 sen = cos 2 cos 2.3.2. + = sen 2 sen + = cos 2 cos

cos

Figura 165

Identidades de suma y resta de angulos

Las siguientes identidades a obtener ser an aquellas que nos permitan obtener el valor del seno y coseno de la suma + y de la diferencia en funci on del seno y coseno de y de . Encontraremos primero una identidad para sen( + ). Sea P el punto sobre el c rculo unitario que describe al angulo a partir del eje de coordenadas x, y Q el que describe al a ngulo a partir del punto P , como se muestra en la gura 166. Sea C el punto donde cae la perpendicular a OP que pasa por Q. Sean B , D y A los puntos en OI donde caen las perpendiculares a OI que pasan por los puntos Q, C y P , respectivamente. Finalmente, sea EC paralela a OI . Los siguiente datos pueden ser extraidos de esta construcci on: OQ = 1. QC = sen y OC = cos . Ya que DCEB es un palalelogramo, EB = CD. sen =
CD OC

P E O B D A I C

Figura 166

CD , cos

de donde CD = sen cos = EB .

Note que para obtener el valor del sen( + ), es necesario conocer el valor de QB . De los datos anteriores obtuvimos el valor de EB , por lo que resta u nicamente el valor de QE . Para esto, note que por angulos alternos internos entre paralelas ECO = . Pero QCE + ECO = 2 y tambi en EQC + QCE = , de donde CQE = ECO = . Sabiendo esto, tenemos que 2 QE cos = QE = , de donde QE = cos sen . As , QC sen sen( + ) = QB = EB + QE = sen cos + cos sen 124

tricas 2.3 Identidades Trigonome

2 Trigonometr a

A partir de esta identidad pueden deducirse las mencionadas en el siguiente teorema, cuyas demostraciones se dejan al lector en forma de ejercicios. Teorema 2.1. Para cualquier pareja de angulos y se cumple que

sen( + ) sen( ) cos( + ) cos( )

= sen cos + cos sen = sen cos cos sen = cos cos sen sen = cos cos + sen sen

El siguiente ejemplo muestra una aplicaci on de las identidades anteriores. EJEMPLO 2.1 . Calcule el valor del seno y el coseno para el a ngulo 12 Soluci on: Note que 3 4 = 12 , por lo que podemos aplicar las identidades para la diferencia de y . Para el valor del coseno: dos angulos y los valores ya conocidos para el seno y coseno de 4 3

= cos 12 3 4 = cos cos + sen sen 3 4 4 3 1 2 3 2 = + 2 2 2 2 2 = ( 3 + 1) 4 2 De manera similar obtenemos que sen 12 = sen = 31 . 3 4 4 cos 2.3.3. Identidades de duplicaci on y linearizaci on

De las f ormulas obtenidas en la secci on anterior, tenemos el caso especial cuando = , de donde: cos(2) = cos2 sen2 y sen(2) = 2 sen cos

Si utilizamos la relaci on cos2 + sen2 = 1 obtenemos: cos(2) = 2 cos2 1 y cos(2) = 1 2 sen2

De donde es posible expresar al cos2 y al sen2 en funci on de cos(2): cos2 = 1 + cos(2) 2 y sen2 = 1 cos(2) 2

EJEMPLO 2.2 Calcule el seno y coseno de . 8 125

tricas. 2.4 Tabla de Identidades Trigonome Soluci on. Ya que conocemos el valor del coseno de 2 de linearizaci on con = . Ya que cos = 8 4
2 : 2

2 Trigonometr a = , podemos utilizar las identidades 8 4

2 2 1 + 1 2 + 2 2 2 2 2 2 2 = = y sen = = cos 8 2 4 8 2 4 son n umeros La posici on en el c rculo unitario del punto asociada con 8 muestra que cos 8 y sen 8 positivos. En consecuencia,

cos = 8

2+ 2

sen = 8

2 2

2.4.

Tabla de Identidades Trigonom etricas.


Identidad Pitag orica Identidad del Seno de una Suma Identidad del Coseno de una Suma Identidad de la Tangente de una Suma Identidad del Producto de Senos Identidad del Producto de Cosenos Identidad del Producto Seno-Coseno Identidad de Angulo Doble Identidad del Angulo Mitad Identidad de la Suma de Senos Identidad de la Resta de Senos Identidad de la Suma de Cosenos Identidad de la Resta de Cosenos

sen2 + cos2 = 1 1 + tan2 = sec2 1 + cot2 = csc2 sen( + ) = sen cos + cos sen sen( ) = sen cos cos sen cos( + ) = cos cos sen sen cos( ) = cos cos + sen sen tan + tan tan ( + ) = 1 tan tan tan tan tan ( ) = 1 + tan tan cos ( ) cos ( + ) sen sen = 2 cos ( + ) + cos ( ) cos cos = 2 sen ( + ) + sen ( ) sen cos = 2 sen(2) = 2 sen cos cos(2) = cos2 sen2 = 1 2 sen2 = 2 cos2 1 1 cos sen2 = 2 2 1 + cos 2 cos = 2 2 A+B AB sen A + sen B = 2 sen cos 2 2 A+B AB sen A sen B = 2 cos sen 2 2 A+B AB cos A + cos B = 2 cos cos 2 2 A+B AB cos A cos B = 2 sen sen 2 2 126

tricas en un tria ngulo. 2.5 Relaciones trigonome

2 Trigonometr a

2.5.

Relaciones trigonom etricas en un tri angulo.

Para la discusi on de esta secci on, usaremos las siguientes notaciones y convenciones para un tri angulo ABC dado: a, b, c: las longitudes de los lados opuestos a los v ertices A, B y C , respectivamente. S : el a rea del tri angulo cos A y sen A: el cos y sen , donde es la medida entre 0 y de BAC .
A A c b a B B C C

El siguiente teorema, que el lector demostrar a en su totalidad en los problemas 2.2 y 2.3, muestra las relaciones trigonom etricas m as importantes dentro de un tri angulo: Teorema 2.2. En un tri angulo ABC cualquiera, se cumple: La generalizaci on del teorema de Pit agoras (ley de cosenos): a2 = b2 + c2 2bc cos A El area del tri angulo: 1 S = bc sen A 2 Ley de senos: a sen A = b sen B = c sen C = abc 2S

2.6.

Una tangente especial.

Otra de las razones importantes estudiadas por la trigonometr a es la tangente, la cual es denida dentro de un tri angulo rect angulo como:
P

tan =

PA OA

O A

Como mencionamos en el caso del seno y el coseno, esto se cumple para todo a ngulo 0 < < . 2 A partir de esta relaci on y tomando en cuenta la denici on del seno y coseno de , es posible demostrar que para todo angulo = 2 + k (k Z): tan = sen cos

La siguiente tabla muestra los valores de la tangente para algunos angulos comunes:

127

tricas. 2.7 Ecuaciones trigonome

2 Trigonometr a

T (1, tan )

T (1, tan )

I O O I O I

T (1, tan )

(a)

(b)

(c)

Figura 167

tan

0 0

6 3 3

4 1

3 3

2 no denido

Los valores de esta tabla pueden obtenerse a partir de los valores correspondientes del seno y coseno. Una interpretaci on interesante de la tangente se obtiene al representarla dentro del c rculo unitario. Sea A un punto cualquiera de la circunferencia que describa al a ngulo y l la recta tangente al c rculo unitario que pasa por el punto (1, 0), entonces el punto T de la intersecci on de la recta determinada por AO y l tendr a coordenadas (1, tan ). La gura 167 muestra lo aqu descrito para algunos valores de . no existe: Note que esta interpretaci on de la tangente nos da una raz on intuitiva por la que tan 2 cuando A dene al angulo 2 , l y la recta denida por OA son paralelas, por lo que nunca se intersectan. De la gura 167 tambi en es posible extraer una explicaci on intuitiva para las siguientes identidades: tan( + k ) = tan y tan() = tan

donde k Z. As mismo, de las identidades del seno y coseno vistas en las secciones anteriores es posible obtener las identidades: 1 + tan2 = 1 cos2 y tan 1 = 2 tan

2.7.

Ecuaciones trigonom etricas.

Como se mencion o anteriormente, a diferencia de las identidades, que se cumplen para cualquier valor de las variables en cuesti on, las igualdades en las ecuaciones se cumplen s olo para algunos valores de las variables y resolverlas implica encontrar dichos valores. En ecuaciones trigonom etricas, la principal estrategia es utilizar las identidades antes vistas para simplicar lo m as posible la igualdad. Por ejemplo, para la siguiente ecuaci on: 2 sen(x) 1 = 0 128

tricas. 2.7 Ecuaciones trigonome Al sumar 1 a ambos lados de la ecuaci on y luego dividir por 2 obtenemos: sen(x) = 1 2

2 Trigonometr a

De los valores obtenidos para sen(x) en la secci on 2.2 . Sin embargo, al resolver una ecuaci on sabemos que x = 6 deben obtenerse todos los valores de la variable que hacen que la igualdad sea verdadera. Para el caso del seno, que en el c rculo trigonom etrico equivale a la coordenada y del punto sobre el c rculo que describe al a ngulo, podemos observar en la gura 168 que x = = 56 tambi en 6 cumple con la igualdad47 . Es decir, tenemos que x= 6 o x= 5 6

Figura 168

Pero hay que recordar, adem as, lo discutido en la secci on 1.2: si desde cualquier punto del c rculo trigonom etrico se realiza una rotaci on completa, se regresa al mismo punto de partida, por lo que el seno del a ngulo descrito originalmente y el del angulo que se forma luego de la rotaci on son exactamente los mismos. As , para la primera de nuestras soluciones, sen( ) = sen( + 2 ), de 6 6 13 donde x = 6 + 2 = 6 es otra soluci on a nuestra ecuaci on. Ya que podemos repetir este mismo proceso de manera indenida (es decir, podemos dar tantas rotaciones completas como nos plazca y siempre llegaremos al mismo punto de partida), podemos decir que a partir de la soluci on x = 6 se pueden obtener innitas soluciones de la forma x = 6 + 2k , donde k es un n umero entero cualquiera. Aplicando esta misma idea a la segunda de las soluciones obtenidas anteriormente, podemos decir ahora que las soluciones de la ecuaci on son: x= + 2k 6 o x= 5 + 2k 6

donde k es un n umero entero. Note que el m etodo utilizado en el ejemplo anterior es similar al que se utilizar a para resolver ecuaciones con coseno o tangente y puede ser descrito en los siguientes pasos: 1. Llevar la ecuaci on a la forma sen(x) = a, cos(x) = a, o tan(x) = a. 2. Encontrar las soluciones entre 0 y 2 de estas ecuaciones simples. Para esto puede utilizar las igualdades: sen(x) = sen( x) cos(x) = cos(x) tan(x) = tan( + x) 3. Utilizar la propiedad peri odica de las funciones para que, a partir de una soluci on espec ca x0 , se pueda obtener la soluci on general x = x0 + 2k , con k entero.
Note que lo que aqu descrito es una aplicaci on de la igualdad sen = sen( ). Si la discusi on se hubiese basado en el coseno, nos hubiese sido u til la identidad cos = cos().
47

129

tricas. 2.7 Ecuaciones trigonome

2 Trigonometr a

A continuaci on se presentan m as ejemplos que utilizan esta estrategia para su soluci on. EJEMPLO 2.3 Resuelva en R la ecuaci on sen x = 23 . Soluci on. Conocemos una soluci on particular de esta ecuaci on: x0 = . La ecuaci on se convierte 3 as en sen x = sen 3 , que tiene como soluciones: x= + 2k 3 y x= 4 + 2k 3

donde k Z. Note que el segundo conjunto de soluciones se obtiene tomando en cuenta la identidad sen = sen ( ). EJEMPLO 2.4 Resuelva en R la ecuaci on cos x = sen . 5 Soluci on. Llevaremos esta ecuaci on a la forma cos x = cos x0 gracias a la relaci on: sen = cos 5 2 5 = cos 3 10

, de donde tenemos la soluci on Por consiguiente, resolvemos la ecuaci on cos x = cos 3 10

x=

3 + 2k 10

(k Z)

Note que dado que cos = cos(), tambi en tenemos como soluciones x= 3 + 2k 10 (k Z)

EJEMPLO 2.5 Resuelva en [0, 2 ] la ecuaci on cos 3x = sen x. x , de donde la ecuaci on propuesta es equivalente a Soluci on. Tenemos que sen x = cos 2 cos 3x = cos 2 x . Obtenemos por lo tanto: 3x = o equivalentemente: x + 2k 2 x= o 3x = x + 2k 2

+k o x = + k 8 2 4 con k Z. De estos valores, aquellos que pertenecen al intervalo [0, 2 ] son: De la primera relaci on: , 8
8

+ , 2

+ ,

3 . 2

Es decir: , 8
3 7 , 4. 4

5 9 9 , 8 , 13 . 8

De la segunda relaci on: , 2 ; es decir, 4 4

130

tricas. 2.7 Ecuaciones trigonome

2 Trigonometr a

EJEMPLO 2.6 Resuelva en [0, 2 ] la ecuaci on 2 sen x cos x = 3 cos(x). Soluci on. Podemos transformar la ecuaci on dada a 2 sen x cos x 3cos(x) = 0, de donde cos x(2 sen x 3) = 0 Esto es cierto cuando cualquiera de los dos t erminos que se est an multiplicando son iguales a 0, por lo que analizamos estos casos por separado. Para cos x = 0, podemos observar que en intervalo dado 3 los valores que cumplen con la ecuaci o n son x = o x = . Para 2 sen x 3 = 0, transformamos 2 2 3 sen(x) = sen . Dentro del intervalo dado, el u nico otro valor la ecuaci on para tener sen(x) = 2 o 3 2 que cumple con esta condici on es x = = . 3 3 3 Combinando los dos casos anteriores, tenemos que las soluciones de la ecuaci on son , , 23 , 2. 3 2

2.7.1.

Ecuaciones de la forma A cos (x) + B sen (x) = C

Note que en todos los ejemplos anteriores se han resuelto ecuaciones de la forma A cos (x) + B sen (x) = C Por lo que ahora describiremos un m etodo general para la soluci on de dichas ecuaciones. Para esto, suponga que un tri angulo rect angulo tiene por catetos A y B , los coecientes de sen (x) y cos (x) 2 2 en la ecuaci on dada; su hipotenusa ser a entonces A + B . Dividimos toda la ecuaci on por este valor, resultandonos A B C cos (x) + sen (x) = . A2 + B 2 A2 + B 2 A2 + B 2 Los coecientes en esta nueva ecuaci on son coordenadas de un punto en la circunferencia unitaria y por lo tanto se pueden identicar con el seno y coseno del a ngulo asociado a dicho punto. Existen dos posibilidades, de acuerdo a qu e coeciente ser a considerado abcisa y cu al de ellos como la ordenada. Por ejemplo, si hacemos A cos = 2 A + B2 B sen = A2 + B 2 la ecuaci on puede ser escrita en la forma: cos () cos (x) + sen () sen (x) = A2 C + B2

De donde las soluciones de la ecuaci on original se reducen a las soluciones de la ecuaci on C cos ( x) = 2 A + B2 131

2.8 Problemas.

2 Trigonometr a

Note que esto nos dice que para que la ecuaci on original tenga soluci on debe cumplirse C 1 1 2 A + B2 Note que la selecci on del sen y cos fue completamente arbitraria. Tambi en pudo haberse establecido que A sen = 2 A + B2 B cos = 2 A + B2 En cuyo caso la ecuaci on original se transforma en sen () cos (x) + cos () sen (x) = sen( + x) = EJEMPLO 2.7 1 . Resuelva en [0, 2 ] la ecuaci on cos (x) + sen (x) = 2 Soluci on. Note que en esta ecuaci on, A = 1, B = 1 y C = C A2 + B 2

De aqu que 2 1 1 cos = = = 2 2 (1)2 + (1)2 1 1 2 sen = = = 2 2 (1)2 + (1)2


3 4

1 . 2

Dentro de [0, 2 ] se tienen dos opciones para en la segunda ecuaci on: y 34 . De estas, s olo 4 cumple con la primera ecuaci on, por lo que la ecuaci on original se convierte en:

de donde

3 4

x=

3 4

1 3 x = 4 2 5 13 x = 3 ; es decir, x = 12 o 12 . cos

2.8.

Problemas.

C rculo trigonom etrico. 1. Complete la siguiente tabla en t erminos de sen y cos


sen cos tan cot sec csc + 2 2 + 2 + 2 + 2 + 2 + 2 = cos = = = = = + sen ( + ) = sen cos ( + ) = tan ( + ) = cot ( + ) = sec ( + ) = csc ( + ) = + 3 2 3 = cos 2 3 + = 2 3 + = 2 3 + = 2 3 + = 2 3 + = 2 sen ( ) = sen cos ( ) = tan ( ) = cot ( ) = sec ( ) = csc ( ) = sen () = sen cos () = tan () = cot () = sec () = csc () = 2 sen 2 cos 2 tan 2 cot 2 sec 2 csc 2

sen + cos tan cot sec csc

sen + cos tan cot sec csc

= cos = = = = =

132

2.8 Problemas. 2. Resuelva las siguientes ecuaciones: a) 2 sen x 1 = 0 b) 4 cos x 2 3 = 0

2 Trigonometr a

c)

2 cos(x + ) = 1 d) 2 sen(3x + ) = 1 2

3. Represente en el c rculo trigonom etrico los valores del a ngulo [0, 2 ] que satisfacen cada una de las siguientes inecuaciones: a) 2 sen x 1 0 b) 4 cos x 2 3 > 0 c) 2 cos(x + ) 1 d) 2 sen(3x + ) 1 2 4. (Verdadero o Falso.) Los siguientes expresiones son todas nulas, para cualquier x. a) cos( x) + cos(x) b) cos( x) + sen(x + ) 2 c) sen( + x) + cos( x) 2 d) sen(x ) cos(x + ) 2 e) sen( x) + cos(x + ) 2 f) cos(x ) + sen(x ) 2

5. Determine cu al de las siguientes expresiones es distinta de sen(). a) cos( ) 2


b) cos( 52 + )

c) sen( )

d) cos( + ) 2

e) sen( + 3 )

6. Emplee el c rculo trigonom etrico para escribir cada una de las expresiones siguientes en t erminos de cos o sen . a) sen( +
2

+ ) + cos(

b) sen(

3 ) cos( ) 2

Identidades b asicas. 7. Determine cos x sabiendo que:

a) sen x =

3 3

y x ] , [. 2

1 b) sen x = 4 y x ] , 0[. 2

8. Determine sen x sabiendo que:


3 y x ] , 0[. a) cos x = 4

b) cos x =

2 5

y x ]0, [.

9. Utilizando identidades trigonom etricas, calcule los valores exactos del seno y coseno de los siguientes a ngulos: 8 3 b) 8 a) 5 8 7 d) 8 c) e) 9 8 3 f) 8 5 8 h) 12 g) 5 12 7 j) 12 i) k) 13 12 11 l) 12

10. Muestre que para todo n umero real x: a) (cos x + sen x)2 + (cos x sen x)2 = 2. b) (1 + sen x + cos x)2 = 2(1 + sen x)(1 + cos x).

133

2.8 Problemas. F ormula de la Adici on. 11. Pruebe que, para todo n umero real x: a) sen x = sen( + x) sen( x). 3 3 b) cos x + cos(x + c) sen x + sen(x +
2 ) + cos(x + 3 2 ) + sen(x + 3 4 ) = 0. 3 4 ) = 0. 3

2 Trigonometr a

12. Demuestre las siguiente identidades: a) cos(x + y ) cos(x y ) = cos2 x sen2 y = cos2 y sen2 x. b) sen(x + y ) sen(x y ) = sen2 x sen2 y = cos2 y cos2 x. c) cos(x + y ) sen(x y ) = sen x cos x sen y cos y . d) cos(x y ) sen(x + y ) = sen x cos x + sen y cos y . F ormula de duplicaci on y linealizaci on. 13. Calcule cos(2x) en cada uno de los siguientes casos.
1 a) cos x = 3

b) sen x =

3 3

c) cos x =

4 5

d) sen x =

3 4

14. Calcule sen(2x) en cada uno de los siguientes casos. a) sen x =


3 5

y x [ , ] 2 2

c) cos x = d) cos x =

1 3 1 2

y x [, 2 ] y x [, 0]

b) sen x = 3 y x [ , 3 ] 5 2 2 15. Exprese en t erminos de cos 2x y sen 2x: a) cos4 x sen4 x b) cos4 x + sen4 x

c) cos4 x

d) sen4 x

16. Si x no es un m ultiplo entero de

sen 3x cos 3x , demuestre que = 2. 2 sen x cos x

Leyes del Coseno y del Seno. 17. Construir un tri angulo cuyos lados miden 11 cm., 15.5 cm., 19 cm. Es un tri angulo rect angulo? De no ser as , determine la medida del a ngulo opuesto al lado m as grande. 18. Suponga que a = BC , b = AC y c = AB , representan los lados del ABC , y que , y , las medidas de los angulos CAB , ABC y BCA, respectivamente. En cada uno de los casos siguientes, aclare cu antos tri angulos satisfacen las condiciones dadas, y luego determine las medidas de los elementos que faltan de los tri angulos soluci on:

134

2.8 Problemas. a) b = 3, c = 4, = 60 b) b = 2, c = 6, = 45 c) b = 5, c = 12, = 90 d) a = 11, b = 9, c = 5

2 Trigonometr a e) a = 15, b = 4, c = 13 f) a = 8, b = 7, c = 8

19. Un tri angulo ABC es tal que BC = 4. Los a ngulos ABC y ACB miden respectivamente y 3 . Determine los valores exactos de los otros dos lados. 4 20. En el tri angulo ABC calcule AB y AC , sabiendo que BC =3.5 cm., el ABC = 55 y el CAB = 80 . 21. Teorema de la Bisectriz: El punto I es el pie de la bisectriz interior del angulo CAB . Aplique la Ley de Senos sobre cada uno de los tri angulos AIB y AIC para demostrar que IB AB = IC AC Demuestre adem as que si I es el pie de la bisectriz externa del a ngulo CAB se cumple IB AB = IC AC 22. En el tri angulo ABC , sea I el punto medio del lado BC , = BAI y = CAI . Demuestre que sen AC = sen AB 23. Muestre que el tri angulo ABC es rect angulo en A si y solamente si sen2 A = sen2 B + sen2 C . F ormula del Area. 24. Calcule el area del cuadril atero ABCD.
B C 2 D 60 4 45

25. Calcule el area del tri angulo ABC , sabiendo que AB = AC = 4 y que el ABC = 50 . Empleo de Identidades Trigonom etricas. 26. Verdadero o Falso.
a) cos2 x + cos2 (x + ) + cos2 (x + 26 ) + cos2 (x + 36 ) + cos2 (x + 42 ) + cos2 (x + 56 ) no depende 2 del valor de x.

b) La ecuaci on sen 2x = 2 sen x es equivalente a cos x = 1. 135

2.8 Problemas. c) La ecuaci on


1 2

2 Trigonometr a cos 2x = cos2 x no tiene soluci on.


1 2

d) En el tri angulo ABC , la longitud de la mediana trazada desde A es: e) En el tri angulo ABC , si sen A sen B sen C entonces a b c. 27. Muestre que:
cos 12 + sen 12 3. = cos 12 sen 12 7 24

a2 + 4bc cos A.

sen 5 sen 7 sen 11 = 1. Sugerencia: 28. Demuestre que 16 sen 24 24 24 24

5 24

24

11 . 24

29. Verique que: 2 sen (cos + cos 37 + cos 57 ) = sen 67 , y deduzca el valor de cos cos 27 + 7 7 7 3 cos 7 .

30. Sea x un n umero real diferente de k (k Z ). a) Muestre que cos x cos 2x cos 4x = b) Calcule cos 2 4 cos cos . 7 7 7 2 4 c) Calcule cos cos cos . 9 9 9 sen 8x . 8 sen x

Trigonometr a del tri angulo rect angulo. 31. En la gura adjunta BH = 1, CH = 3 y ACB = . Cu al es el valor exacto de per metro 6 del tri angulo ABC ?
A

32. De acuerdo con la gura siguiente. Cu al es el valor exacto del sen ?

1 2 1 2
33. En la gura adjunta, ABCD es un cuadrado, I y J son puntos medios. Cu al es el valor exacto de sen ? 136

2.8 Problemas.

2 Trigonometr a

J A B

34. En la gura anexa, OA = OB = OI = 1 y el BOI = . 4

2+ 2 . a ) Calcule OH y AH . Deduzca que cos BAI = 2 AI b ) Qu e puede decir del tri angulo AIB? Deduzca que cos BAI = AI . 2 c ) Calcule el valor del AIB y utilice los resultados de los literales anteriores para probar 2+ 2 = . Determine el valor de sen . que: cos 8 2 8

35. En la gura adjunta, ACDE es un cuadrado de lado 1 y ABC es un tri angulo equil atero: a ) Muestre que: BED =
. 12

b ) Calcule BH y deduzca que: tan 12 =2

3.

c ) Utilice identidades trigonom etricas para calcular cos 12 y sen 12 .

137

2.8 Problemas.

2 Trigonometr a

36. Con la informaci on que se muestra en la gura adjunta, se propone calcular tan . M etodo 1: Calcule el valor exacto de AB , AI , despu es utilice la Ley de Cosenos sobre el tri angulo ACI y determine el valor de cos y sen . M etodo 2: Sean x e y las medidas de BAC y BAI respectivamente. Calcule tan(x y ).
C AC = 12 I A IB = 3 B CI = 3

37. 1 Exprese el a rea del tri angulo is osceles y del cuadrado en funci on de a = OA y del a ngulo = AOB . (Ver gura). 2 Demuestre que estas areas son iguales si, y s olamente si, cos = que la igualdad de areas se traduce en: sen = 4(1 cos ).
15 . 17

Sugerencia: Pruebe

Ecuaciones Trigonom etricas. 38. Resuelva las siguientes ecuaciones: a) cos() = 1 2 b) cos(4) = 3 2 c) sen(2) = 1 2 d) tan(2) = 1.

39. Resuelva las ecuaciones propuestas y represente las soluciones sobre el c rculo trigonom etrico.
a ) sen(3x + ) = sen( 23 x) 3

b ) sen 3x = cos(x ). 6 c ) 2 cos2 x cos x = 0. d ) cos x sen x = cos(4x


2 2 ). 4

f ) cos2012 x sen2012 x = 1. Sugerencia: Pruebe que, cos2012 x 1. g ) sen x + sen 2x + sen 3x = 0. h ) sen x + 2 cos x + sen 3x = 0. i ) cos x + cos 3x = sen x + sen 3x

e ) 2 sen2 x + 3 sen x + 1 = 0. Sugerencia: Haga un cambio de variable.

138

2.9 Problemas aplicados.

2 Trigonometr a
3 ) 2

40. Si ]0, 2 [, el n umero de soluciones de la ecuaci on (2 sen 1)(sen 2 + 4, 5 o 6?

= 0 es 2, 3,

41. Si f (x) = cos x, g (x) = sen x y x [0, 1000 ], cu al es el n umero de puntos de intersecci on de las curvas que representan a f y g ? 42. 1 Sean x y y n umeros reales, demuestre que: tan x = tan y es equivalente a: Existe un n umero entero k tal que x = y + k . 2 Resuelva cada una de las ecuaciones siguientes: b) tan 2x = 1 a) tan x = 3

c) 3 tan2 x = 1

43. Utilizando el m etodo descrito en la secci on 2.7.1, resuelva las ecuaciones trigonom etricas siguientes: 3 a ) 3 cos (x) sen (x) = 1. b ) 2 cos (x) + sen (x) = . c ) 3 cos (x) + sen (x) = 2 2. 44. Determine los valores del angulo que satisface la siguiente inecuaci on: cos(2) < 0. 3 45. Para qu e valores de se satisface que sen(3) > ? 2

2.9.

Problemas aplicados.

PROBLEMA 2.1 Considere las deniciones de secante y cosecante en un tri angulo rect angulo:
P

sec = csc = cot =

OP AP OP OA OA OP

= = =

1 sen 1 cos 1 tan

O A

En la siguiente gura, la recta que pasa por D y C es tangente al c rculo unitario en el punto P .
D P

139

2.9 Problemas aplicados.

2 Trigonometr a

1. Demuestre que C y D tiene coordenadas (sec , 0) y (0, csc ), respectivamente. 2. Demuestre que para todo : a ) 1 sec , o bien sec 1. b ) 1 csc , o bien csc 1. 3. Aplicando el teorema de Pit agoras, demuestre que para todo se cumple que48 : a ) 1 + tan2 = sec2 . b ) 1 + cot2 = csc2 .

PROBLEMA 2.2 tricas en un Tria ngulo: Ley de Senos. Las Relaciones Me Sea ABC un tri angulo; a, b, c sus lados opuestos a los v ertices A, B , C , respectivamente. Sea CH su altura desde el v ertice C . Sean O y R el circuncentro y circunradio del ABC . 1. Verique que el punto H se encuentra: a ) Sobre la semirecta de origen A que contiene al v ertice B , si el angulo A es agudo. b ) Sobre la semirecta de origen A opuesta al v ertice B , si el angulo A es obtuso. c ) En el v ertice A si el a ngulo A es recto. 2. Demuestre que en todos los casos anteriores, el a rea del tri angulo S , viene dada por: 1 S = bc sen A 2 3. An alogamente, considerando las otras alturas del tri angulo verique que: 1 S = ac sen B 2 1 S = ab sen C 2

Deduzca de las relaciones encontradas que: El area de un tri angulo es igual al semiproducto de dos de sus lados y del seno del angulo que estos forman. 4. A partir de los resultados anteriores, deduzca la Ley de los Senos: a sen A = b sen B = c sen C = abc 2S

5. Sea D el otro punto que determina el di ametro con origen B . Argumente por qu e el angulo BDC tiene la misma medida que el a ngulo A. En el tri angulo BDC verique que sen A =
48

a 2R

Siempre que permita que la funci on trigonom etrica que se est a evaluando est e bien denida.

140

2.9 Problemas aplicados. 6. Deduzca de los ejercicios anteriores que: a sen A = b sen B = c sen C = abc = 2R 2S

2 Trigonometr a

7. Demuestre que S tambi en puede ser escrita como: S= abc 4R

PROBLEMA 2.3 tricas en un Tria ngulo: Ley de Cosenos. Las Relaciones Me Considere un tri angulo ABC cuyos a ngulos son A, B y C , y sus lados opuestos a, b, c. Suponga que el a ngulo A es obtuso. 1. Sea H el pie de la altura que sale de B . Utilice el teorema de Pit agoras en el tri angulo rect angulo CHB para demostrar que: a2 = (BH )2 + (b + AH )2 2. En el tri angulo rect angulo AHB , calcule AH y BH en funci on de cos A y sen A.

3. En los resultados del numeral 1 sustituya los resultados obtenidos en el numeral 2 para concluir que a2 = b2 + c2 2bc cos A 4. Analice el resultado en el caso que el angulo A es recto o en el caso que sea agudo. Verique que en ambos casos el resultado es el mismo: a2 = b2 + c2 2bc cos A

PROBLEMA 2.4 Primer Teorema de Ptolomeo: Un cuadril atero convexo ABCD es conc clico si y s olo si AC BD = AB CD + BC DA Una demostraci on de este teorema de basa en la Ley del Coseno; para simplicar la notaci on, establezca AB = a, BC = b, CD = c, DA = d, AC = m, BD = n y ADB = ; adem as, ABCD es conc clico si y s olo si ABC = 180 , si y s olo si m2 = a2 + b2 2ab cos(180 ) m2 = c2 + d2 2cd cos

141

2.9 Problemas aplicados.


B a 180 A n b d D m

2 Trigonometr a

c C

1. Multiplique la primera ecuaci on por cd y la segunda ecuaci on por ab. Sume estas nuevas expresiones y utilizando el resultado cos = cos(180 ), demuestre que m2 = 2. An alogamente, demuestre que n2 = ab + cd (ac + bd) ad + bc ad + bc (ac + bd) ab + cd

3. Demuestre el Primer Teorema de Ptolomeo mn = ac + bd 4. Demuestre el Segundo Teorema de Ptolomeo m ad + bc = n ab + cd

PROBLEMA 2.5 Una demostraci on alternativa de la identidad del seno de la suma de dos a ngulos se basa en la Ley de Senos y el Teorema de Ptolomeo, que establece lo siguiente: Un cuadril atero es conc clico y convexo si y s olo si se cumple que el producto de las diagonales es igual a la suma de los productos de lados opuestos. Considere en particular un cuadril atero conc clico convexo ABCD tal que AC es di ametro y es igual a 1, BAC = , CAD = .

142

2.9 Problemas aplicados.

2 Trigonometr a

D C O A B
1. Demuestre que AB = cos , BC = sen , CD = sen , DA = cos . 2. Aplique el Teorema de Ptolomeo al cuadril atero ABCD para demostrar que BD = sen cos + cos sen . 3. Aplique la Ley de Senos al BCD para demostrar que BD = sen( + ).

4. Concluya que sen( + ) = sen cos + cos sen .

PROBLEMA 2.6 En la discusi on previa demostramos que sen( + ) = sen cos + cos sen . 1. Dado que sen( ) = sen(+( )), deduzca la f ormula sen( ) = sen cos cos sen . 2. Demuestre cos( + ) = cos cos sen sen y cos( ) = cos cos + sen sen , . usando la identidad cos = sen 2

PROBLEMA 2.7 = 2 agudo. Sea ABC un tri angulo is osceles con AB = AC = a y A


A

143

2.9 Problemas aplicados. 1. Calcule BC en t erminos de a y sen .

2 Trigonometr a

2. Sea BH una altura. Calcule BH de dos maneras diferentes y demuestre a partir de estas relaciones que sen(2) = 2 sen cos . 3. Calcule AH y CH y deduzca la relaci on: cos(2) = 1 2 sen2 .

PROBLEMA 2.8 Sea ABC un tri angulo rect angulo en A, O el punto medio de BC y H la proyecci on ortogonal de A sobre BC . Se supone AB > AC , BC = 2a y ABC = .
A

B O H C

1. Calcule AB , AC , BH , CH y AH en funci on de a y . 2. Demuestre que: AOC = 2 y calcule BH y CH en funci on de a y 2. 3. Deduzca cos(2), sen(2) y tan 2 en funci on de cos , sen y tan . 4. Los resultados del ejercicio anterior siguen siendo v alidos cuando AB < AC ?

PROBLEMA 2.9 tricas en un tria ngulo: Fo rmula de Hero n. Las Relaciones Me Sea ABC un tri angulo cuyos a ngulos son A, B y C y sus lados opuestos a, b, c.

1. Utilice la f ormula a2 = b2 + c2 2bc cos A para demostrar el teorema de Pit agoras y su rec proco. 2. Haciendo uso de la f ormula a2 = b2 + c2 2bc cos A y tomando en cuenta que en todo tri angulo no degenerado se tiene que: 1 < cos A < 1 demuestre que |b c| < a < b + c. 3. Demuestre que en todo tri angulo ABC se tiene: b2 c2 = a(b cos C c cos B ). Deduzca de lo anterior que b = c si y s olo si cos B = cos C . Recuerde que (b + c a cos B ) > 0. 4. A partir de la ley de cosenos a2 = b2 + c2 2bc cos A, deduzca que cos A = (b + c a)(b + c + a) 1 2bc 144

2.9 Problemas aplicados.

2 Trigonometr a

5. Utilice la identidad sen2 + cos2 = 1 y el resultado anterior para deducir deduzca que: sen2 A = (a + b + c)(a + b c)(b + c a)(c + a b) 4b2 c2

6. Se denota por s el semiper metro del tri angulo, es decir a + b + c = 2s. Verique que el resultado del ejercicio anterior puede expresarse en la forma sen2 A = (2s)(2s 2c)(2s 2a)(2s 2b) 4s(s a)(s b)(s c) = . 2 2 4b c b2 c 2

7. A partir del resultado del ejercicio anterior y usando la f ormula para calcular el a rea del 1 bc sen A, deduzca la conocida como F ormula de Her on : tri angulo S = 2 S= s(s a)(s b)(s c)

8. Suponga conocidas las longitudes de los tres lados a, b, c de un tri angulo. Calcule en t erminos de a, b y c: a ) Su a rea. b ) Sus tres angulos. c ) El radio de su circunferencia circunscrita. 9. Demuestre las f ormulas de a rea de un tri angulo S = sr = (s a)ra = (s b)rb = (s c)rc donde r es el inradio del tri angulo, y ra , rb , rc , los exradios respecto a A, B , C . 10. Calcule las longitudes del inradio y los exradios en funci on de a, b y c.

PROBLEMA 2.10 2 Considere un tri angulo is osceles ABC , con AB = AC , BC = a y B = . La bisectriz del 5 a ngulo en B corta al lado AC en el punto D.

B
2 5

a D C
145

2.9 Problemas aplicados. 1. Demuestre que los tri angulos 2. Deduzca que DA = DB = a. 3. Demuestre que AB = 2a cos 4. Deduzca que cos 5. Sea x = cos y de y .
5 5 5 2 5 2 5

2 Trigonometr a ABD y BCD son tri angulos is osceles.

y que CD = 2a cos =
1 2

2 5

cos

y cos

cos

2 5

1 =4 . 1 2

y y = cos

. Sabiendo que x y =

y xy = 1 , encuentre los valores de x 4

PROBLEMA 2.11 Se considera un tri angulo ABC rect angulo en A, tal que BC = 2a y B = medio del lado BC y H la proyecci on ortogonal de A sobre BC .

. 8

Sea O el punto

O
2 a. 2

C
2.

1. Demuestre que OH = HA = 2. En el tri angulo rect angulo

Deducir que AB = a 2 +
8

AHB , calcule cos

, sen

PROBLEMA 2.12 Se considera un tri angulo

ABC rect angulo en A, tal que BC = 4a y B =

. 12

A
12

D C

1. Sea O el punto medio del lado BC y D la proyecci on ortogonal de A sobre BC . Demuestre que AD = a y OD = 3a. 2. Encuentre en t erminos de a los segmentos AC y CD. 3. Calcule el seno, coseno y tangente de los a ngulos
12

5 . 12

146

También podría gustarte